*NURSING > TEST BANK > TEST BANK Maternity and Pediatric Nursing 3rd Edition by Ricci, Kyle, and Carman (All Chapters 1-51) (All)

TEST BANK Maternity and Pediatric Nursing 3rd Edition by Ricci, Kyle, and Carman (All Chapters 1-51)

Document Content and Description Below

Test Bank Maternity and Pediatric Nursing 3rd Edition By Susan Ricci, Theresa Kyle, and Susan Carman Contents Chapter 1- Perspectives on Maternal, Newborn, and Women’s ... Health Care 3 Chapter 2- Family-Centered Community-Based Care 7 Chapter 3- Anatomy and Physiology of the Reproductive System 12 Chapter 4- Common Reproductive Issues 18 Chapter 5- Sexually Transmitted Infections 23 Chapter 6- Disorders of the Breasts 27 Chapter 7- Benign Disorders of the Female Reproductive Tract 32 Chapter 8- Cancers of the Female Reproductive Tract 37 Chapter 9- Violence and Abuse 42 Chapter 10- Fetal Development and Genetics 47 Chapter 11- Maternal Adaptation During Pregnancy 52 Chapter 12- Nursing Management During Pregnancy 57 Chapter 13- Labor and Birth Process 62 Chapter 14- Nursing Management During Labor and Birth 68 Chapter 15- Postpartum Adaptations 74 Chapter 16- Nursing Management During the Postpartum Period 80 Chapter 17- Newborn Transitioning 86 Chapter 18- Nursing Management of the Newborn 92 Chapter 19- Nursing Management of Pregnancy at Risk- Pregnancy 98 Chapter 20- Nursing Management of the Pregnancy at Risk 103 Chapter 21- Nursing Management of Labor and Birth at Risk 109 Chapter 22- Nursing Management of the Postpartum Woman at Risk 114 Chapter 23- Nursing Care of the Newborn With Special Needs 120 Chapter 24- Nursing Management of the Newborn at Risk 125 Chapter 25- Growth and Development of the Newborn and Infant 131 Chapter 26- Growth and Development of the Toddler 137 Chapter 27- Growth and Development of the Preschooler 143 Chapter 28- Growth and Development of the School-Age Child 150 Chapter 29- Growth and Development of the Adolescent 157 Chapter 30- Atraumatic Care of Children and Families 164 Chapter 31- Health Supervision 168 Chapter 32- Health Assessment of Children 175 Chapter 33- Caring for Children in Diverse Settings 181 Chapter 34- Caring for the Special Needs Child 188 Chapter 35- Key Pediatric Nursing Interventions 195 Chapter 36- Pain Management in Children 201 Chapter 37- Nursing Care of the Child With an Infectious or Communicable Disorder 208 Chapter 38- Nursing Care of the Child With an Alteration in Intracranial Regulation / Neurologic Disorder 215 Chapter 39- Nursing Care of the Child With an Alteration in Sensory Perception / Disorder of the Eyes or Ears 222 Chapter 40- Nursing Care of the Child With an Alteration in Gas Exchange / Respiratory Disorder 229 Chapter 41- Nursing Care of the Child With an Alteration in Perfusion / Cardiovascular Disorder 235 Chapter 42- Nursing Care of the Child With an Alteration in Bowel Elimination / Gastrointestinal Disorder 241 Chapter 43- Nursing Care of the Child With an Alteration in Urinary Elimination / Genitourinary Disorder 247 Chapter 44- Nursing Care of the Child With an Alteration in Mobility / Neuromuscular or Musculoskeletal Disorder 254 Extra Questions: Musculoskeletal Disorder 260 Chapter 45- Nursing Care of the Child With an Alteration in Tissue Integrity / Integumentary Disorder 266 Chapter 46- Nursing Care of the Child With an Alteration in Cellular Regulation / Hematologic or Neoplastic Disorder 272 Extra Questions: Neoplastic Disorder 278 Chapter 47- Nursing Care of the Child With an Alteration in Immunity or Immunologic Disorder 284 Chapter 48- Nursing Care of the Child With an Alteration in Metabolism / Endocrine Disorder 290 Chapter 49- Nursing Care of the Child With an Alteration in Genetics 296 Chapter 50- Nursing Care of the Child With an Alteration in Behavior, Cognition, or Development 303 Chapter 51- Nursing Care During a Pediatric Emergency 310 Chapter 1- Perspectives on Maternal, Newborn, and Women’s Health Care 1. The United States ranks 50th in the world for maternal mortality and 41st among industrialized nations for infant mortality rate. When developing programs to assist in decreasing these rates, which factor would most likely need to be addressed as having the greatest impact? A) Resolving all language and cultural differences B) Assuring early and adequate prenatal care C) Providing more extensive women’s shelters D) Encouraging all women to eat a balanced diet 2. When integrating the principles of family-centered care, the nurse would include which of the following? A) Childbirth is viewed as a procedural event B) Families are unable to make informed choices C) Childbirth results in changes in relationships D) Families require little information to make appropriate decisions 3. When preparing a teaching plan for a group of first-time pregnant women, the nurse expects to review how maternity care has changed over the years. Which of the following would the nurse include when discussing events of the 20th century? A) Epidemics of puerperal fever B) Performance of the first cesarean birth C) Development of the x-ray to assess pelvic size D) Creation of free-standing birth centers 4. After teaching a group of students about pregnancy-related mortality, the instructor determines that additional teaching is needed when the students identify which condition as a leading cause? A) Hemorrhage B) Embolism C) Obstructed labor D) Infection 5. The nurse is working with a group of community health members to develop a plan to address the special health needs of women. Which of the following conditions would the group address as the major problem? A) Smoking B) Heart disease C) Diabetes D) Cancer 6. When assessing a family for possible barriers to health care, the nurse would consider which factor to be most important? A) Language B) Health care workers’ attitudes C) Transportation D) Finances 7. After teaching a group of nursing students about the issue of informed consent. Which of the following, if identified by the student, would indicate an understanding of a violation of informed consent? A) Performing a procedure on a 15-year-old without consent B) Serving as a witness to the signature process C) Asking whether the client understands what she is signing D) Getting verbal consent over the phone for emergency procedures 8. The nurse is trying to get consent to care for an 11-year-old boy with diabetic ketoacidosis. His parents are out of town on vacation, and the child is staying with a neighbor. Which action would be the priority? A) Getting telephone consent with two people listening to the verbal consent B) Providing emergency care without parental consent C) Contacting the child’s aunt or uncle to obtain their consent D) Advocating for termination of parental rights for this situation 9. After teaching nursing students about the basic concepts of family-centered care, the instructor determines that the teaching was successful when the students state which of the following? A) “Childbirth affects the entire family, and relationships will change.” B) “Families are not capable of making health care decisions for themselves.” C) “Mothers are the family members affected by childbirth.” D) “Childbirth is a medical procedure.” 10. A nursing instructor is preparing a class discussion on the trends in health care and health care delivery over the past several centuries. When discussing the changes during the past century, which of the following would the instructor be least likely to include? A) Disease prevention B) Health promotion C) Wellness D) Analysis of morbidity and mortality 11. A nurse is assigned to care for an Asian American client. The nurse develops a plan of care with the understanding that based on this client’s cultural background, the client most likely views illness as which of the following? A) Caused by supernatural forces. B) A punishment for sins. C) Due to spirits or demons. D) From an imbalance of yin and yang 12. A nurse is developing a plan of care for a woman to ensure continuity of care during pregnancy, labor, and childbirth. Which of the following would be most important for the nurse to incorporate into that plan? A) Adhering to strict, specific routines B) Involving a pediatric physician C) Educating the client about the importance of a support person D) Assigning several nurses as a support team 13. A nursing instructor is preparing a class discussion on case management in maternal and newborn health care. Which of the following would the instructor include as a key component? Select all that apply. A) Advocacy B) Coordination C) Communication D) Resource management E) Event managed care 14. After teaching a group of students about the concept of maternal mortality, the instructor determines that additional teaching is needed when the students state which of the following? A) “The rate includes accidental causes for deaths.” B) “It addresses pregnancy-related causes.” C) “The duration of the pregnancy is not a concern.” D) “The time frame is typically for a specified year.” 15. A group of students are reviewing the historical aspects about childbirth. The students demonstrate understanding of the information when they identify the use of twilight sleep as a key event during which time frame? A) 1700s B) 1800s C) 1900s D) 2000s 16. A nurse is providing care to a woman who has just delivered a healthy newborn. Which action would least likely demonstrate application of the concept of family-centered care? A) Focusing on the birth as a normal healthy event for the family B) Creating opportunities for the family to make informed decisions C) Encouraging the woman to keep her other children at home D) Fostering a sense of respect for the mother and the family 17. When discussing fetal mortality with a group of students, a nurse addresses maternal factors. Which of the following would the nurse most likely include? Select all that apply. A) Chromosomal abnormalities B) Malnutrition C) Preterm cervical dilation D) Underlying disease condition E) Poor placental attachment 18. A nurse is preparing a presentation for a local community group about health status and children’s health. Which of the following would the nurse include as one of the most significant measures? A) Fetal mortality rate B) Neonatal mortality rate C) Infant mortality rate D) Maternal mortality rate 19. A group of students are reviewing an article describing information related to indicators for women’s health and the results of a national study. Which of the following would the students identify as being satisfactory for women? Select all that apply. A) Smoking cessation B) Colorectal cancer screening C) Violence against women D) Health insurance coverage E) Mammograms 20. A nurse is preparing a presentation for a local women’s group about heart disease and women. Which of the following would the nurse expect to address when discussing measures to promote health. A) Women have similar symptoms as men for a heart attack. B) Heart disease is no longer viewed as a “man’s disease.” C) Women experiencing a heart attack are at greater risk for dying. D) Heart attacks in women are more easily diagnosed. 21. A nurse is working to develop a health education program for a local community to address breast cancer awareness. Which of the following would the nurse expect to include when describing this problem to the group? Select all that apply. A) White women have higher rates of breast cancer than African American women. B) African American women are more likely to die from breast cancer at any age. C) Survival at any stage is worse among white women. D) Women living in South America have the highest rates of breast cancer. E) Breast cancer is the leading cause of cancer mortality in women. 22. A group of nursing students are reviewing information about factors affecting maternal, newborn, and women’s health. The students demonstrate understanding of the information when they identify which of the following deficiencies as being associated with poverty? Select all that apply. A) Literacy B) Employment opportunities C) Mobility D) Political representation E) Skills Answer Key 1. B 2. C 3. D 4. B 5. B 6. D 7. A 8. A 9. A 10. D 11. D 12. C 13. A, B, C, D 14. A 15. C 16. C 17. B, C, D 18. C 19. B, E 20. C 21. A, B 22. A, B, C, D, E Chapter 2- Family-Centered Community-Based Care 1. The nurse is caring for a 2-week-old newborn girl with a metabolic disorder. Which of the following activities would deviate from the characteristics of family-centered care? A) Softening unpleasant information or prognoses B) Evaluating and changing the nursing plan of care C) Collaborating with the child and family as equals D) Showing respect for the family’s beliefs and wishes 2. The nurse is providing home care for a 6-year-old girl with multiple medical challenges. Which of the following activities would be considered the tertiary level of prevention? A) Arranging for a physical therapy session B) Teaching parents to administer albuterol C) Reminding parent to give a full course of antibiotics D) Giving a DTaP vaccination at the proper interval 3. A nursing student is reviewing information about documenting client care and education in the medical record and the purposes that it serves. The student demonstrates a need for additional study when the nurse identifies which of the following as a reason? A) Serves as a communication tool for the interdisciplinary team. B) Demonstrates education the family has received if legal matters arise. C) Permits others access to allow refusal of medical insurance coverage. D) Verifies meeting client education standards set by the Joint Commission. 4. A pregnant client tells her nurse that she is interested in arranging a home birth. After educating the client on the advantages and disadvantages, which statement would indicate that the client understood the information? A) “I like having the privacy, but it might be too expensive for me to set up in my home.” B) “I want to have more control, but I am concerned if an emergency would arise.” C) “It is safer because I will have a midwife.” D) “The midwife is trained to resolve any emergency, and she can bring any pain meds.” 5. The nurse is making a home visit to a client who had a cesarean birth 3 days ago. Assessment reveals that the client is complaining of intermittent pain, rating it as 8 on a scale of 1 to 10. She states, “I’m pretty tired. And with this pain, I haven’t been drinking and eating like I should. The medication helps a bit but not much. My mom has been helping with the baby.” Her incision is clean, dry, and intact. Which nursing diagnosis would the nurse identify as the priority for this client? A) Impaired skin integrity related to cesarean birth incision B) Fatigue related to effects of surgery and caretaking activities C) Imbalanced nutrition, less than body requirements related to poor fluid and food intake D) Acute pain related to incision and cesarean birth 6. When caring for childbearing families from cultures different from one’s own, which of the following must be accomplished first? A) Adapt to the practices of the family’s culture B) Determine similarities between both cultures C) Assess personal feelings about that culture D) Learn as much as possible about that culture 7. After teaching a group of students about the changes in health care delivery and funding, which of the following, if identified by the group as a current trend seen in the maternal and child health care settings, would indicate that the teaching was successful? A) Increase in community settings for care B) Decrease in family poverty level C) Increase in hospitalization of children D) Decrease in managed care 8. The nurse would recommend the use of which supplement as a primary prevention strategy to prevent neural tube defects with pregnant women? A) Calcium B) Folic acid C) Vitamin C D) Iron 9. Which action would the nurse include in a primary prevention program in the community to help reduce the incidence of HIV infection? A) Provide treatment for clients who test positive for HIV B) Monitor viral load counts periodically C) Educate clients in how to practice safe sex D) Offer testing for clients who practice unsafe sex 10. When assuming the role of discharge planner for a woman requiring ventilator support at home, the nurse would do which of the following? A) Confer with the client’s mother B) Teach new self-care skills to the client C) Determine if there is a need for back-up power D) Discuss coverage with the insurance company 11. When comparing community-based nursing with nursing in the acute care setting to a group of nursing students, the nurse describes the challenges associated with community-based nursing. Which of the following would the nurse include? A) Increased time available for education B) Improved access to resources C) Decision making in isolation D) Greater environmental structure 12. After teaching a group of students about the different levels of prevention, the instructor determines a need for additional teaching when the students identify which of the following as a secondary prevention level activity in community-based health care? A) Teaching women to take folic acid supplements to prevent neural tube defects B) Working with women who are victims of domestic violence C) Working with clients at an HIV clinic to provide nutritional and CAM therapies D) Teaching hypertensive clients to monitor blood pressure 13. A nursing instructor is describing trends in maternal and newborn health care. The instructor addresses the length of stay for vaginal births during the past decade, citing that which of the following denotes the average stay? A) 24–48 hours or less B) 72–96 hours or less C) 48–72 hours or less D) 96–120 hours or less 14. Which of the following statements is accurate regarding women’s health care in today’s system? A) Women spend 95 cents of every dollar spent on health care. B) Women make almost 90% of all health care decisions. C) Women are still the minority in the United States. D) Men use more health services than women. 15. A nurse is educating a client about a care plan. Which of the following statements would be appropriate to assess the client’s learning ability? A) “Did you graduate from high school; how many years of schooling did you have?” B) “Do you have someone in your family who would understand this information?” C) “Many people have trouble remembering information; is this a problem for you?” D) “Would you prefer that the doctor give you more detailed medical information?” 16. A nurse is developing cultural competence. Which of the following indicates that the nurse is in the process of developing cultural knowledge? Select all that apply. A) Examining personal sociocultural heritage B) Reviewing personal biases and prejudices C) Seeking resources to further understanding of other cultures D) Becoming familiar with other culturally diverse lifestyles E) Performing a competent cultural assessment F) Advocating for social justice to eliminate disparities. 17. A nurse is engaged in providing family-centered care for a woman and her family. The nurse is providing instrumental support with which activity? A) Explaining to the woman and family what to expect during the birth process. B) Assisting the woman in breathing techniques to cope with labor contractions. C) Reinforcing the woman’s role as a mother after birth D) Helping the family obtain extra financial help for prescribed phototherapy 18. A nurse is considering a change in employment from the acute care setting to community- based nursing. The nurse is focusing her job search on ambulatory care settings. Which of the following would the nurse most likely find as a possible setting? Select all that apply. A) Urgent care center B) Hospice care C) Immunization clinic D) Physician’s office E) Day surgery center F) Nursing home 19. A nursing instructor is presenting a class for a group of students about community-based nursing interventions. The instructor determines that additional teaching is needed when the students identify which of the following? A) Conducting childbirth education classes B) Counseling a pregnant teen with anemia C) Consulting with a parent of a child who is vomiting D) Performing epidemiologic investigations 20. During class, a nursing student asks, “I read an article that was talking about integrative medicine. What is that?” Which response by the instructor would be most appropriate? A) “It refers to the use of complementary and alternative medicine in place of traditional therapies for a condition.” B) “It means that complementary and alternative medicine is used together with conventional therapies to reduce pain or discomfort.” C) “It means that mainstream medical therapies and complementary and alternative therapies are combined based on scientific evidence for being effective.” D) “It refers to situations when a client and his or her family prefer to use an unproven method of treatment over a proven one.” 21. While a nurse is obtaining a health history, the client tells the nurse that she practices aromatherapy. The nurse interprets this as which of the following? A) Use of essential oils to stimulate the sense of smell to balance the mind and body B) Application of pressure to specific points to allow self-healing C) Use of deep massage of areas on the foot or hand to rebalance body parts D) Participation in chanting and praying to promote healing. 22. A pregnant woman asks the nurse about giving birth in a birthing center. She says, “I’m thinking about using one but I’m not sure.” Which of the following would the nurse need to integrate into the explanation about this birth setting? (Select all that apply.) A) An alternative for women who are uncomfortable with a home birth. B) The longer length of stay needed when compared to hospital births C) Focus on supporting women through labor instead of managing labor D) View of labor and birth as a normal process requiring no intervention E) Care provided primarily by obstetricians with midwives as backup care 23. A nurse practicing in the community is preparing a presentation for a group of nursing students about this practice setting. Which of the following would the nurse include as characteristic of this role? A) Greater emphasis on direct physical care B) Broader assessment to include the environment C) Increased dependency on physician D) Limited decision making and support 24. A nurse is preparing a teaching plan for a woman who is pregnant for the first time. Which of the following would the nurse incorporate into the teaching plan to foster the client’s learning? (Select all that apply.) A) Teach “survival skills” first B) Use simple, nonmedical language C) Refrain from using a hands-on approach D) Avoid repeating information E) Use visual materials such as photos and videos 25. A group of nurses are reviewing the steps for developing cultural competence. The students demonstrate understanding when they identify which of the following as the final step? A) Cultural knowledge B) Cultural skills C) Cultural encounter D) Cultural awareness Answer Key 1. A 2. A 3. C 4. B 5. D 6. C 7. A 8. B 9. C 10. C 11. C 12. A 13. A 14. B 15. C 16. C, D 17. D 18. A, D, E 19. D 20. C 21. A 22. A, C, D 23. B 24. A, B, E 25. C Chapter 3- Anatomy and Physiology of the Reproductive System 1. When describing the menstrual cycle to a group of young women, the nurse explains that estrogen levels are highest during which phase of the endometrial cycle? A) Menstrual B) Proliferative C) Secretory D) Ischemic 2. After teaching a group of adolescent girls about female reproductive development, the nurse determines that teaching was successful when the girls state that menarche is defined as a woman’s first: A) Sexual experience B) Full hormonal cycle C) Menstrual period D) Sign of breast development 3. A client with a 28-day cycle reports that she ovulated on May 10. The nurse would expect the client’s next menses to begin on: A) May 24 B) May 26 C) May 30 D) June 1 4. Which female reproductive tract structure would the nurse describe to a group of young women as containing rugae that enable it to dilate during labor and birth? A) Cervix B) Fallopian tube C) Vagina D) Vulva 5. After teaching a group of pregnant women about breast-feeding, the nurse determines that the teaching was successful when the group identifies which hormone as important for the production of breast milk after childbirth? A) Placental estrogen B) Progesterone C) Gonadotropin-releasing hormone D) Prolactin 6. The nurse is assessing a 13-year-old girl who has had her first menses. Which of the following events would the nurse expect to have occurred first? A) Evidence of pubic hair B) Development of breast buds C) Onset of menses D) Growth spurt 7. When describing the ovarian cycle to a group of students, which phase would the instructor include? A) Luteal phase B) Proliferative phase C) Menstrual phase D) Secretory phase 8. The nurse is explaining the events that lead up to ovulation. Which hormone would the nurse identify as being primarily responsible for ovulation? A) Estrogen B) Progesterone C) Follicle-stimulating hormone D) Luteinizing hormone 9. The nurse is teaching a health education class on male reproductive anatomy and asks the students to identify the site of sperm production. Which structure, if identified by the group, would indicate to the nurse that the teaching was successful? A) Testes B) Seminal vesicles C) Scrotum D) Prostate gland 10. The nurse is creating a diagram that illustrates the components of the male reproductive system. Which structure would be inappropriate for the nurse to include as an accessory gland? A) Seminal vesicles B) Prostate gland C) Cowper’s glands D) Vas deferens 11. The nurse is preparing an outline for a class on the physiology of the male sexual response. Which event would the nurse identify as occurring first? A) Sperm emission B) Penile vasodilation C) Psychological release D) Ejaculation 12. A woman comes to the clinic complaining that she has little sexual desire. As part of the client’s evaluation, the nurse would anticipate the need to evaluate which hormone level? A) Progesterone B) Estrogen C) Gonadotropin-releasing hormone D) Testosterone 13. A nurse is conducting a class for a group of teenage girls about female reproductive anatomy and physiology. Which of the following would the nurse include as an external female reproductive organ? Select all that apply. A) Mons pubis B) Labia C) Vagina D) Clitoris E) Uterus 14. When describing the hormones involved in the menstrual cycle, a nurse identifies which hormone as responsible for initiating the cycle? A) Estrogen B) Luteinizing hormone C) Progesterone D) Prolactin 15. A nursing instructor is describing the hormones involved in the menstrual cycle to a group of nursing students. The instructor determines the teaching was successful when the students identify follicle-stimulating hormone as being secreted by which of the following? A) Hypothalamus B) Anterior pituitary gland C) Ovaries D) Corpus luteum 16. A woman comes to the clinic for an evaluation. During the visit, the woman tells the nurse that her menstrual cycles have become irregular. “I’ve also been waking up at night feeling really hot and sweating. The nurse interprets these findings as which of the following? A) Menopause B) Perimenopause C) Climacteric D) Menarche 17. After teaching a group of students about female reproductive anatomy, the instructor determines that the teaching was successful when the students identify which of the following as the site of fertilization? A) Vagina B) Uterus C) Fallopian tubes D) Vestibule 18. A woman comes to the clinic complaining of a vaginal discharge. The nurse suspects that the client has an infection. When gathering additional information, which of the following would the nurse be least likely to identify as placing the client at risk for an infection? A) Recent antibiotic therapy for an upper respiratory infection B) Last menstrual period about 5 days ago. C) Weekly douching D) Frequent use of feminine hygiene sprays. 19. A group of nursing students are reviewing information about the male reproductive structures. The students demonstrate understanding of the information when they identify which of the following as accessory organs? (Select all that apply.) A) Testes B) Vas deferens C) Bulbourethral glands D) Prostate gland E) Penis 20. A nurse is examining a female client and tests the client’s vaginal pH. Which finding would the nurse interpret as normal? A) 4.5 B) 7 C) 8.5 D) 10 21. When describing the male sexual response to a group of students, the instructor determines that the teaching was successful when they identify emission as which of the following? A) Semen forced through the urethra to the outside B) Movement of sperm from the testes and fluid into the urethras C) Dilation of the penile arteries with increased blood flow to the tissues. D) Body’s return to the physiologic nonstimulated state 22. A nurse is describing the structure and function of the reproductive system to an adolescent health class. The nurse describes the secretion of the seminal vesicles as which of the following? A) Mucus-like B) Alkaline C) Acidic D) Semen Answer Key 1. B 2. C 3. A 4. C 5. D 6. B 7. A 8. D 9. A 10. D 11. B 12. D 13. A, B, D 14. B 15. B 16. B 17. C 18. B 19. B, C, D 20. A 21. B 22. B   Chapter 4- Common Reproductive Issues 1. After discussing various methods of contraception with a client and her partner, the nurse determines that the teaching was successful when they identify which contraceptive method as providing protection against sexually transmitted infections (STIs)? A) Oral contraceptives B) Tubal ligation C) Condoms D) Intrauterine system 2. When discussing contraceptive options, which method would the nurse recommend as being the most reliable? A) Coitus interruptus B) Lactational amenorrheal method (LAM) C) Natural family planning D) Intrauterine system 3. A client comes to the clinic with abdominal pain. Based on her history the nurse suspects endometriosis. The nurse expects to prepare the client for which of the following to confirm this suspicion? A) Pelvic examination B) Transvaginal ultrasound C) Laparoscopy D) Hysterosalpingogram 4. A client is to receive an implantable contraceptive. The nurse describes this contraceptive as containing: A) Synthetic progestin B) Combined estrogen and progestin C) Concentrated spermicide D) Concentrated estrogen 5. The nurse discusses various contraceptive methods with a client and her partner. Which method would the nurse explain as being available only with a prescription? A) Condom B) Spermicide C) Diaphragm D) Basal body temperature 6. When developing a teaching plan for a couple considering contraception options, which of the following statements would the nurse include? A) “You should select one that is considered to be 100% effective.” B) “The best one is the one that is the least expensive and most convenient.” C) “A good contraceptive doesn’t require a physician’s prescription.” D) “The best contraceptive is one that you will use correctly and consistently.” 7. Which of the following measures would the nurse include in the teaching plan for a woman to reduce the risk of osteoporosis after menopause? A) Taking vitamin supplements B) Eating high-fiber, high-calorie foods C) Restricting fluid to 1,000 mL daily D) Participating in regular daily exercise 8. When teaching a group of postmenopausal women about hot flashes and night sweats, the nurse would address which of the following as the primary cause? A) Poor dietary intake B) Estrogen deficiency C) Active lifestyle D) Changes in vaginal pH 9. A client states that she is to have a test to measure bone mass to help diagnose osteoporosis. The nurse would most likely plan to prepare the client for: A) DEXA scan B) Ultrasound C) MRI D) Pelvic x-ray 10. The nurse is reviewing the medical records of several clients. Which client would the nurse expect to have an increased risk for developing osteoporosis? A) A woman of African American descent B) A woman who plays tennis twice a week C) A thin woman with small bones D) A woman who drinks one cup of coffee a day 11. Which of the following would the nurse emphasize when teaching postmenopausal women about ways to reduce the risk of osteoporosis? A) Swimming daily B) Taking vitamin A C) Following a low-fat diet D) Taking calcium supplements 12. Which finding would the nurse expect to find in a client with endometriosis? A) Hot flashes B) Dysuria C) Fluid retention D) Fever 13. After the nurse teaches a client about ways to reduce the symptoms of premenstrual syndrome, which client statement indicates a need for additional teaching? A) “I will make sure to take my estrogen supplements a week before my period.” B) “I’ve signed up for an aerobic exercise class three times a week.” C) “I’ll cut down on the amount of coffee and colas I drink.” D) “I quit smoking about a month ago, so that should help.” 14. A woman has opted to use the basal body temperature method for contraception. The nurse instructs the client that a rise in basal body temperature indicates which of the following? A) Onset of menses B) Ovulation C) Pregnancy D) Safe period for intercourse 15. A woman using the cervical mucus ovulation method of fertility awareness reports that her cervical mucus looks like egg whites. The nurse interprets this as which of the following? A) Spinnbarkeit mucus B) Purulent mucus C) Postovulatory mucus D) Normal preovulation mucus 16. The nurse is reviewing the laboratory test results of a client with dysfunctional uterine bleeding (DUB). Which finding would be of concern? A) Negative pregnancy test B) Hemoglobin level of 10.1 g/dL C) Prothrombin time of 60 seconds D) Serum cholesterol of 140 mg/dL 17. A nurse is preparing a class for a group of women at a family planning clinic about contraceptives. When describing the health benefits of oral contraceptives, which of the following would the nurse most likely include? (Select all that apply.) A) Protection against pelvic inflammatory disease B) Reduced risk for endometrial cancer C) Decreased risk for depression D) Reduced risk for migraine headaches E) Improvement in acne 18. After teaching a group of students about the different methods for contraception, the instructor determines that the teaching was successful when the students identify which of the following as a mechanical barrier method? (Select all that apply.) A) Condom B) Cervical cap C) Cervical sponge D) Diaphragm E) Vaginal ring 19. After assessing a woman who has come to the clinic, the nurse suspects that the woman is experiencing dysfunctional uterine bleeding. Which statement by the client would support the nurse’s suspicions? A) “I’ve been having bleeding off and on that’s irregular and sometimes heavy.” B) “I get sharp pain in my lower abdomen usually starting soon after my period comes.” C) “I get really irritable and moody about a week before my period.” D) “My periods have been unusually long and heavy lately.” 20. After teaching a group of students about premenstrual syndrome, the instructor determines that additional teaching is needed when the students identify which of the following as a prominent assessment finding? A) Bloating B) Tension C) Dysphoria D) Weight loss 21. A nurse is describing the criteria needed for the diagnosis of premenstrual dysphoric disorder (PMDD). Which of the following would the nurse include as a mandatory requirement for the diagnosis? A) Appetite changes B) Sleep difficulties C) Persistent anger D) Chronic fatigue 22. When reviewing the medical record of a client diagnosed with endometriosis, which of the following would the nurse identify as a risk factor for this woman? A) Low fat in the diet B) Age of 14 years for menarche C) Menstrual cycles of 24 days D) Short menstrual flow 23. A client who has come to the clinic is diagnosed with endometriosis. Which of the following would the nurse expect the physician to prescribe as a first-line treatment? A) Progestins B) Antiestrogens C) Gonadotropin-releasing hormone analogues D) NSAIDs 24. A woman comes to the clinic because she has been unable to conceive. When reviewing the woman’s history, which of the following would the nurse least likely identify as a possible risk factor? A) Age of 25 years B) History of smoking C) Diabetes since age 15 years D) Weight below standard for height and age 25. A couple comes to the clinic for a fertility evaluation. The male partner is to undergo a semen analysis. After teaching the partner about this test, which client statement indicates that the client has understood the instructions? A) “I need to bring the specimen to the lab the day after collecting it.” B) “I will place the specimen in a special plastic bag to transport it.” C) “I have to abstain from sexual activity for about 1–2 days before the sample.” D) “I will withdraw before I ejaculate during sex to collect the specimen.” 26. A nurse is preparing a class for a group of young adult women about emergency contraceptives (ECs). Which of the following would the nurse need to stress to the group. Select all that apply. A) ECs induce an abortion like reaction. B) ECs provide some protection against STIs C) ECs are birth control pills in higher, more frequent doses D) ECs are not to be used in place of regular birth control E) ECs provide little protection for future pregnancies. Answer Key 1. C 2. D 3. C 4. A 5. C 6. D 7. D 8. B 9. A 10. C 11. D 12. B 13. A 14. B 15. A 16. B 17. A, B, E 18. A, B, C, D 19. A 20. D 21. C 22. C 23. D 24. A 25. C 26. C, D, E Chapter 5- Sexually Transmitted Infections 1. The nurse is developing a plan of care for a client who is receiving highly active antiretroviral therapy (HAART) for treatment of HIV. The goal of this therapy is to: A) Promote the progression of disease B) Intervene in late-stage AIDS C) Improve survival rates D) Conduct additional drug research 2. A woman who is HIV-positive is receiving HAART and is having difficulty with compliance. To promote adherence, which of the following areas would be most important to assess initially? A) The woman’s beliefs and education B) The woman’s financial situation and insurance C) The woman’s activity level and nutrition D) The woman’s family and living arrangements 3. When developing a teaching plan for a community group about HIV infection, which group would the nurse identify as an emerging risk group for HIV infection? A) Native Americans B) Heterosexual women C) New health care workers D) Asian immigrants 4. After teaching a group of adolescents about HIV, the nurse asks them to identify the major means by which adolescents are exposed to the virus. The nurse determines that the teaching was successful when the group identifies which of the following? A) Sexual intercourse B) Sharing needles for IV drug use C) Perinatal transmission D) Blood transfusion 5. The nurse reviews the CD4 cell count of a client who is HIV-positive. A result less than which of the following would indicate to the nurse that the client has AIDS? A) 1,000 cells/mm3 B) 700 cells/mm3 C) 450 cells/mm3 D) 200 cells/mm3 6. When obtaining the health history from a client, which factor would lead the nurse to suspect that the client has an increased risk for sexually transmitted infections (STIs)? A) Hive-like rash for the past 2 days B) Five different sexual partners C) Weight gain of 5 lbs in 1 year D) Clear vaginal discharge 7. Assessment of a female client reveals a thick, white vaginal discharge. She also reports intense itching and dyspareunia. Based on these findings, the nurse would suspect that the client has: A) Trichomoniasis B) Bacterial vaginosis C) Candidiasis D) Genital herpes simplex 8. A client with trichomoniasis is to receive metronidazole (Flagyl). The nurse instructs the client to avoid which of the following while taking this drug? A) Alcohol B) Nicotine C) Chocolate D) Caffeine 9. A woman gives birth to a healthy newborn. As part of the newborn’s care, the nurse instills erythromycin ophthalmic ointment as a preventive measure related to which STI? A) Genital herpes B) Hepatitis B C) Syphilis D) Gonorrhea 10. Which findings would the nurse expect to find in a client with bacterial vaginosis? A) Vaginal pH of 3 B) Fish-like odor of discharge C) Yellowish-green discharge D) Cervical bleeding on contact 11. A pregnant woman diagnosed with syphilis comes to the clinic for a visit. The nurse discusses the risk of transmitting the infection to her newborn, explaining that this infection is transmitted to the newborn through the: A) Amniotic fluid B) Placenta C) Birth canal D) Breast milk 12. The nurse encourages a female client with human papillomavirus (HPV) to receive continued follow-up care because she is at risk for: A) Infertility B) Dyspareunia C) Cervical cancer D) Dysmenorrhea 13. A client is diagnosed with pelvic inflammatory disease (PID). When reviewing the client’s medical record, which of the following would the nurse expect to find? (Select all that apply.) A) Oral temperature of 100.4 degrees F B) Dysmenorrhea C) Dysuria D) Lower abdominal tenderness E) Discomfort with cervical motion F) Multiparity 14. Which instructions would the nurse include when teaching a woman with pediculosis pubis? A) “Take the antibiotic until you feel better.” B) “Wash your bed linens in bleach and cold water.” C) “Your partner doesn’t need treatment at this time.” D) “Remove the nits with a fine-toothed comb.” 15. A client with genital herpes simplex infection asks the nurse, “Will I ever be cured of this infection?” Which response by the nurse would be most appropriate? A) “There is a new vaccine available that prevents the infection from returning.” B) “All you need is a dose of penicillin and the infection will be gone.” C) “There is no cure, but drug therapy helps to reduce symptoms and recurrences.” D) “Once you have the infection, you develop an immunity to it.” 16. A nurse is preparing a presentation for a group of women at the clinic who have been diagnosed with genital herpes. Which of the following would the nurse expect to include as a possible precipitating factor for a recurrent outbreak? (Select all that apply.) A) Exposure to ultraviolet light B) Exercise C) Use of corticosteroids D) Emotional stress E) Sexual intercourse. 17. After teaching a class on sexually transmitted infections, the instructor determines that the teaching was successful when the class identifies which statement as true? A) STIs can affect anyone if exposed to the infectious organism. B) STIs have been addressed more on a global scale. C) Clients readily view the diagnosis of STI openly. D) Most individuals with STIs are over the age of 30. 18. A group of students are reviewing information about STIs. The students demonstrate understanding of the information when they identify which of the following as the most common bacterial STI in the United States? A) Gonorrhea B) Chlamydia C) Syphilis D) Candidiasis 19. A nurse is assessing a client for possible risk factors for chlamydia and gonorrhea. Which of the following would the nurse identify? A) Asian American ethnicity B) Age under 25 years C) Married D) Consistent use of barrier contraception 20. A nurse at a local community clinic is developing a program to address STI prevention. Which of the following would the nurse least likely include in the program? A) Outlining safer sexual behavior B) Recommending screening for symptomatic individuals C) Promoting the use of barrier contraceptives D) Offering education about STI transmission 21. After teaching a class on preventing pelvic inflammatory disease, the instructor determines that the teaching was successful when the class identifies which of the following as an effective method? A) Advising sexually active females to use hormonal contraception B) Encouraging vaginal douching on a weekly basis. C) Emphasizing the need for infected sexual partners to receive treatment D) Promoting routine treatment for asymptomatic females as risk 22. A group of nursing students are reviewing information about vaccines used to prevent STIs. The students would expect to find information about which of the following? A) HIV B) HSV C) HPV D) HAV E) HBV 23. A mother brings her 12-year-old daughter in for well-visit checkup. During the visit, the nurse is discussing the use of prophylactic HPV vaccine for the daughter. The mother agrees and the daughter receives her first dose. The nurse schedules the daughter for the next dose, which would be given at which time? A) In 2 month B) In 2 months C) In 3 months D) In 4 months 24. A woman comes to the clinic complaining of a vaginal discharge. The nurse suspects trichomoniasis based on which of the following? (Select all that apply.) A) Urinary frequency B) Yellow/green discharge C) Joint pain D) Blister-like lesions E) Muscle aches 25. A nurse is teaching a women with genital ulcers how to care for them. Which statement by the client indicates a need for additional teaching? A) “I need to wash my hands after touching any of the ulcers.” B) “I need to abstain from intercourse primarily when the lesions are present.” C) “I should avoid applying ice or heat to my genital area.” D) “I can try lukewarm sitz baths to help ease the discomfort.” Answer Key 1. C 2. A 3. B 4. A 5. D 6. B 7. C 8. A 9. D 10. B 11. B 12. C 13. B, C, D, E 14. D 15. C 16. A, C, D, E 17. A 18. B 19. B 20. B 21. C 22. C, D, E 23. B 24. A, B 25. B Chapter 6- Disorders of the Breasts 1. The nurse is developing the discharge plan for a woman who has had a left-sided modified radical mastectomy. The nurse is including instructions for ways to minimize lymphedema. Which suggestion would most likely increase the woman’s symptoms? A) “Wear gloves when you are doing any gardening.” B) “Have your blood pressure taken in your right arm.” C) “Wear clothing with elasticized sleeves.” D) “Avoid driving to and from work every day.” 2. A laboratory technician arrives to draw blood for a complete blood count (CBC. for a client who had a right-sided mastectomy 8 hours ago. The client has an intravenous line with fluid infusing in her left antecubital space. To obtain the blood specimen, the technician places a tourniquet on the client’s right arm. Which action by the nurse would be most appropriate? A) Assist in holding the client’s arm still. B) Suggest a finger stick be done on one of the client’s left fingers. C) Tell the technician to obtain the blood sample from the client’s left arm. D) Call the surgeon to perform a femoral puncture. 3. The nurse determines that a woman has implemented prescribed therapy for her fibrocystic breast disease when the client reports that she has eliminated what from her diet? A) Caffeine B) Cigarettes C) Dairy products D) Sweets 4. When assessing a client with suspected breast cancer, which of the following would the nurse expect to find? A) Painful lump B) Absence of dimpling C) Regularly shaped mass D) Nipple retraction 5. A woman who has undergone a right modified-radical mastectomy returns from surgery. Which nursing intervention would be most appropriate at this time? A) Ask the client how she feels about having her breast removed. B) Attach a sign above her bed to have BP, IV lines, and lab work in her right arm. C) Encourage her to turn, cough, and deep breathe at frequent intervals. D) Position her right arm below heart level. 6. A breast biopsy indicates the presence of malignant cells, and the client is scheduled for a mastectomy. Which nursing diagnosis would the nurse most likely include in the client’s preoperative plan of care as the priority? A) Risk for deficient fluid volume B) Activity intolerance C) Disturbed body image D) Impaired urinary elimination 7. A 42-year-old woman is scheduled for a mammogram. Which of the following would the nurse include when teaching the woman about the procedure? A) “The room will be darkened throughout the procedure.” B) “Each breast will be firmly compressed between two plates.” C) “Make sure to refrain from eating or drinking after midnight.” D) “A small needle will be inserted to get a sample for evaluation.” 8. During a clinical breast examination, the nurse palpates a well-defined, firm, mobile lump in a 60-year-old woman’s left breast. The nurse notifies the physician. Which of the following would the nurse anticipate the physician to order next? A) Mammogram B) Hormone receptor status C) Fine-needle aspiration D) Genetic testing for BRCA 9. A client with advanced breast cancer, who has had both chemotherapy and radiation therapy, is to start hormonal therapy. Which agent would the nurse expect the client to receive? A) Progestins B) Tamoxifen C) Cortisone D) Estrogen 10. As part of discharge planning, the nurse refers a woman to Reach to Recovery. This group’s primary purpose is to: A) Help support women who have undergone mastectomies B) Raise funds to support early breast cancer detection programs C) Provide all supplies needed after breast surgery for no cost D) Collect statistics for research for the American Cancer Society 11. A woman with breast cancer is undergoing chemotherapy. Which of the following side effects would the nurse interpret as being most serious? A) Vomiting B) Hair loss C) Fatigue D) Myelosuppression 12. A woman comes to the clinic reporting a nipple discharge. On examination, the area below the areola is red and slightly swollen, with tortuous tubular swelling. The nurse interprets these findings as suggestive of which of the following? A) Fibrocystic breast disorder B) Intraductal papilloma C) Duct ectasia D) Fibroadenoma 13. When performing a clinical breast examination, which would the nurse do first? A) Palpate the axillary area. B) Compress the nipple for a discharge. C) Palpate the breasts. D) Inspect the breasts. 14. Evaluation of a woman with breast cancer reveals that her mass is approximately 1.25 inches in diameter. Three adjacent lymph nodes are positive. The nurse interprets this as indicating that the woman has which stage of breast cancer? A) 0 B) I C) II D) III 15. After teaching a woman how to perform breast self-examination, which statement would indicate that the nurse’s instructions were successful? A) “I should lie down with my arms at my side when looking at my breasts.” B) “I should use the fingerpads of my three middle fingers to apply pressure to my breast.” C) “I don’t need to check under my arm on that side if my breast feels fine.” D) “I need to work from the center of my breast outward toward my shoulder.” 16. A nurse is working with a woman who has been diagnosed with severe fibrocystic breast disease. When describing the medications that can be used as treatment, which of the following would the nurse be least likely to include? A) Tamoxifen B) Bromocriptine C) Danazol D) Penicillin 17. A group of students are reviewing information about benign and malignant breast masses. The students demonstrate understanding when they identify which of the following as indicating a benign breast mass. (Select all the apply.) A) Painless B) Unilateral location C) Firm consistency D) Absence of dimpling E) Fixed to chest wall 18. The nurse is developing a plan of care for a woman with breast cancer who is scheduled to undergo breast-conserving surgery. The nurse interprets this as which of the following? A) Removal of nipple and areolar area B) Lump removal followed by radiation C) Entire breast removal without lymph nodes D) Axillary lymph node removal 19. A woman comes to the clinic and asks the nurse about when she should have her first mammogram. Using the recommendations of the American Cancer Society, which would the nurse suggest? A) 30 years B) 35 years C) 40 years D) 45 years 20. After teaching a group of nursing students about the different types of chemotherapeutic agents used to treat breast cancer, the instructor determines that the teaching was successful when the students identify which of the following as an example of a selective estrogen receptor modulator (SERM)? (Select all that apply.) A) Tamoxifen B) Letozole C) Raloxifene D) Exemestane E) Anastrozole 21. A woman diagnosed with breast cancer is to receive trastuzumab. Which of the following would the nurse incorporate into the explanation about how this drug works? A) It blocks the effect of the HER-2/neu protein inhibiting the growth of cancer cells. B) The drug blocks the conversion of androgens to estrogens C) It interferes with hormone receptors that allow estrogen to enter a cell D) The drug ultimately attacks areas where micrometastasis has occurred. 22. A nurse is conducting a class on breast cancer prevention. Which statement would the nurse most likely include in the discussion? A) “Most often a lump is felt before it is seen.” B) “Early breast cancer usually has some symptoms.” C) “If the mass is not painful, it is usually benign.” D) “If lump is palpable, it has been there for some time.” 23. When describing programs for breast cancer screening, the nurse include breast self- examination (BSE). Which of the following most accurately reflects the current thinking about breast self-examination? A) BSE is essential for early breast cancer detection. B) A woman performing BSE has breast awareness. C) BSE plays a minimal role in detecting breast cancer D) A clinical breast exam has replaced BSE. 24. During a wellness visit to the clinic, a woman asks the nurse if there is anything she can do to reduce her risk for developing breast cancer. Which of the following would the nurse most likely include? (Select all that apply.) A) Eating three servings of fruit daily B) Keeping weight gain under 11 pounds after age 18 C) Eating at least seven portions of complex carbohydrates daily D) Limiting the intake of refined sugar products E) Using salt liberally when cooking 25. A woman comes to the clinic and tells the nurse that she has read an article about certain foods that have anticancer properties and help boost the immune system. The nurse identifies A) Garlic B) Soybeans C) Milk D) Leeks E) Flax seeds Answer Key 1. C 2. B 3. A 4. D 5. C 6. C 7. B 8. A 9. B 10. A 11. D 12. C 13. D 14. C 15. B 16. D 17. C, D 18. B 19. C 20. A, C 21. A 22. D 23. B 24. B, C, D 25. A, B, D, E Chapter 7- Benign Disorders of the Female Reproductive Tract 1. A woman is admitted for repair of cystocele and rectocele. She has nine living children. In taking her health history, which of the following would the nurse expect to find? A) Sporadic vaginal bleeding accompanied by chronic pelvic pain B) Heavy leukorrhea with vulvar pruritus C) Menstrual irregularities and hirsutism on the chin D) Stress incontinence with feeling of low abdominal pressure 2. To assist the woman in regaining control of the urinary sphincter for urinary incontinence, the nurse should teach the client to do which of the following? A) Perform Kegel exercises daily. B) Void every hour while awake. C) Limit her intake of fluid. D) Take a laxative every night. 3. When developing the plan of care for a woman who has had an abdominal hysterectomy, which of the following would be contraindicated? A) Ambulating the client B) Massaging the client’s legs C) Applying elasticized stockings D) Encouraging range-of-motion exercises 4. Which of the following would the nurse include when teaching women about preventing pelvic support disorders? A) Performing Kegel isometric exercises B) Consuming low-fiber diets C) Using hormone replacement D) Voiding every 2 hours 5. A client is diagnosed with an enterocele. The nurse interprets this condition as: A) Protrusion of the posterior bladder wall downward through the anterior vaginal wall B) Sagging of the rectum with pressure exerted against the posterior vaginal wall C) Bulging of the small intestine through the posterior vaginal wall D) Descent of the uterus through the pelvic floor into the vagina 6. A woman is scheduled for an anterior and posterior colporrhaphy as treatment for a cystocele. When the nurse is explaining this treatment to the client, which of the following descriptions would be most appropriate to include? A) “This procedure helps to tighten the vaginal wall in the front and back so that your bladder and urethra are in the proper position.” B) “Your uterus will be removed through your vagina, helping to relieve the organ that is putting the pressure on your bladder.” C) “This is a series of exercises that you will learn to do so that you can strengthen your bladder muscles.” D) “These are plastic devices that your physician will insert into your vagina to provide support to the uterus and keep it in the proper position.” 7. The nurse would be least likely to find which of the following in a client with uterine fibroids? A) Regularly shaped, shrunken uterus B) Acute pelvic pain C) Menorrhagia D) Complaints of bloating 8. A client with polycystic ovarian syndrome (PCOS. is receiving oral contraceptives as part of her treatment plan. The nurse understands that the rationale for this therapy is to: A) Restore menstrual regularity B) Induce ovulation C) Improve insulin uptake D) Alleviate hirsutism 9. When teaching a woman how to perform Kegel exercises, the nurse explains that these exercises are designed to strengthen which muscles? A) Gluteus B) Lower abdominal C) Pelvic floor D) Diaphragmatic 10. A postmenopausal woman with uterine prolapse is being fitted with a pessary. The nurse would be most alert for which side effect? A) Increased vaginal discharge B) Urinary tract infection C) Vaginitis D) Vaginal ulceration 11. When preparing the discharge teaching plan for the woman who had surgery to correct pelvic organ prolapse, which of the following would the nurse include? A) Care of the indwelling catheter at home B) Emphasis on coughing to prevent complications C) Return to usual activity level in a few days D) Daily douching with dilute vinegar solution 12. A woman with polycystic ovary syndrome tells the nurse, “I hate this disease. Just look at me! I have no hair on the front of my head but I’ve got hair on my chin and upper lip. I don’t feel like a woman anymore.” Further assessment reveals breast atrophy and increased muscle mass. Which nursing diagnosis would most likely be a priority? A) Situational low self-esteem related to masculinization effects of the disease B) Social isolation related to feelings about appearance C) Risk for suicide related to effects of condition and fluctuating hormone levels D) Ineffective peripheral tissue perfusion related to effects of disease on vasculature 13. After teaching a local woman’s group about incontinence, the nurse determines that the teaching was successful when the group identifies which of the following as characteristic of stress incontinence? A) Feeling a strong need to void B) Passing a large amount of urine C) Most common in women after childbirth D) Sneezing may be an initiating stimulus 14. A woman is being evaluated for pelvic organ prolapse. A postvoid residual urine specimen is obtained via a catheter. Which residual volume finding would lead the nurse to suspect the need for further testing? A) 50 mL B) 75 mL C) 100 mL D) 120 mL 15. After teaching a woman with pelvic organ prolapse about dietary and lifestyle measures, which of the following statements would indicate the need for additional teaching? A) “If I wear a girdle, it will help support the muscles in the area.” B) “I should take up jogging to make sure I exercise enough.” C) “I will try to drink at least 64 oz of fluid each day.” D) “I need to increase the amount of fiber I eat every day.” 16. After teaching a group of students about genital fistulas, the instructor determines that the teaching was successful when the students identify which of the following as a major cause? A) Radiation therapy B) Congenital anomaly C) Female genital cutting D) Bartholin’s gland abscess 17. A nurse is providing care to a female client receiving treatment for a Bartholin’s cyst. The client has had a small loop of plastic tubing secured in place to allow for drainage. The nurse instructs the client that she will have a follow-up appointment for removal of the plastic tubing at which time? A) 1 week B) 2 weeks C) 3 weeks D) 4 weeks 18. After undergoing diagnostic testing, a woman is diagnosed with a corpus luteum cyst. The nurse anticipates that the woman will require: A) Biopsy B) No treatment C) Oral contraceptives D) Glucophage 19. A nurse is teaching a client how to perform Kegel exercises. Which of the following would the nurse include? (Select all that apply.) A) “Squeeze your rectal muscles as if you are trying to avoid passing flatus.” B) “Tighten your pubococcygeal muscles for a count of 10.” C) “Contract and relax your pubococcygeal muscles rapidly 10 times.” D) “Try bearing down for about 10 seconds for no more than 5 times.” E) “Do these exercises at least 5 times every hour.” 20. After teaching a group of students about pelvic organ prolapse, the instructor determines that the teaching was successful when the group identifies leiomyomas as which of the following? A) Cysts B) Pelvic organ prolapse C) Fistula D) Fibroid 21. A nurse is assessing a female client and suspects that the client may have endometrial polyps based on which of the following? A) Bleeding after intercourse B) Vaginal discharge C) Bleeding between menses D) Metrorrhagia 22. After teaching a group of students about ovarian cysts, the instructor determines that the teaching was successful when the students identify which type of cyst as being associated with hydatiform mole? A) Theca-lutein cyst B) Corpus luteum cyst C) Follicular cyst D) Polycystic ovarian syndrome 23. A nurse is reading a journal article about care of the woman with pelvic organ prolapse. The nurse would expect to find information related to which of the following? (Select all that apply.) A) Rectocele B) Fecal incontinence C) Cystocele D) Urinary incontinence E) Enterocele 24. A nurse is reviewing the medical record of a client. Which of the following would lead the nurse to suspect that the client is experiencing polycystic ovarian syndrome? (Select all that apply) A) Decreased androgen levels B) Elevated blood insulin levels C) Anovulation D) Waist circumference of 32 inches E) Triglyceride level of 175 mg/dL F) High-density lipoprotein level of 40 mg/dL 25. A group of students are preparing a class presentation about polyps. Which of the following would the students most likely include in the presentation? A) Polyps are rarely the result of an infection. B) Endocervical polyps commonly appear after menarche. C) Cervical polyps are more common than endocervical polyps. D) Endocervical polyps are most common in women in their 50s. Answer Key 1. D 2. A 3. B 4. A 5. C 6. A 7. B 8. A 9. C 10. D 11. A 12. A 13. D 14. D 15. B 16. C 17. C 18. B 19. A, C 20. D 21. D 22. A 23. A, C, E 24. B, C, E 25. D Chapter 8- Cancers of the Female Reproductive Tract 1. The nurse would refer a client, age 54, for follow-up for suspected endometrial carcinoma if she reports which of the following? A) Use of oral contraceptives between ages 18 and 25 B) Onset of painless, red postmenopausal bleeding C) Menopause occurring at age 46 D) Use of intrauterine device for 3 years 2. Which of the following instructions would the nurse include when preparing a woman for a Pap smear? A) “Refrain from sexual intercourse for 1 week before the test.” B) “Wear cotton panties on the day of the test.” C) “Avoid taking any medications for 24 hours.” D) “Do not douche for 48 hours before the test.” 3. A woman comes to the clinic for a routine checkup. A history of exposure to which of the following would alert the nurse that she is at increased risk for cervical cancer? A) Hepatitis B) Human papillomavirus C) Cytomegalovirus D) Epstein-Barr virus 4. A client is scheduled to have a Pap smear. After the nurse teaches the client about the Pap smear, which of the following client statements indicates successful teaching? A) “I need to douche the night before with a mild vinegar solution.” B) “I will take a bath first thing that morning to make sure I’m clean.” C) “I will not engage in sexual intercourse for 48 hours before the test.” D) “I will get a clean urine specimen when I first wake up the morning of the test.” 5. Which finding obtained during a client history would the nurse identify as increasing a client’s risk for ovarian cancer? A) Multiple sexual partners B) Consumption of a high-fat diet C) Underweight D) Grand multiparity (more than five children) 6. A client is scheduled for cryosurgery to remove some abnormal tissue on the cervix. The nurse teaches the client about this treatment, explaining that the tissue will be removed by which method? A) Freezing B) Cutting C) Burning D) Irradiating 7. Which of the following statements best indicates that a client has taken self-care measures to reduce her risk for cervical cancer? A) “I’ve really cut down on the amount of caffeine I drink every day.” B) “I’ve thrown out all my bubble baths and just use soap and water now.” C) “Every time I have sexual intercourse, I douche.” D) “My partner always uses a condom when we have sexual intercourse.” 8. A client is suspected of having endometrial cancer. The nurse would most likely prepare the client for which procedure to confirm the diagnosis? A) Transvaginal ultrasound B) Colposcopy C) Pap smear D) Endometrial biopsy 9. Which of the following descriptions would the nurse include when teaching a client about her scheduled colposcopy? A) “A gel will be applied to your abdomen and a microphone-like device will be moved over the area to identify problem areas.” B) “A solution will be wiped on your cervix to identify any abnormal cells, which will be visualized with a magnifying instrument.” C) “Scrapings of tissue will be obtained and placed on slides to be examined under the microscope.” D) “After you receive anesthesia, a small device will be inserted into your abdomen near your belly button to obtain tissue samples.” 10. The nurse is preparing a presentation for a local women’s group about ways to reduce the risk of reproductive tract cancers. Which of the following would the nurse include? A) Blood pressure evaluation every 6 months B) Yearly Pap smears starting at age 40 C) Yearly cholesterol screening starting at age 45 D) Consumption of two to three glasses of red wine per day 11. The daughter of a woman who has been diagnosed with ovarian cancer asks the nurse about screening for this cancer. Which response by the nurse would be most appropriate? A) “Currently there is no reliable screening test for ovarian cancer.” B) “A Pap smear is almost always helpful in identifying this type of cancer.” C) “There’s a blood test for a marker, CA-125, that if elevated indicates cancer.” D) “A genetic test for two genes, if positive, will identify the ovarian cancer.” 12. Which of the following would the nurse be least likely to suggest when teaching a group of young women how to reduce their risk for ovarian cancer? A) Pregnancy B) Oral contraceptives C) Feminine hygiene sprays D) Breast-feeding 13. A woman is diagnosed with adenocarcinoma of the endometrium in situ. The nurse interprets this as indicating which of the following about the cancer? A) Spread to the uterine muscle wall B) Found on the endometrial surface C) Spread to the cervix D) Invaded the bladder 14. When preparing a woman with suspected vulvar cancer for a biopsy, the nurse expects that the lesion would most likely be located at which area? A) Labia majora B) Labia minora C) Clitoris D) Prepuce 15. When describing the various types of reproductive tract cancers to a local women’s group, which of the following would the nurse identify as the least common type? A) Vulvar B) Vaginal C) Endometrial D) Ovarian 16. When assessing a female client for the possibility of vulvar cancer, which of the following would the nurse most likely expect the client to report? (Select all that apply.) A) Abnormal vaginal bleeding B) Persistent vulvar itching C) History of herpes simplex D) Lesion on the cervix E) Abnormal Pap smear 17. A nurse is reviewing the medical record of a woman diagnosed with vulvar cancer. Which of the following would the nurse identify as a risk factor for this cancer? (Select all that apply.) A) Age under 40 years B) HPV 16 exposure C) Monogamous sexual partner D) Hypertension E) Diabetes 18. A nurse is assisting with the collection of a Pap smear. When collecting the specimen, which of the following is done first? A) Insertion of the speculum B) Swabbing of the endocervix C) Spreading of the labia D) Insertion of the cytobrush 19. The plan of care for a woman diagnosed with a suspected reproductive cancer includes a nursing diagnosis of disturbed body image related to suspected reproductive tract cancer and impact on sexuality as evidenced by the client’s statement that she is worried that she won’t be the same. Which of the following would be an appropriate outcome for this client? A) Client will verbalize positive statements about self and sexuality. B) Client will demonstrate understanding of the condition and associated treatment. C) Client will exhibit positive coping strategies related to diagnosis. D) Client will identify misconceptions related to her diagnosis. 20. During a routine health check-up, a young adult woman asks the nurse about ways to prevent endometrial cancer. Which of the following would the nurse most likely include? (Select all that apply.) A) Eating a high-fat diet B) Having regular pelvic exams C) Engaging in daily exercise D) Becoming pregnant E) Using estrogen contraceptives 21. After teaching a group of students about cervical cancer, the instructor determines that the teaching was successful when the students identify which of the following as the area included with a cone biopsy? A) Clitoris B) Uterine fundus C) Ovarian follicle D) Transformation zone 22. A woman is scheduled for diagnostic testing to evaluate for endometrial cancer. The nurse would expect to prepare the woman for which of the following? A) CA-125 testing B) Transvaginal ultrasound C) Pap smear D) Mammography 23. A nurse is conducting a class for a local woman’s group about recommendations for a Pap smear. One of the participants asks, “At what age should a woman have her first Pap smear?” The nurse responds by stating that a woman should have her first Pap smear at which age? A) 18 B) 21 C) 25 D) 28 24. An instructor is describing the development of cervical cancer to a group of students. The instructor determines that the teaching was successful when the students identify which area as most commonly involved? A) Internal cervical os B) Junction of the cervix and fundus C) Squamous-columnar junction D) External cervical os 25. A client has an abnormal Pap smear that is classified as ASC-US. Based on the nurse’s understanding of this classification, the nurse would expect which of the following? A) Immediate colposcopy B) Testing for HPV C) Repeat Pap smear in 4 to 6 months D) Cone biopsy Answer Key 1. B 2. D 3. B 4. C 5. B 6. A 7. D 8. D 9. B 10. C 11. A 12. C 13. B 14. A 15. B 16. B, C 17. B, D, E 18. C 19. A 20. B, C, D 21. D 22. B 23. B 24. C 25. C Chapter 9- Violence and Abuse 1. The nurse is presenting a class at a local community health center on violence during pregnancy. Which of the following would the nurse include as a possible complication? A) Hypertension of pregnancy B) Chorioamnionitis C) Placenta previa D) Postterm labor 2. Which approach would be most appropriate when counseling a woman who is a suspected victim of violence? A) Offer her a pamphlet about the local battered women’s shelter. B) Call her at home to ask her some questions about her marriage. C) Wait until she comes in a few more times to make a better assessment. D) Ask, “Have you ever been physically hurt by your partner?” 3. When describing an episode, the victim reports that she attempted to calm her partner down to keep things from escalating. This behavior reflects which phase of the cycle of violence? A) Battering B) Honeymoon C) Tension-building D) Reconciliation 4. A battered pregnant woman reports to the nurse that her husband has stopped hitting her and promises never to hurt her again. Which of the following is an appropriate response? A) “That’s great. I wish you both the best.” B) “The cycle of violence often repeats itself.” C) “He probably didn’t mean to hurt you.” D) “You need to consider leaving him.” 5. Which of the following nursing actions would be least helpful for a client who is a victim of violence? A) Assist the client to project her anger. B) Provide information about a safe home and crisis line. C) Teach her about the cycle of violence. D) Discuss her legal and personal rights. 6. When describing the cycle of violence to a community group, the nurse explains that the first phase usually is: A) Somehow triggered by the victim’s behavior B) Characterized by tension-building and minor battery C) Associated with loss of physical and emotional control D) Like a honeymoon that lulls the victim 7. Which of the following statements would be most appropriate to empower victims of violence to take action? A) “Give your partner more time to come around.” B) “Remember—children do best in two-parent families.” C) “Change your behavior so as not to trigger the violence.” D) “You are a good person and you deserve better than this.” 8. When a nurse suspects that a client may have been abused, the first action should be to: A) Ask the client about the injuries and if they are related to abuse. B) Encourage the client to leave the batterer immediately. C) Set up an appointment with a domestic violence counselor. D) Ask the suspected abuser about the victim’s injuries. 9. Which of the following would the nurse describe as a characteristic of the second phase of the cycle of violence? A) The batterer is contrite and attempts to apologize for the behavior. B) The physical battery is abrupt and unpredictable. C) Verbal assaults begin to escalate toward the victim. D) The victim accepts the anger as legitimately directed at her. 10. In addition to providing privacy, which of the following would be most appropriate initially in situations involving suspected abuse? A) Allow the client to have a good cry over the situation. B) Tell the client, “Injuries like these don’t usually happen by accident.” C) Call the police immediately so they can question the victim. D) Ask the abuser to describe his side of the story first. 11. When the nurse is alone with a client, the client says, “It was all my fault. The house was so messy when he got home and I know he hates that.” Which response would be most appropriate? A) “It is not your fault. No one deserves to be hurt. ” B) “What else did you do to make him so angry with you?” C) “You need to start to clean the house early in the day.” D) “Remember, he works hard and you need to meet his needs.” 12. When developing a presentation for a local community organization on violence, the nurse is planning to include statistics on intimate partner abuse and its effects on children. In what percentage of the cases in which a parent is abused are the children battered also? A) 50% to 75% B) 25% to 50% C) 10% to 25% D) Less than 5% 13. The primary goal when working with victims of intimate partner violence is to: A) Convince them to leave the abuser soon B) Help them cope with their life as it is C) Empower them to regain control of their life D) Arrest the abuser so he or she can’t abuse again 14. Teaching for victims who are recovering from abusive situations must focus on ways to: A) Enhance their personal appearance and hairstyle B) Develop their creativity and work ethic C) Improve their communication skills and assertiveness D) Plan more nutritious meals to improve their own health 15. During a follow-up visit to the clinic, a victim of sexual assault reports that she has changed her job and moved to another town. She tells the nurse, “I pretty much stay to myself at work and at home.” The nurse interprets these findings to indicate that the client is in which phase of rape recovery? A) Disorganization B) Denial C) Reorganization D) Integration 16. A nurse is assessing a rape survivor for post-traumatic stress disorder. The nurse asks the woman, “Do you feel as though you are reliving the trauma?” The nurse is assessing for which of the following? A) Physical symptoms B) Intrusive thoughts C) Avoidance D) Hyperarousal 17. A group of students are preparing a class discussion about rape and sexual assault. Which of the following would the students include as being most accurate? (Select all that apply.) A) Most victims of rape tell someone about it. B) Few women falsely cry “rape.” C) Women have rape fantasies desiring to be raped. D) A rape victim feels vulnerable and betrayed afterwards. E) Medication and counseling can help a rape victim cope. 18. After teaching a class on date rape, the instructor determines that the teaching was successful when the class identifies which of the following as the most common date rape drug? A) Gamma hydroxybutyrate B) Liquid ecstasy C) Ketamine D) Rohypnol 19. A nurse is caring for a woman who was recently raped. The nurse would expect this woman to experience which of the following first? A) Denial B) Disorganization C) Reorganization D) Integration 20. A group of nurses are researching information about risk factors for intimate partner violence in men. Which of the following would the nurses expect to find related to the individual person? (Select all that apply.) A) Dysfunctional family system B) Low academic achievement C) Victim of childhood violence D) Heavy alcohol consumption E) Economic stress 21. A nurse is working with a victim of intimate partner violence and helping her develop a safety plan. Which of the following would the nurse suggest that the woman take with her? (Select all that apply.) A) Driver’s license B) Social security number C) Cash D) Phone cards E) Health insurance cards 22. A nurse is presenting a discussion on sexual violence at a local community college. When describing the incidence of sexual violence, the nurse would identify that a woman has which chance of experiencing a sexual assault in her lifetime? A) One in three B) One in six C) Two in 15 D) Three in 20 23. After teaching a class on sexual violence, the instructor determines that the teaching was successful when the class identifies which of the following as a type of sexual violence. (Select all that apply.) A) Female genital cutting B) Bondage C) Infanticide D) Human trafficking E) Rape 24. A nurse is reading a journal article about sexual abuse. Which age range would the nurse expect to find as the peak age for such abuse? A) 7–10 years B) 8–12 years C) 14–18 years D) 18–22 years 25. After teaching a group of students about sexual abuse and violence, the instructor determines that the teaching was successful when the students describe incest as involving which of the following? A) Sexual exploitation by blood or surrogate relatives B) Sexual abuse of individuals over age 18 C) Violent aggressive assault on a person D) Consent between perpetrator and victim. Answer Key 1. B 2. D 3. C 4. B 5. A 6. B 7. D 8. A 9. B 10. B 11. A 12. A 13. C 14. C 15. C 16. B 17. B, D, E 18. D 19. B 20. B, C, D 21. A, B, C, E 22. B 23. A, B, C, D, E 24. B 25. A Chapter 10- Fetal Development and Genetics 1. While talking with a pregnant woman who has undergone genetic testing, the woman informs the nurse that her baby will be born with Down syndrome. The nurse understands that Down syndrome is an example of: A) Multifactorial inheritance B) X-linked recessive inheritance C) Trisomy numeric abnormality D) Chromosomal deletion 2. A nurse is describing advances in genetics to a group of students. Which of the following would the nurse least likely include? A) Genetic diagnosis is now available as early as the second trimester. B) Genetic testing can identify presymptomatic conditions in children. C) Gene therapy can be used to repair missing genes with normal ones. D) Genetic agents may be used in the future to replace drugs. 3. After teaching a group of students about fetal development, the instructor determines that the teaching was successful when the students identify which of the following as providing the barrier to other sperm after fertilization? A) Zona pellucida B) Zygote C) Cleavage D) Morula 4. A nurse is teaching a class on X-linked recessive disorders. Which of the following statements would the nurse most likely include? A) Males are typically carriers of the disorders. B) No male-to-male transmission occurs. C) Daughters are more commonly affected with the disorder. D) Both sons and daughters have a 50% risk of the disorder. 5. A pregnant woman undergoes maternal serum alpha-fetoprotein (MSAFP) testing at 16 to 18 weeks’ gestation. Which of the following would the nurse suspect if the woman’s level is decreased? A) Down syndrome B) Sickle-cell anemia C) Cardiac defects D) Open neural tube defect 6. The nurse is developing a presentation for a community group of young adults discussing fetal development and pregnancy. The nurse would identify that the sex of offspring is determined at the time of: A) Meiosis B) Fertilization C) Formation of morula D) Oogenesis 7. When describing amniotic fluid to a pregnant woman, the nurse would include which of the following? A) “This fluid acts as transport mechanism for oxygen and nutrients.” B) “The fluid is mostly protein to provide nourishment to your baby.” C) “This fluid acts as a cushion to help to protect your baby from injury.” D) “The amount of fluid remains fairly constant throughout the pregnancy.” 8. Assessment of a pregnant woman reveals oligohydramnios. The nurse would be alert for the development of which of the following? A) Maternal diabetes B) Placental insufficiency C) Neural tube defects D) Fetal gastrointestinal malformations 9. A couple comes to the clinic for preconception counseling and care. As part of the visit, the nurse teaches the couple about fertilization and initial development, stating that the zygote formed by the union of the ovum and sperm consists of how many chromosomes? A) 22 B) 23 C) 44 D) 46 10. A woman just delivered a healthy term newborn. Upon assessing the umbilical cord, the nurse would identify which of the following as normal? (Select all that apply.) A) One vein B) Two veins C) One artery D) Two arteries E) One ligament F) Two ligaments 11. After teaching a pregnant woman about the hormones produced by the placenta, the nurse determines that the teaching was successful when the woman identifies which hormone produced as being the basis for pregnancy tests? A) Human placental lactogen (hPL) B) Estrogen (estriol) C) Progesterone (progestin) D) Human chorionic gonadotropin (hCG) 12. After the nurse describes fetal circulation to a pregnant woman, the woman asks why her fetus has a different circulation pattern than hers. In planning a response, the nurse integrates understanding of which of the following? A) Fetal blood is thicker than that of adults and needs different pathways. B) Fetal circulation carries highly oxygenated blood to vital areas first. C) Fetal blood has a higher oxygen saturation and circulates more slowly. D) Fetal heart rates are rapid and circulation time is double that of adults. 13. When describing genetic disorders to a group of childbearing couples, the nurse would identify which as an example of an autosomal dominant inheritance disorder? A) Huntington’s disease B) Sickle cell disease C) Phenylketonuria D) Cystic fibrosis 14. Prenatal testing is used to assess for genetic risks and to identify genetic disorders. In explaining to a couple about an elevated alpha-fetoprotein screening test result, the nurse would discuss the need for: A) Special care needed for a Down syndrome infant B) A more specific determination of the acid–base status C) Further, more definitive evaluations to conclude anything D) Immediate termination of the pregnancy based on results 15. A nursing instructor is preparing a teaching plan for a group of nursing students about the potential for misuse of genetic discoveries and advances. Which the following would the instructor most likely include? A) Gene replacement therapy for defective genes B) Individual risk profiling and confidentiality C) Greater emphasis on the causes of diseases D) Slower diagnosis of specific diseases 16. After teaching a class on the stages of fetal development, the instructor determines that the teaching was successful when the students identify which of the following as a stage? (Select all that apply.) A) Placental B) Preembryonic C) Umbilical D) Embryonic E) Fetal 17. A nurse is discussing fetal development with a pregnant woman. The woman is 12 weeks pregnant and asks, “What’s happening with my baby?” Which of the following would the nurse integrate into the response? (Select all that apply.) A) Continued sexual differentiation B) Eyebrows forming C) Startle reflex present D) Digestive system becoming active E) Lanugo present on the head 18. After teaching a group of students about fetal development, the instructor determines that the teaching was successful when the students identify which of the following as essential for fetal lung development? A) Umbilical cord B) Amniotic fluid C) Placenta D) Trophoblasts 19. During a prenatal class for a group of new mothers, the nurse is describing the hormones produced by the placenta. Which of the following would the nurse include? (Select all that apply.) A) Prolactin B) Estriol C) Relaxin D) Progestin E) Human chorionic somatomammotropin 20. When describing the structures involved in fetal circulation, the nursing instructor describes which structure as the opening between the right and left atrium? A) Ductus venosus B) Foramen ovale C) Ductus arteriosus D) Umbilical artery 21. A group of students are reviewing information about genetic inheritance. The students demonstrate understanding of the information when they identify which of the following as an example of an autosomal recessive disorder? (Select all that apply.) A) Cystic fibrosis B) Phenylketonuria C) Tay-Sachs disease D) Polycystic kidney disease E) Achondroplasia 22. A nurse is assessing a child with Klinefelter’s syndrome. Which of the following would the nurse expect to assess? (Select all that apply.) A) Gross mental retardation B) Long arms C) Profuse body hair D) Gynecomastia E) Enlarged testicles 23. A woman is scheduled to undergo fetal nuchal translucency testing. Which of the following would the nurse include when describing this test? A) “A needle will be inserted directly into the fetus’s umbilical vessel.” B) “You’ll have an intravaginal ultrasound to measure fluid in the fetus.” C) “The doctor will take a sample of fluid from your bag of waters.” D) “A small piece of tissue from the fetal part of the placenta is taken.” Answer Key 1. C 2. A 3. A 4. B 5. A 6. B 7. C 8. B 9. D 10. A 11. D 12. B 13. A 14. C 15. B 16. B, C, E 17. A, D 18. B 19. B, C, D, E 20. B 21. A 22. B, D 23. B Chapter 11- Maternal Adaptation During Pregnancy 1. During a vaginal exam, the nurse notes that the cervix has a bluish color. The nurse documents this finding as: A) Hegar’s sign B) Goodell’s sign C) Chadwick’s sign D) Ortolani’s sign 2. The nurse teaches a primigravida client that lightening occurs about 2 weeks before the onset of labor. The mother will most likely experience which of the following at that time? A) Dysuria B) Dyspnea C) Constipation D) Urinary frequency 3. A gravida 2 para 1 client in the 10th week of her pregnancy says to the nurse, “I’ve never urinated as often as I have for the past three weeks.” Which response would be most appropriate for the nurse to make? A) “Having to urinate so often is annoying. I suggest that you watch how much fluid you are drinking and limit it.” B) “You shouldn’t be urinating this frequently now; it usually stops by the time you’re eight weeks pregnant. Is there anything else bothering you?” C) “By the time you are 12 weeks pregnant, this frequent urination should no longer be a problem, but it is likely to return toward the end of your pregnancy.” D) “Women having their second child generally don’t have frequent urination. Are you experiencing any burning sensations?” 4. In a client’s seventh month of pregnancy, she reports feeling “dizzy, like I’m going to pass out, when I lie down flat on my back.” The nurse integrates which of the following in to the explanation? A) Pressure of the gravid uterus on the vena cava B) A 50% increase in blood volume C) Physiologic anemia due to hemoglobin decrease D) Pressure of the presenting fetal part on the diaphragm 5. A primiparous client is being seen in the clinic for her first prenatal visit. It is determined that she is 11 weeks pregnant. The nurse develops a teaching plan to educate the client about what she will most likely experience during this period. Which of the following would the nurse include? A) Ankle edema B) Urinary frequency C) Backache D) Hemorrhoids 6. A pregnant client in her second trimester has a hemoglobin level of 11 g/dL. The nurse interprets this as indicating which of the following? A) Iron-deficiency anemia B) A multiple gestation pregnancy C) Greater-than-expected weight gain D) Hemodilution of pregnancy 7. The nurse is discussing the insulin needs of a primaparous client with diabetes who has been using insulin for the past few years. The nurse informs the client that her insulin needs will increase during pregnancy based on the nurse’s understanding that the placenta produces: A) hCG, which increases maternal glucose levels B) hPL, which deceases the effectiveness of insulin C) Estriol, which interferes with insulin crossing the placenta D) Relaxin, which decreases the amount of insulin produced 8. When teaching a pregnant client about the physiologic changes of pregnancy, the nurse reviews the effect of pregnancy on glucose metabolism. Which of the following would the nurse include as the underlying reason for the effect? A) Pancreatic function is affected by pregnancy. B) Glucose is utilized more rapidly during a pregnancy. C) The pregnant woman increases her dietary intake. D) Glucose moves through the placenta to assist the fetus. 9. When assessing a woman in her first trimester, which emotional response would the nurse most likely expect to find? A) Ambivalence B) Introversion C) Acceptance D) Emotional lability 10. The nurse is assessing a pregnant woman in the second trimester. Which of the following tasks would indicate to the nurse that the client is incorporating the maternal role into her personality? A) The woman demonstrates concern for herself and her fetus as a unit. B) The client identifies what she must give up to assume her new role. C) The woman acknowledges the fetus as a separate entity within her. D) The client demonstrates unconditional acceptance without rejection. 11. A woman comes to the prenatal clinic suspecting that she is pregnant, and assessment reveals probable signs of pregnancy. Which of the following would be included as part of this assessment? (Select all that apply.) A) Positive pregnancy test B) Ultrasound visualization of the fetus C) Auscultation of a fetal heart beat D) Ballottement E) Absence of menstruation F) Softening of the cervix 12. The nurse is teaching a pregnant woman about recommended weight gain. The woman has a prepregnancy body mass index of 26. The nurse determines that the teaching was successful when the woman states that she should gain no more than which amount during pregnancy? A) 35 to 40 pounds B) 25 to 35 pounds C) 28 to 40 pounds D) 15 to 25 pounds 13. A nurse strongly encourages a pregnant client to avoid eating swordfish and tilefish because these fish contain which of the following? A) Excess folic acid, which could increase the risk for neural tube defects B) Mercury, which could harm the developing fetus if eaten in large amounts C) Lactose, which leads to abdominal discomfort, gas, and diarrhea D) Low-quality protein that does not meet the woman’s requirements 14. Which of the following changes in the musculoskeletal system would the nurse mention when teaching a group of pregnant women about the physiologic changes of pregnancy? A) Ligament tightening B) Decreased swayback C) Increased lordosis D) Joint contraction 15. Assessment of a pregnant woman reveals a pigmented line down the middle of her abdomen. The nurse documents this as which of the following? A) Linea nigra B) Striae gravidarum C) Melasma D) Vascular spiders 16. A nurse is assessing a pregnant woman on a routine checkup. When assessing the woman’s gastrointestinal tract, which of the following would the nurse expect to find? (Select all that apply.) A) Hyperemic gums B) Increased peristalsis C) Complaints of bloating D) Heartburn E) Nausea 17. A woman suspecting she is pregnant asks the nurse about which signs would confirm her pregnancy. The nurse would explain that which of the following would confirm the pregnancy? A) Absence of menstrual period B) Abdominal enlargement C) Palpable fetal movement D) Morning sickness 18. A nurse is developing a teaching plan about nutrition for a group of pregnant women. Which of the following would the nurse include in the discussion? (Select all that apply.) A) Keep weight gain to 15 lb B) Eat three meals with snacking C) Limit the use of salt in cooking D) Avoid using diuretics E) Participate in physical activity 19. Assessment of a pregnant woman reveals that she compulsively craves ice. The nurse documents this finding as which of the following? A) Quickening B) Pica C) Ballottement D) Linea nigra 20. A woman in her second trimester comes for a follow-up visit and says to the nurse, “I feel like I’m on an emotional rollercoaster.” Which response by the nurse would be most appropriate? A) “How often has this been happening to you?” B) “Maybe you need some medication to level things out.” C) “Mood swings are completely normal during pregnancy.” D) “Have you been experiencing any thoughts of harming yourself?” 21. While talking with a woman in her third trimester, which behavior indicates to the nurse that the woman is learning to give of oneself? A) Showing concern for self and fetus as a unit B) Unconditionally accepting the pregnancy without rejection C) Longing to hold infant D) Questioning ability to become a good mother 22. A group of students are reviewing the signs of pregnancy. The students demonstrate understanding of the information when they identify which as presumptive signs? (Select all that apply.) A) Amenorrhea B) Nausea C) Abdominal enlargement D) Braxton-Hicks contractions E) Fetal heart sounds 23. A nursing instructor is teaching a class to a group of students about pregnancy, insulin, and glucose. Which of the following would the instructor least likely include as opposing insulin? A) Prolactin B) Estrogen C) Progesterone D) Cortisol 24. A woman is at 20 weeks’ gestation. The nurse would expect to find the fundus at which of the following? A) Just above the symphysis pubis B) Mid-way between the pubis and umbilicus C) At the level of the umbilicus D) Mid-way between the umbilicus and xiphoid process 25. A pregnant woman comes to the clinic and tells the nurse that she has been having a whitish vaginal discharge. The nurse suspects vulvovaginal candidiasis based on which assessment finding? A) Fever B) Vaginal itching C) Urinary frequency D) Incontinence Answer Key 1. C 2. D 3. C 4. A 5. B 6. D 7. B 8. D 9. A 10. C 11. A, D, F 12. D 13. B 14. C 15. A 16. A, C, D, E 17. C 18. B, D, E 19. B 20. C 21. D 22. A, B 23. D 24. C 25. B Chapter 12- Nursing Management During Pregnancy 1. A woman in the 34th week of pregnancy says to the nurse, “I still feel like having intercourse with my husband.” The woman’s pregnancy has been uneventful. The nurse responds based on the understanding that: A) It is safe to have intercourse at this time. B) Intercourse at this time is likely to cause rupture of membranes. C) There are other ways that the couple can satisfy their needs. D) Intercourse at this time is likely to result in premature labor. 2. On the first prenatal visit, examination of the woman’s internal genitalia reveals a bluish coloration of the cervix and vaginal mucosa. The nurse records this finding as: A) Hegar’s sign B) Goodell’s sign C) Chadwick’s sign D) Homans’ sign 3. When describing perinatal education to a pregnant woman and her partner, the nurse emphasizes that the primary goal of these classes is to: A) Equip a couple with the knowledge to experience a pain-free childbirth B) Provide knowledge and skills to actively participate in birth and parenting C) Eliminate anxiety so that they can have an uncomplicated birth D) Empower the couple to totally control the birth process 4. When assessing a woman at follow-up prenatal visits, the nurse would anticipate which of the following to be performed? A) Hemoglobin and hematocrit B) Urine for culture C) Fetal ultrasound D) Fundal height measurement 5. During a routine prenatal visit, a client, 36 weeks pregnant, states she has difficulty breathing and feels like her pulse rate is really fast. The nurse finds her pulse to be 100 beats per minute (increased from baseline readings of 70 to 74 beats per minute and irregular, with bilateral crackles in the lower lung bases. Which nursing diagnosis would be the priority for this client? A) Ineffective tissue perfusion related to supine hypotensive syndrome B) Impaired gas exchange related to pulmonary congestion C) Activity intolerance related to increased metabolic requirements D) Anxiety related to fear of pregnancy outcome 6. When preparing a woman for an amniocentesis, the nurse would instruct her to do which of the following? A) Shower with an antiseptic scrub. B) Swallow the preprocedure sedative. C) Empty her bladder. D) Lie on her left side. 7. A client who is 4 months pregnant is at the prenatal clinic for her initial visit. Her history reveals she has 7-year-old twins who were born at 34 weeks gestation, a 2-year old son born at 39 weeks gestation, and a spontaneous abortion 1 year ago at 6 weeks gestation. Using the GTPAL method, the nurse would document her obstetric history as: A) 3 2 1 0 3 B) 3 1 2 2 3 C) 4 1 1 1 3 D) 4 2 1 3 1 8. A client’s last menstrual period was April 11. Using Nagele’s rule, her expected date of birth (EDB). would be: A) January 4 January 18 January 25 February 24 9. During a nonstress test, when monitoring the fetal heart rate, the nurse notes that when the expectant mother reports fetal movement, the heart rate increases 15 beats or more above the baseline. The nurse interprets this as: A) Variable decelerations B) Fetal tachycardia C) A nonreactive pattern D) Reactive pattern 10. A client’s maternal serum alpha-fetoprotein (MSAFP. level was unusually elevated at 17 weeks. The nurse suspects which of the following? A) Fetal hypoxia B) Open spinal defects C) Down syndrome D) Maternal hypertension 11. When assessing a pregnant woman in her last trimester, which question would be most appropriate to use to gather information about weight gain and fluid retention? A) “What’s your usual dietary intake for a typical day?” B) “What size maternity clothes are you wearing now?” C) “How puffy does your face look by the end of a day?” D) “How swollen do your ankles appear before you go to bed? 12. A pregnant woman in the 36th week of gestation complains that her feet are quite swollen at the end of the day. After careful assessment, the nurse determines that this is an expected finding at this stage of pregnancy. Which intervention would be most appropriate for the nurse to suggest? A) “Limit your intake of fluids.” B) “Eliminate salt from your diet.” C) “Try elevating your legs when you sit.” D) “Wear Spandex-type full-length pants.” 13. A pregnant woman needs an update in her immunizations. Which of the following vaccinations would the nurse ensure that the woman receives? A) Measles B) Mumps C) Rubella D) Hepatitis B 14. A pregnant woman is flying across the country to visit her family. After teaching the woman about traveling during pregnancy, which statement indicates that the teaching was successful? A) “I’ll sit in a window seat so I can focus on the sky to help relax me.” “I won’t drink too much fluid so I don’t have to urinate so often.” “I’ll get up and walk around the airplane about every 2 hours.” “I’ll do some upper arm stretches while sitting in my seat.” 15. Which of the following would the nurse include when teaching a pregnant woman about chorionic villus sampling? A) “The results should be available in about a week.” B) “You’ll have an ultrasound first and then the test.” C) “Afterwards, you can resume your exercise program.” D) “This test is very helpful for identifying spinal defects.” 16. A pregnant woman is scheduled to undergo percutaneous umbilical blood sampling. When discussing this test with the woman, the nurse reviews what can be evaluated with the specimens collected. Which of the following would the nurse include? (Select all that apply.) A) Rh incompatibility B) Fetal acid–base status C) Sex-linked disorders D) Enzyme deficiencies E) Coagulation studies 17. A biophysical profile has been completed on a pregnant woman. The nurse interprets which score as normal? A) 9 B) 7 C) 5 D) 3 18. After teaching a group of students about the discomforts of pregnancy, the students demonstrate understanding of the information when they identify which as common during the first trimester? (Select all that apply.) A) Urinary frequency B) Breast tenderness C) Cravings D) Backache E) Leg cramps 19. A nurse is reviewing the medical record of a pregnant woman and notes that she is gravid II. The nurse interprets this to indicate the number of: A) Deliveries B) Pregnancies C) Spontaneous abortions D) Pre-term births 20. A nurse measures a pregnant woman’s fundal height and finds it to be 28 cm. The nurse interprets this to indicate which of the following? A) 14 weeks’ gestation 20 weeks’ gestation 28 weeks’ gestation 36 weeks’ gestation 21. A pregnant woman has a rubella titer drawn on her first prenatal visit. The nurse explains that this test measures which of the following? A) Platelet level B) Rh status C) Immunity to German measles D) Red blood cell count 22. A nurse is working with a pregnancy woman to schedule follow-up visits for her pregnancy. Which statement by the woman indicates that she understands the scheduling? A) “I need to make visits every 2 months until I’m 36 weeks pregnant.” B) “Once I get to 28 weeks, I have to come twice a month.” C) “From now until I’m 28 weeks, I’ll be coming once a month.” D) “I’ll make sure to get a day off every 2 weeks to make my visits.” 23. A nursing instructor is describing the various childbirth methods. Which of the following would the instructor include as part of the Lamaze method? A) Focus on the pleasurable sensations of childbirth B) Concentration on sensations while turning on to own bodies C) Interruption of the fear-tension-pain cycle D) Use of specific breathing and relaxation techniques 24. After teaching a group of students about the different perinatal education methods, the instructor determines that the teaching was successful when the students identify which of the following as the Bradley method? A) Psychoprophylactic method B) Partner-coached method C) Natural childbirth method D) Mind prevention method 25. A pregnant woman in her second trimester tells the nurse, “I’ve been passing a lot of gas and feel bloated.” Which of the following suggestions would be helpful for the woman? A) “Watch how much beans and onions you eat.” B) “Limit the amount of fluid you drink with meals” C) “Try exercising a little more.” D) “Some say that eating mints can help.” E) “Cut down on your intake of cheeses.” Answer Key 1. A 2. C 3. B 4. D 5. B 6. C 7. C 8. B 9. D 10. B 11. D 12. C 13. D 14. C 15. B 16. A, B, E 17. A 18. A, B, C 19. B 20. C 21. C 22. C 23. D 24. B 25. A, C, D Chapter 13- Labor and Birth Process 1. A woman in her 40th week of pregnancy calls the nurse at the clinic and says she’s not sure whether she is in true or false labor. Which statement by the client would lead the nurse to suspect that the woman is experiencing false labor? A) “I’m feeling contractions mostly in my back.” B) “My contractions are about 6 minutes apart and regular.” C) “The contractions slow down when I walk around.” D) “If I try to talk to my partner during a contraction, I can’t.” 2. Which of the following would indicate to the nurse that the placenta is separating? A) Uterus becomes globular B) Fetal head is at vaginal opening C) Umbilical cord shortens D) Mucous plug is expelled 3. When assessing cervical effacement of a client in labor, the nurse assesses which of the following characteristics? A) Extent of opening to its widest diameter B) Degree of thinning C) Passage of the mucous plug D) Fetal presenting part 4. A woman calls the health care facility stating that she is in labor. The nurse would urge the client to come to the facility if the client reports which of the following? A) Increased energy level with alternating strong and weak contractions B) Moderately strong contractions every 4 minutes, lasting about 1 minute C) Contractions noted in the front of abdomen that stop when she walks D) Pink-tinged vaginal secretions and irregular contractions lasting about 30 seconds 5. A woman is in the first stage of labor. The nurse would encourage her to assume which position to facilitate the progress of labor? A) Supine B) Lithotomy C) Upright D) Knee–chest 6. A client has not received any medication during her labor. She is having frequent contractions every 1 to 2 minutes and has become irritable with her coach and no longer will allow the nurse to palpate her fundus during contractions. Her cervix is 8 cm dilated and 90% effaced. The nurse interprets these findings as indicating: A) Latent phase of the first stage of labor B) Active phase of the first stage of labor C) Transition phase of the first stage of labor D) Pelvic phase of the second stage of labor 7. The fetus of a nulliparous woman is in a shoulder presentation. The nurse would most likely prepare the client for which type of birth? A) Cesarean B) Vaginal C) Forceps-assisted D) Vacuum extraction 8. Assessment of a woman in labor reveals cervical dilation of 3 cm, cervical effacement of 30%, and contractions occurring every 7 to 8 minutes, lasting about 40 seconds. The nurse determines that this client is in: A) Latent phase of the first stage B) Active phase of the first stage C) Transition phase of the first stage D) Perineal phase of the second stage 9. A client is admitted to the labor and birthing suite in early labor. On review of her medical record, the nurse determines that the client’s pelvic shape as identified in the antepartal progress notes is the most favorable one for a vaginal delivery. Which pelvic shape would the nurse have noted? A) Platypelloid B) Gynecoid C) Android D) Anthropoid 10. A woman telephones her health care provider and reports that her “water just broke.” Which suggestion by the nurse would be most appropriate? A) “Call us back when you start having contractions.” B) “Come to the clinic or emergency department for an evaluation.” C) “Drink 3 to 4 glasses of water and lie down.” D) “Come in as soon as you feel the urge to push.” 11. After teaching a group of students about the maternal bony pelvis, which statement by the group indicates that the teaching was successful? A) The bony pelvis plays a lesser role during labor than soft tissue. B) The pelvic outlet is associated with the true pelvis. C) The false pelvis lies below the imaginary linea terminalis. D) The false pelvis is the passageway through which the fetus travels. 12. A fetus is assessed at 2 cm above the ischial spines. The nurse would document fetal station as: A) +4 B) +2 C) 0 D) –2 13. Assessment of a fetus identifies the buttocks as the presenting part, with the legs extended upward. The nurse identifies this as which type of breech presentation? A) Frank B) Full C) Complete D) Footling 14. A woman in her third trimester comes to the clinic for a prenatal visit. During assessment the woman reports that her breathing has become much easier in the last week but she has noticed increased pelvic pressure , cramping and lower back pain. The nurse determines that which of the following has most likely occurred? A) Cervical dilation B) Lightening C) Bloody show D) Braxton-Hicks contractions 15. After teaching a group of students about the factors affecting the labor process, the instructor determines that the teaching was successful when the group identifies which of the following as a component of the true pelvis? (Select all that apply.) A) Pelvic inlet B) Cervix C) Mid pelvis D) Pelvic outlet E) Vagina F) Pelvic floor muscles 16. A nurse is documenting fetal lie of a woman in labor. Which term would the nurse most likely use? A) Flexion B) Extension C) Longitudinal D) Cephalic 17. The nurse is reviewing the medical record of a woman in labor and notes that the fetal position is documented as LSA. The nurse interprets this information as indicating which of the following is the presenting part? A) Occiput B) Face C) Buttocks D) Shoulder 18. A nurse is preparing a class for pregnant women about labor and birth. When describing the typical movements that the fetus goes through as it travels through the passageway, which of the following would the nurse most likely include? (Select all that apply.) A) Internal rotation B) Abduction C) Descent D) Pronation E) Flexion 19. The nurse is reviewing the monitoring strip of a woman in labor who is experiencing a contraction. The nurse notes the time the contraction takes from its onset to reach its highest intensity. The nurse interprets this time as which of the following? A) Increment B) Acme C) Peak D) Decrement 20. A nurse is assessing a woman in labor. Which finding would the nurse identify as a cause for concern during a contraction? A) Heart rate increase from 76 bpm to 90 bpm B) Blood pressure rise from 110/60 mm Hg to 120/74 C) White blood cell count of 12,000 cells/mm3 D) Respiratory rate of 10 breaths /minute 21. When describing the stages of labor to a pregnant woman, which of the following would the nurse identify as the major change occurring during the first stage? A) Regular contractions B) Cervical dilation C) Fetal movement through the birth canal D) Placental separation 22. A nurse is caring for several women in labor. The nurse determines that which woman is in the transition phase of labor? A) Contractions every 5 minutes, cervical dilation 3 cm B) Contractions every 3 minutes, cervical dilation 5 cm C) Contractions every 2½ minutes, cervical dilation 7 cm D) Contractions every 1 minute, cervical dilation 9 cm 23. A nurse is preparing a presentation for a group of pregnant women about the labor experience. Which of the following would the nurse most likely include when discussing measures to promote coping for a positive labor experience? (Select all that apply.) A) Presence of a support partner B) View of birth as a stressor C) Low anxiety level D) Fear of loss of control E) Participation in a pregnancy exercise program 24. During a follow-up prenatal visit, a pregnant woman asks the nurse, “How long do you think I will be in labor?” Which response by the nurse would be most appropriate? A) “It’s difficult to predict how your labor will progress, but we’ll be there for you the entire time.” B) “Since this is your first pregnancy, you can estimate it will be about 10 hours.” C) “It will depend on how big the baby is when you go into labor.” D) “Time isn’t important; your health and the baby’s health are key.” 25. A nurse is describing how the fetus moves through the birth canal. Which of the following would the nurse identify as being most important in allowing the fetal head to move through the pelvis? A) Sutures B) Fontanelles C) Frontal bones D) Biparietal diameter 26. Assessment of a pregnant woman reveals that the presenting part of the fetus is at the level of the maternal ischial spines. The nurse documents this as which station? A) –2 B) –1 C) 0 D) +1   Answer Key 1. C 2. A 3. B 4. B 5. C 6. C 7. A 8. A 9. B 10. B 11. B 12. D 13. A 14. B 15. A, C, D 16. C 17. C 18. A, C, E 19. A 20. D 21. B 22. D 23. A, C, E 24. A 25. A 26. C Chapter 14- Nursing Management During Labor and Birth 1. A woman in labor who received an opioid for pain relief develops respiratory depression. The nurse would expect which agent to be administered? A) Butorphanol B) Fentanyl C) Naloxone D) Promethazine 2. A client’s membranes spontaneously ruptured, as evidenced by a gush of clear fluid with a contraction. Which of the following would the nurse do next? A) Check the fetal heart rate. B) Perform a vaginal exam. C) Notify the physician immediately. D) Change the linen saver pad. 3. A woman has just entered the second stage of labor. The nurse would focus care on which of the following? A) Encouraging the woman to push when she has a strong desire to do so B) Alleviating perineal discomfort with the application of ice packs C) Palpating the woman’s fundus for position and firmness D) Completing the identification process of the newborn with the mother 4. The nurse notes persistent early decelerations on the fetal monitoring strip. Which of the following would the nurse do next? A) Continue to monitor the FHR because this pattern is benign. B) Perform a vaginal exam to assess cervical dilation and effacement. C) Stay with the client while reporting the finding to the physician. D) Administer oxygen after turning the client on her left side. 5. A woman is admitted to the labor and birthing suite. Vaginal examination reveals that the presenting part is approximately 2 cm above the ischial spines. The nurse documents this finding as: A) +2 station B) 0 station C) –2 station D) Crowning 6. The nurse is performing Leopold’s maneuvers to determine fetal presentation, position, and lie. Which action would the nurse do first? A) Feel for the fetal buttocks or head while palpating the abdomen. B) Feel for the fetal back and limbs as the hands move laterally on the abdomen. C) Palpate for the presenting part in the area just above the symphysis pubis. D) Determine flexion by pressing downward toward the symphysis pubis. 7. A client states, “I think my waters broke! I felt this gush of fluid between my legs.” The nurse tests the fluid with Nitrazine paper and confirms membrane rupture if the paper turns: A) Yellow B) Olive green C) Pink D) Blue 8. A woman in labor is to receive continuous internal electronic fetal monitoring. The nurse reviews the woman’s medical record to ensure which of the following as being required? A) Intact membranes B) Cervical dilation of 2 cm or more C) Floating presenting fetal part D) A neonatologist to insert the electrode 9. When assessing fetal heart rate, the nurse finds a heart rate of 175 bpm, accompanied by a decrease in variability and late decelerations. Which of the following would the nurse do next? A) Have the woman change her position. B) Administer oxygen. C) Notify the health care provider. D) Continue to monitor the pattern every 15 minutes. 10. A woman in labor has chosen to use hydrotherapy as a method of pain relief. Which statement by the woman would lead the nurse to suspect that the woman needs additional teaching? A) “The warmth and buoyancy of the water has a nice relaxing effect.” B) “I can stay in the bath for as long as I feel comfortable.” C) “My cervix should be dilated more than 5 cm before I try using this method.” D) “The temperature of the water should be at least 105° F.” 11. A woman in labor received an opioid close to the time of birth. The nurse would assess the newborn for which of the following? A) Respiratory depression B) Urinary retention C) Abdominal distention D) Hyperreflexia 12. When applying the ultrasound transducers for continuous external electronic fetal monitoring, at which location would the nurse place the transducer to record the FHR? A) Over the uterine fundus where contractions are most intense B) Above the umbilicus toward the right side of the diaphragm C) Between the umbilicus and the symphysis pubis D) Between the xiphoid process and umbilicus 13. After describing continuous internal electronic fetal monitoring to a laboring woman and her partner, which of the following would indicate the need for additional teaching? A) “This type of monitoring is the most accurate method for our baby.” B) “Unfortunately, I’m going to have to stay quite still in bed while it is in place.” C) “This type of monitoring can only be used after my membranes rupture.” D) “You’ll be inserting a special electrode into my baby’s scalp.” 14. When planning the care of a woman in the active phase of labor, the nurse would anticipate assessing the fetal heart rate at which interval? A) Every 2 to 4 hours B) Every 45 to 60 minutes C) Every 15 to 30 minutes D) Every 10 to 15 minutes 15. Which of the following is a priority when caring for a woman during the fourth stage of labor? A) Assessing the uterine fundus B) Offering fluids as indicated C) Encouraging the woman to void D) Assisting with perineal care 16. When palpating the fundus during a contraction, the nurse notes that it feels like a chin. The nurse interprets this finding as indicating which type of contraction? A) Intense B) Strong C) Moderate D) Mild 17. A nurse palpates a woman’s fundus to determine contraction intensity. Which of the following would be most appropriate for the nurse to use for palpation? A) Finger pads B) Palm of the hand C) Finger tips D) Back of the hand 18. A woman’s amniotic fluid is noted to be cloudy. The nurse interprets this finding as which of the following? A) Normal B) Possible infection C) Meconium passage D) Transient fetal hypoxia 19. After teaching a group of students about fetal heart rate patterns, the instructor determines the need for additional teaching when the students identify which of the following as indicating normal fetal acid–base status? (Select all that apply.) A) Sinusoidal pattern B) Recurrent variable decelerations C) Fetal bradycardia D) Absence of late decelerations E) Moderate baseline variability 20. A nurse is reviewing the fetal heart rate pattern and observes abrupt decreases in FHR below the baseline, appearing as a U-shape. The nurse interprets these changes as reflecting which of the following? A) Early decelerations B) Variable decelerations C) Prolonged decelerations D) Late decelerations 21. A nurse is explaining the use of therapeutic touch as a pain relief measure during labor. Which of the following would the nurse include in the explanation? A) “This technique focuses on manipulating body tissues.” B) “The technique requires focusing on a specific stimulus.” C) “This technique redirects energy fields that lead to pain.” D) “The technique involves light stroking of the abdomen with breathing.” 22. A group of nursing students are reviewing the various medications used for pain relief during labor. The students demonstrate understanding of the information when they identify which agent as the most commonly used opioid? A) Butorphanol B) Nalbuphine C) Fentanyl D) Meperidine 23. A nurse is describing the different types of regional analgesia and anesthesia for labor to a group of pregnant women. Which statement by the group indicates that the teaching was successful? A) “We can get up and walk around after receiving combined spinal–epidural analgesia.” B) “Higher anesthetic doses are needed for patient-controlled epidural analgesia.” C) “A pudendal nerve block is highly effective for pain relief in the first stage of labor.” D) “Local infiltration using lidocaine is an appropriate method for controlling contraction pain.” 24. A nurse is completing the assessment of a woman admitted to the labor and birth suite. Which of the following would the nurse expect to include as part of the physical assessment? (Select all that apply.) A) Current pregnancy history B) Fundal height measurement C) Support system D) Estimated date of birth E) Membrane status F) Contraction pattern 25. A pregnant woman admitted to the labor and birth suite undergoes rapid HIV testing and is found to be HIV-positive. Which of the following would the nurse expect to include when developing a plan of care for this women? (Select all that apply.) A) Administration of penicillin G at the onset of labor B) Avoidance of scalp electrodes for fetal monitoring C) Refraining from obtaining fetal scalp blood for pH testing D) Adminstering zidovudine at the onset of labor. E) Electing for the use of forceps-assisted delivery 26. Which position would be most appropriate for the nurse to suggest as a comfort measure to a woman who is in the first stage of labor? (Select all that apply.) A) Walking with partner support B) Straddling with forward leaning over a chair C) Closed knee–chest position D) Rocking back and forth with foot on chair E) Supine with legs raised at a 90-degree angle 27. Which of the following would be most appropriate for the nurse to suggest about pushing to a woman in the second stage of labor? A) “Lying flat with your head elevated on two pillows makes pushing easier.” B) “Choose whatever method you feel most comfortable with for pushing.” C) “ Let me help you decide when it is time to start pushing.” D) “Bear down like you’re having a bowel movement with every contraction.” 28. A nurse is assessing a woman after birth and notes a second-degree laceration. The nurse interprets this as indicating that the tear extends through which of the following? A) Skin B) Muscles of perineal body C) Anal sphincter D) Anterior rectal wall 29. A nurse is assisting with the delivery of a newborn. The fetal head has just emerged. Which of the following would be done next? A) Suctioning of the mouth and nose B) Clamping of the umbilical cord C) Checking for the cord around the neck D) Drying of the newborn 30. A nurse is providing care to a woman during the third stage of labor. Which of the following would alert the nurse that the placenta is separating? (Select all that apply.) A) Boggy, soft uterus B) Uterus becoming discoid shaped C) Sudden gush of dark blood from the vagina D) Shortening of the umbilical cord Answer Key 1. C 2. A 3. A 4. A 5. C 6. A 7. D 8. B 9. C 10. D 11. A 12. C 13. B 14. C 15. A 16. C 17. A 18. B 19. A, B, C 20. B 21. C 22. D 23. A 24. B, E, F 25. B, C, D 26. A, B, D 27. B 28. B 29. C 30. C Chapter 15- Postpartum Adaptations 1. A primipara client gave birth vaginally to a healthy newborn girl 48 hours ago. The nurse palpates the client’s fundus, expecting it to be at which location? A) Two fingerbreadths above the umbilicus B) At the level of the umbilicus C) Two fingerbreadths below the umbilicus D) Four fingerbreadths below the umbilicus 2. When caring for a mother who has had a cesarean birth, the nurse would expect the client’s lochia to be: A) Greater than after a vaginal delivery B) About the same as after a vaginal delivery C) Less than after a vaginal delivery D) Saturated with clots and mucus 3. The nurse is developing a teaching plan for a client who has decided to bottle feed her newborn. Which of the following would the nurse include in the teaching plan to facilitate suppression of lactation? A) Encouraging the woman to manually express milk B) Suggesting that she take frequent warm showers to soothe her breasts C) Telling her to limit the amount of fluids that she drinks D) Instructing her to apply ice packs to both breasts every other hour 4. The nurse is making a follow-up home visit to a woman who is 12 days postpartum. Which of the following would the nurse expect to find when assessing the client’s fundus? A) Cannot be palpated B) 2 cm below the umbilicus C) 6 cm below the umbilicus D) 10 cm below the umbilicus 5. A client who is breast-feeding her newborn tells the nurse, “I notice that when I feed him, I feel fairly strong contractionlike pain. Labor is over. Why am I having contractions now?” Which response by the nurse would be most appropriate? A) “Your uterus is still shrinking in size; that’s why you’re feeling this pain.” B) “Let me check your vaginal discharge just to make sure everything is fine.” C) “Your body is responding to the events of labor, just like after a tough workout.” D) “The baby’s sucking releases a hormone that causes the uterus to contract.” 6. When the nurse is assessing a postpartum client approximately 6 hours after delivery, which finding would warrant further investigation? A) Deep red, fleshy-smelling lochia B) Voiding of 350 cc C) Heart rate of 120 beats/minute D) Profuse sweating 7. A postpartum client who is bottle feeding her newborn asks, “When should my period return?” Which response by the nurse would be most appropriate? A) “It’s difficult to say, but it will probably return in about 2 to 3 weeks.” B) “It varies, but you can estimate it returning in about 7 to 9 weeks.” C) “You won’t have to worry about it returning for at least 3 months.” D) “You don’t have to worry about that now. It’ll be quite a while.” 8. The nurse interprets which of the following as evidence that a client is in the taking-in phase? A) Client states, “He has my eyes and nose.” B) Client shows interest in caring for the newborn. C) Client performs self-care independently. D) Client confidently cares for the newborn. 9. Which of the following would the nurse interpret as being least indicative of paternal engrossment? A) Demonstrating pleasure when touching or holding the newborn B) Identifying imperfections in the newborn’s appearance C) Being able to distinguish his newborn from others in the nursery D) Showing feelings of pride with the birth of the newborn 10. A postpartum client comes to the clinic for her 6-week postpartum checkup. When assessing the client’s cervix, the nurse would expect the external cervical os to appear: A) Shapeless B) Circular C) Triangular D) Slit-like 11. The nurse develops a teaching plan for a postpartum client and includes teaching about how to perform Kegel exercises. The nurse includes this information for which reason? A) Reduce lochia B) Promote uterine involution C) Improve pelvic floor tone D) Alleviate perineal pain 12. A father of a newborn tells the nurse, “I may not know everything about being a dad, but I’m going to do the best I can for my son.” The nurse interprets this as indicating the father is in which stage of adaptation? A) Expectations B) Transition to mastery C) Reality D) Taking-in 13. A postpartum client is experiencing subinvolution. When reviewing the woman’s labor and birth history, which of the following would the nurse identify as being least significant to this condition? A) Early ambulation B) Prolonged labor C) Large fetus D) Use of anesthetics 14. Which of the following would lead the nurse to suspect that a postpartum woman is experiencing a problem? A) Elevated white blood cell count B) Acute decrease in hematocrit C) Increased levels of clotting factors D) Pulse rate of 60 beats/minute 15. A woman who gave birth 24 hours ago tells the nurse, “I’ve been urinating so much over the past several hours.” Which response by the nurse would be most appropriate? A) “You must have an infection, so let me get a urine specimen.” B) “Your body is undergoing many changes that cause your bladder to fill quickly.” C) “Your uterus is not contracting as quickly as it should.” D) “The anesthesia that you received is wearing off and your bladder is working again.” 16. A group of students are reviewing the process of breast milk production. The students demonstrate understanding when they identify which hormone as responsible for milk let-down? A) Prolactin B) Estrogen C) Progesterone D) Oxytocin 17. A nurse is making a home visit to a postpartum woman who delivered a healthy newborn 4 days ago. The woman’s breasts are swollen, hard, and tender to the touch. The nurse documents this finding as which of the following? A) Involution B) Engorgement C) Mastitis D) Engrossment 18. A nurse is assessing a postpartum woman’s adjustment to her maternal role. Which of the following would the nurse expect to occur first? A) Reestablishing relationships with others B) Demonstrating increasing confidence in care of the newborn C) Assuming a passive role in meeting her own needs D) Becoming preoccupied with the present 19. The partner of a woman who has given birth to a healthy newborn says to the nurse, “I want to be involved, but I’m not sure that I’m able to care for such a little baby.” The nurse interprets this as indicating which of the following stages? A) Expectations B) Reality C) Transition to mastery D) Taking-hold 20. A group of nursing students are reviewing information about maternal and paternal adaptations to the birth of a newborn. The nurse observes the parents interacting with their newborn physically and emotionally. The nurse documents this as which of the following? A) Puerperium B) Lactation C) Attachment D) Engrossment 21. After teaching a group of nursing students about the process of involution, the instructor determines that additional teaching is needed when the students identify which of the following as being involved? A) Catabolism B) Muscle fiber contraction C) Epithelial regeneration D) Vasodilation 22. A nurse is visiting a postpartum woman who delivered a healthy newborn 5 days ago. Which of the following would the nurse expect to find? A) Bright red discharge B) Pinkish brown discharge C) Deep red mucus-like discharge D) Creamy white discharge 23. A nurse teaches a postpartum woman about her risk for thromboembolism. Which of the following would the nurse be least likely to include as a factor increasing her risk? A) Increased clotting factors B) Vessel damage C) Immobility D) Increased red blood cell production 24. A nursing student is preparing a class presentation about changes in the various body systems during the postpartum period and their effects. Which of the following would the student include as influencing a postpartum woman’s ability to void? (Select all that apply.) A) Use of an opioid anesthetic during labor B) Generalized swelling of the perineum C) Decreased bladder tone from regional anesthesia D) Use of oxytocin to augment labor E) Need for an episiotomy 25. A postpartum woman who has experienced diastasis recti asks the nurse about what to expect related to this condition. Which response by the nurse would be most appropriate? A) “You’ll notice that this will fade to silvery lines.” B) “Exercise will help to improve the muscles.” C) “Expect the color to lighten somewhat.” D) “You’ll notice that your shoe size will increase.” 26. A group of nursing students are reviewing respiratory system adaptations that occur during the postpartum period. The students demonstrate understanding of the information when they identify which of the following as a postpartum adaptation? A) Continued shortness of breath B) Relief of rib aching C) Diaphragmatic elevation D) Decrease in respiratory rate 27. A woman who delivered a healthy newborn several hours ago asks the nurse, “Why am I perspiring so much?” The nurse integrates knowledge that a decrease in which hormone plays a role in this occurrence? A) Estrogen B) hCG C) hPL D) Progesterone   Answer Key 1. C 2. C 3. D 4. A 5. D 6. C 7. B 8. A 9. B 10. D 11. C 12. B 13. A 14. B 15. B 16. D 17. B 18. C 19. B 20. C 21. D 22. B 23. D 24. B, C, D 25. B 26. B 27. A Chapter 16- Nursing Management During the Postpartum Period 1. A woman who is 12 hours postpartum had a pulse rate around 80 beats per minute during pregnancy. Now, the nurse finds a pulse of 60 beats per minute. Which of these actions should the nurse take? A) Document the finding, as it is a normal finding at this time. B) Contact the physician, as it indicates early DIC. C) Contact the physician, as it is a first sign of postpartum eclampsia. D) Obtain an order for a CBC, as it suggests postpartum anemia. 2. To decrease the pain associated with an episiotomy immediately after birth, which action by the nurse would be most appropriate? A) Offer warm blankets. B) Encourage the woman to void. C) Apply an ice pack to the site. D) Offer a warm sitz bath. 3. A postpartum client has a fourth-degree perineal laceration. The nurse would expect which of the following medications to be ordered? A) Ferrous sulfate (Feosol) B) Methylergonovine (Methergine) C) Docusate (Colace) D) Bromocriptine (Parlodel) 4. Which statement would alert the nurse to the potential for impaired bonding between mother and newborn? A) “You have your daddy’s eyes.” B) “He looks like a frog to me.” C) “Where did you get all that hair?” D) “He seems to sleep a lot.” 5. After a normal labor and birth, a client is discharged from the hospital 12 hours later. When the community health nurse makes a home visit 2 days later, which finding would alert the nurse to the need for further intervention? A) Presence of lochia serosa B) Frequent scant voidings C) Fundus firm, below umbilicus D) Milk filling in both breasts 6. A primipara client who is bottle feeding her baby begins to experience breast engorgement on her third postpartum day. Which instruction would be most appropriate to aid in relieving her discomfort? A) “Express some milk from your breasts every so often to relieve the distention.” B) “Remove your bra to relieve the pressure on your sensitive nipples and breasts.” C) “Apply ice packs to your breasts to reduce the amount of milk being produced.” D) “Take several warm showers daily to stimulate the milk let-down reflex.” 7. The nurse administers RhoGAM to an Rh-negative client after delivery of an Rh-positive newborn based on the understanding that this drug will prevent her from: A) Becoming Rh positive B) Developing Rh sensitivity C) Developing AB antigens in her blood D) Becoming pregnant with an Rh-positive fetus 8. Which of the following factors in a client’s history would alert the nurse to an increased risk for postpartum hemorrhage? A) Multiparity, age of mother, operative delivery B) Size of placenta, small baby, operative delivery C) Uterine atony, placenta previa, operative procedures D) Prematurity, infection, length of labor 9. After teaching parents about their newborn, the nurse determines that the teaching was successful when they identify the development of a close emotional attraction to a newborn by parents during the first 30 to 60 minutes after birth as which of the following? A) Reciprocity B) Engrossment C) Bonding D) Attachment 10. A nurse is working as part of a committee to establish policies to promote bonding and attachment. Which practice would be least effective in achieving this goal? A) Allowing unlimited visiting hours on maternity units B) Offering round-the-clock nursery care for all infants C) Promoting rooming-in D) Encouraging infant contact immediately after birth 11. When developing the plan of care for the parents of a newborn, the nurse identifies interventions to promote bonding and attachment based on the rationale that bonding and attachment are most supported by which measure? A) Early parent–infant contact following birth B) Expert medical care for the labor and birth C) Good nutrition and prenatal care during pregnancy D) Grandparent involvement in infant care after birth 12. A postpartum woman is having difficulty voiding for the first time after giving birth. Which of the following would be least effective in helping to stimulate voiding? A) Pouring warm water over her perineal area B) Having her hear the sound of water running nearby C) Placing her hand in a basin of cool water D) Standing her in the shower with the warm water on 13. The nurse is assisting a postpartum woman out of bed to the bathroom for a sitz bath. Which of the following would be a priority? A) Placing the call light within her reach B) Teaching her how the sitz bath works C) Telling her to use the sitz bath for 30 minutes D) Cleaning the perineum with the peri-bottle 14. A nurse is reviewing the medical record of a postpartum client. The nurse identifies that the woman is at risk for a postpartum infection based on which of the following? (Select all that apply.) A) History of diabetes B) Labor of 12 hours C) Rupture of membranes for 16 hours D) Hemoglobin level 10 mg/dL E) Placenta requiring manual extraction 15. A nurse is completing a postpartum assessment. Which finding would alert the nurse to a potential problem? A) Lochia rubra with a fleshy odor B) Respiratory rate of 16 breaths per minute C) Temperature of 101° F D) Pain rating of 2 on a scale from 0 to 10 16. The nurse is assessing a postpartum client’s lochia and finds that there is about a 4-inch stain on the perineal pad. The nurse documents this finding as which of the following? A) Scant B) Light C) Moderate D) Large 17. When reviewing the medical record of a postpartum client, the nurse notes that the client has experienced a third-degree laceration. The nurse understands that the laceration extends to which of the following? A) Superficial structures above the muscle B) Through the perineal muscles C) Through the anal sphincter muscle D) Through the anterior rectal wall 18. A nurse is observing a postpartum client interacting with her newborn and notes that the mother is engaging with the newborn in the en face position. Which of the following would the nurse be observing? A) Mother placing the newborn next to bare breast. B) Mother making eye-to-eye contact with the newborn C) Mother gently stroking the newborn’s face D) Mother holding the newborn upright at the shoulder 19. After teaching a group of students about risk factors associated with postpartum hemorrhage, the instructor determines that the teaching was successful when the students identify which of the following as a risk factor? (Select all that apply.) A) Prolonged labor B) Placenta previa C) Null parity D) Hydramnios E) Labor augmentation 20. A postpartum woman who is breast-feeding tells the nurse that she is experiencing nipple pain. Which of the following would be least appropriate for the nurse to suggest? A) Use of a mild analgesic about 1 hour before breast-feeding B) Application of expressed breast milk to the nipples C) Application of glycerin-based gel to the nipples D) Reinstruction about proper latching-on technique 21. A nurse is developing a teaching plan for a postpartum woman who is breast-feeding about sexuality and contraception. Which of the following would the nurse most likely include? (Select all that apply.) A) Resumption of sexual intercourse about two weeks after delivery B) Possible experience of fluctuations in sexual interest C) Use of a water-based lubricant to ease vaginal discomfort D) Use of combined hormonal contraceptives for the first three weeks E) Possibility of increased breast sensitivity during sexual activity 22. After teaching a postpartum woman about breast-feeding, the nurse determines that the teaching was successful when the woman states which of the following? A) “I should notice a decrease in abdominal cramping during breast-feeding.” B) “I should wash my hands before starting to breast-feed.” C) “The baby can be awake or sleepy when I start to feed him.” D) “The baby’s mouth will open up once I put him to my breast.” 23. A postpartum woman who is bottle-feeding her newborn asks the nurse, “About how much should my newborn drink at each feeding?” The nurse responds by saying that to feel satisfied, the newborn needs which amount at each feeding? A) 1 to 2 ounces B) 2 to 4 ounces C) 4 to 6 ounces D) 6 to 8 ounces 24. A nurse is observing a postpartum woman and her partner interact with the their newborn. The nurse determines that the parents are developing parental attachment with their newborn when they demonstrate which of the following? (Select all that apply.) A) Frequently ask for the newborn to be taken from the room B) Identify common features between themselves and the newborn C) Refer to the newborn as having a monkey-face D) Make direct eye contact with the newborn E) Refrain from checking out the newborn’s features 25. After reviewing information about postpartum blues, a group of students demonstrate understanding when they state which of the following about this condition? A) “Postpartum blues is a long-term emotional disturbance.” B) “Sleep usually helps to resolve the blues.” C) “The mother loses contact with reality.” D) “Extended psychotherapy is needed for treatment.”   Answer Key 1. A 2. C 3. C 4. B 5. B 6. C 7. B 8. C 9. C 10. B 11. A 12. C 13. A 14. A, D, E 15. C 16. B 17. C 18. B 19. B, D, E 20. A 21. B, C, E 22. B 23. B 24. B, D 25. B Chapter 17- Newborn Transitioning 1. When explaining how a newborn adapts to extrauterine life, the nurse would describe which body systems as undergoing the most rapid changes? A) Gastrointestinal and hepatic B) Urinary and hematologic C) Respiratory and cardiovascular D) Neurological and integumentary 2. A new mother reports that her newborn often spits up after feeding. Assessment reveals regurgitation. The nurse responds integrating understanding that this most likely is due to which of the following? A) Placing the newborn prone after feeding B) Limited ability of digestive enzymes C) Underdeveloped pyloric sphincter D) Relaxed cardiac sphincter 3. After teaching a class about hepatic system adaptations after birth, the instructor determines that the teaching was successful when the class identifies which of the following as the process of changing bilirubin from a fat-soluble product to a water-soluble product? A) Hemolysis B) Conjugation C) Jaundice D) Hyperbilirubinemia 4. Twenty minutes after birth, a baby begins to move his head from side to side, making eye contact with the mother, and pushes his tongue out several times. The nurse interprets this as indicating which of the following? A) A good time to initiate breast-feeding B) The period of decreased responsiveness preceding sleep C) The need to be alert for gagging and vomiting D) Evidence that the newborn is becoming chilled 5. The nurse institutes measure to maintain thermoregulation based on the understanding that newborns have limited ability to regulate body temperature because they: A) Have a smaller body surface compared to body mass B) Lose more body heat when they sweat than adults C) Have an abundant amount of subcutaneous fat all over D) Are unable to shiver effectively to increase heat production 6. A new mother is changing the diaper of her 20-hour-old newborn and asks why the stool is almost black. Which response by the nurse would be most appropriate? A) “You probably took iron during your pregnancy.” B) “This is meconium stool, normal for a newborn.” C) “I’ll take a sample and check it for possible bleeding.” D) “This is unusual and I need to report this.” 7. A client expresses concern that her 2-hour-old newborn is sleepy and difficult to awaken. The nurse explains that this behavior indicates which of the following? A) Normal progression of behavior B) Probable hypoglycemia C) Physiological abnormality D) Inadequate oxygenation 8. After the birth of a newborn, which of the following would the nurse do first to assist in thermoregulation? A) Dry the newborn thoroughly. B) Put a hat on the newborn’s head. C) Check the newborn’s temperature. D) Wrap the newborn in a blanket. 9. Assessment of a newborn reveals rhythmic spontaneous movements. The nurse interprets this as indicating: A) Habituation B) Motor maturity C) Orientation D) Social behaviors 10. After teaching new parents about the sensory capabilities of their newborn, the nurse determines that the teaching was successful when they identify which sense as being the least mature? A) Hearing B) Touch C) Taste D) Vision 11. The nurse places a warmed blanket on the scale when weighing a newborn. The nurse does so to minimize heat loss via which mechanism? A) Evaporation B) Conduction C) Convection D) Radiation 12. Which of the following would alert the nurse to the possibility of respiratory distress in a newborn? A) Symmetrical chest movements B) Periodic breathing C) Respirations of 40 breaths/minute D) Sternal retractions 13. A nurse is counseling a mother about the immunologic properties of breast milk. The nurse integrates knowledge of immunoglobulins, emphasizing that breast milk is a major source of which immunoglobulin? A) IgA B) IgG C) IgM D) IgE 14. The nurse is teaching a group of students about the similarities and differences between newborn skin and adult skin. Which statement by the group indicates that additional teaching is needed? A) The newborn’s skin and that of an adult are similar in thickness. B) The lipid composition of the skin of a newborn and adult is about the same. C) Skin development in the newborn is complete at birth. D) The newborn has more fibrils connecting the dermis and epidermis. 15. A nurse is developing a teaching plan for the parents of a newborn. When describing the neurologic development of a newborn to his parents, the nurse would explain that the development occurs in which fashion? A) Head-to-toe B) Lateral-to-medial C) Outward-to-inward D) Distal-to-caudal 16. The nurse is assessing the respirations of several newborns. The nurse would notify the health care provider for the newborn with which respiratory rate at rest? A) 38 breaths per minute B) 46 breaths per minute C) 54 breaths per minute D) 68 breaths per minute 17. A new mother asks the nurse, “Why has my baby lost weight since he was born?” The nurse integrates knowledge of which of the following when responding to the new mother? A) Insufficient calorie intake B) Shift of water from extracellular space to intracellular space C) Increase in stool passage D) Overproduction of bilirubin 18. The nurse observes the stool of a newborn who has begun to breast-feed. Which of the following would the nurse expect to find? A) Greenish black, tarry stool B) Yellowish-brown, seedy stool C) Yellow-gold, stringy stool D) Yellowish-green, pasty stool 19. A nurse is assessing a newborn who is about 4½ hours old. The nurse would expect this newborn to exhibit which of the following? (Select all that apply.) A) Sleeping B) Interest in environmental stimuli C) Passage of meconium D) Difficulty arousing the newborn E) Spontaneous Moro reflexes 20. A nurse is assessing a newborn and observes the newborn moving his head and eyes toward a loud sound. The nurse interprets this as which of the following? A) Habituation B) Motor maturity C) Social behavior D) Orientation 21. A newborn is experiencing cold stress. Which of the following would the nurse expect to assess? (Select all that apply.) A) Respiratory distress B) Decreased oxygen needs C) Hypoglycemia D) Metabolic alkalosis E) Jaundice 22. A group of nursing students are reviewing the changes in the newborn’s lungs that must occur to maintain respiratory function. The students demonstrate understanding of this information when they identify which of the following as the first event? A) Expansion of the lungs B) Increased pulmonary blood flow C) Initiation of respiratory movement D) Redistribution of cardiac output 23. A nurse is reviewing the laboratory test results of a newborn. Which result would the nurse identify as a cause for concern? A) Hemoglobin 19 g/dL B) Platelets 75,000/uL C) White blood cells 20,000/mm3 D) Hematocrit 52% 24. A nursing instructor is preparing a class on newborn adaptations. When describing the change from fetal to newborn circulation, which of the following would the instructor most likely include? (Select all that apply.) A) Decrease in right atrial pressure leads to closure of the foramen ovale. B) Increase in oxygen levels leads to a decrease in systemic vascular resistance. C) Onset of respirations leads to a decrease in pulmonary vascular resistance. D) Increase in pressure in the left atrium results from increases in pulmonary blood flow. E) Closure of the ductus venosus eventually forces closure of the ductus arteriosus. 25. A nursing student is preparing a presentation on minimizing heat loss in the newborn. Which of the following would the student include as a measure to prevent heat loss through convection? A) Placing a cap on a newborn’s head B) Working inside an isolette as much as possible. C) Placing the newborn skin-to-skin with the mother D) Using a radiant warmer to transport a newborn 26. After teaching a group of nursing students about a neutral thermal environment, the instructor determines that the teaching was successful when the students identify which of the following as the newborn’s primary method of heat production? A) Convection B) Nonshivering thermogenesis C) Cold stress D) Bilirubin conjugation 27. While observing the interaction between a newborn and his mother, the nurse notes the newborn nestling into the arms of his mother. The nurse identifies this behavior as which of the following? A) Habituation B) Self-quieting ability C) Social behaviors D) Orientation   Answer Key 1. C 2. D 3. B 4. A 5. D 6. B 7. A 8. A 9. B 10. D 11. B 12. D 13. A 14. C 15. A 16. D 17. A 18. B 19. B, C 20. D 21. A, C, E 22. C 23. B 24. A, C, D, E 25. B 26. B 27. C Chapter 18- Nursing Management of the Newborn 1. Prior to discharging a 24-hour-old newborn, the nurse assesses her respiratory status. Which of the following would the nurse expect to assess? A) Respiratory rate 45, irregular B) Costal breathing pattern C) Nasal flaring, rate 65 D) Crackles on auscultation 2. The nurse encourages the mother of a healthy newborn to put the newborn to the breast immediately after birth for which reason? A) To aid in maturing the newborn’s sucking reflex B) To encourage the development of maternal antibodies C) To facilitate maternal–infant bonding D) To enhance the clearing of the newborn’s respiratory passages 3. When making a home visit, the nurse observes a newborn sleeping on his back in a bassinet. In one corner of the bassinet is a soft stuffed animal and at the other end is a bulb syringe. The nurse determines that the mother needs additional teaching because of which of the following? A) The newborn should not be sleeping on his back. B) Stuffed animals should not be in areas where infants sleep. C) The bulb syringe should not be kept in the bassinet. D) This newborn should be sleeping in a crib. 4. Assessment of a newborn reveals a heart rate of 180 beats/minute. To determine whether this finding is a common variation rather than a sign of distress, what else does the nurse need to know? A) How many hours old is this newborn? B) How long ago did this newborn eat? C) What was the newborn’s birth weight? D) Is acrocyanosis present? 5. Just after delivery, a newborn’s axillary temperature is 94° C. What action would be most appropriate? A) Assess the newborn’s gestational age. B) Rewarm the newborn gradually. C) Observe the newborn every hour. D) Notify the physician if the temperature goes lower. 6. The parents of a newborn become concerned when they notice that their baby seems to stop breathing for a few seconds. After confirming the parents’ findings by observing the newborn, which of the following actions would be most appropriate? A) Notify the health care provider immediately. B) Assess the newborn for signs of respiratory distress. C) Reassure the parents that this is an expected pattern. D) Tell the parents not to worry since his color is fine. 7. When assessing a newborn 1 hour after birth, the nurse measures an axillary temperature of 95.8° F, an apical pulse of 114 beats/minute, and a respiratory rate of 60 breaths/minute. Which nursing diagnosis takes highest priority? A) Hypothermia related to heat loss during birthing process B) Impaired parenting related to addition of new family member C) Risk for deficient fluid volume related to insensible fluid loss D) Risk for infection related to transition to extrauterine environment 8. The nurse places a newborn with jaundice under the phototherapy lights in the nursery to achieve which goal? A) Prevent cold stress B) Increase surfactant levels in the lungs C) Promote respiratory stability D) Decrease the serum bilirubin level 9. The nurse completes the initial assessment of a newborn. Which finding would lead the nurse to suspect that the newborn is experiencing difficulty with oxygenation? A) Respiratory rate of 54 breaths/minute B) Abdominal breathing C) Nasal flaring D) Acrocyanosis 10. During a physical assessment of a newborn, the nurse observes bluish markings across the newborn’s lower back. The nurse documents this finding as which of the following? A) Milia B) Mongolian spots C) Stork bites D) Birth trauma 11. While making rounds in the nursery, the nurse sees a 6-hour-old baby girl gagging and turning bluish. Which of the following would the nurse do first? A) Alert the physician stat and turn the newborn to her right side. B) Administer oxygen via facial mask by positive pressure. C) Lower the newborn’s head to stimulate crying. D) Aspirate the oral and nasal pharynx with a bulb syringe. 12. While performing a physical assessment of a newborn boy, the nurse notes diffuse edema of the soft tissues of his scalp that crosses suture lines. The nurse documents this finding as: A) Molding B) Microcephaly C) Caput succedaneum D) Cephalhematoma 13. Assessment of a newborn reveals uneven gluteal (buttocks. skin creases and a “clunk” when Ortolani’s maneuver is performed. Which of the following would the nurse suspect? A) Slipping of the periosteal joint B) Developmental hip dysplasia C) Normal newborn variation D) Overriding of the pelvic bone 14. The nurse strokes the lateral sole of the newborn’s foot from the heel to the ball of the foot when evaluating which reflex? A) Babinski B) Tonic neck C) Stepping D) Plantar grasp 15. The nurse administers vitamin K intramuscularly to the newborn based on which of the following rationales? A) Stop Rh sensitization B) Increase erythopoiesis C) Enhance bilirubin breakdown D) Promote blood clotting 16. The nurse is assessing the skin of a newborn and notes a rash on the newborn’s face, and chest. The rash consists of small papules and is scattered with no pattern. The nurse interprets this finding as which of the following? A) Harlequin sign B) Nevus flammeus C) Erythema toxicum D) Port wine stain 17. After teaching a group of nursing students about variations in newborn head size and appearance, the instructor determines that the teaching was successful when the students identify which of the following as a normal variation? (Select all that apply) A) Cephalhematoma B) Molding C) Closed fontanels D) Caput succedaneum E) Posterior fontanel diameter 1.5 cm 18. The nurse is assessing a newborn’s eyes. Which of the following would the nurse identify as normal? (Select all that apply.) A) Slow blink response B) Able to track object to midline C) Transient deviation of the eyes D) Involuntary repetitive eye movement E) Absent red reflex 19. Assessment of a newborn’s head circumference reveals that it is 34 cm. The nurse would suspect that this newborn’s chest circumference would be: A) 30 cm B) 32 cm C) 34 cm D) 36 cm 20. The nurse is auscultating a newborn’s heart and places the stethoscope at the point of maximal impulse at which location? A) Just superior to the nipple, at the midsternum B) Lateral to the midclavicular line at the fourth intercostal space C) At the fifth intercostal space to the left of the sternum D) Directly adjacent to the sternum at the second intercostals space 21. The nurse is inspecting the external genitalia of a male newborn. Which of the following would alert the nurse to a possible problem? A) Limited rugae B) Large scrotum C) Palpable testes in scrotal sac D) Absence of engorgement 22. When assessing a newborn’s reflexes, the nurse strokes the newborn’s cheek and the newborn turns toward the side that was stroked and begins sucking. The nurse documents which reflex as being positive? A) Palmar grasp reflex B) Tonic neck reflex C) Moro reflex D) Rooting reflex 23. A nurse is teaching new parents about bathing their newborn. The nurse determines that the teaching was successful when the parents state which of the following? A) “We can put a tiny bit of lotion on his skin and then rub it in gently.” B) “We should avoid using any kind of baby powder.” C) “We need to bathe him at least four to five times a week.” D) “We should clean his eyes after washing his face and hair.” 24. A new mother who is breast-feeding her newborn asks the nurse, “How will I know if my baby is drinking enough?” Which response by the nurse would be most appropriate? A) “If he seems content after feeding, that should be a sign.” B) “Make sure he drinks at least 5 minutes on each breast.” C) “He should wet between 6 to 12 diapers each day.” D) “If his lips are moist, then he’s okay.” 25. A nurse is teaching postpartum client and her partner about caring for their newborn’s umbilical cord site. Which statement by the parents indicates a need for additional teaching? A) “We can put him in the tub to bathe him once the cord falls off and is healed.” B) “The cord stump should change from brown to yellow.” C) “Exposing the stump to the air helps it to dry.” D) “We need to call the doctor if we notice a funny odor.” 26. While changing a female newborn’s diaper, the nurse observes a mucus-like, slightly bloody vaginal discharge. Which of the following would the nurse do next? A) Document this as pseudomenstruation B) Notify the practitioner immediately C) Obtain a culture of the discharge D) Inspect for engorgement 27. A nursing instructor is describing the advantages and disadvantages associated with newborn circumcision to a group of nursing students. Which statement by the students indicates effective teaching? A) “Sexually transmitted infections are more common in circumcised males.” B) “The rate of penile cancer is less for circumcised males.” C) “Urinary tract infections are more easily treated in circumcised males.” D) “Circumcision is a risk factor for acquiring HIV infection.” 28. A newborn is scheduled to undergo a screening test for phenylketonuria (PKU). The nurse prepares to obtain the blood sample from the newborn’s: A) Finger B) Heel C) Scalp vein D) Umbilical vein 29. Assessment of a newborn reveals transient tachypnea. The nurse reviews the newborn’s medical record. Which of the following would the nurse be least likely to identify as a risk factor for this condition? A) Cesarean birth B) Shortened labor C) Central nervous system depressant during labor D) Maternal asthma 30. A nurse is providing teaching to a new mother about her newborn’s nutritional needs. Which of the following would the nurse be most likely to include in the teaching? (Select all that apply.) A) Supplementing with iron if the woman is breast-feeding B) Providing supplemental water intake with feedings C) Feeding the newborn every 2 to 4 hours during the day D) Burping the newborns frequently throughout each feeding E) Using feeding time for promoting closeness Answer Key 1. A 2. C 3. B 4. A 5. B 6. B 7. A 8. D 9. C 10. B 11. D 12. C 13. B 14. A 15. D 16. C 17. A, B, D 18. B, C, D 19. B 20. B 21. A 22. D 23. B 24. C 25. B 26. A 27. B 28. B 29. B 30. C, D, E Chapter 19- Nursing Management of Pregnancy at Risk- Pregnancy 1. After teaching a woman who has had an evacuation for a hydatidiform mole (molar pregnancy. about her condition, which of the following statements indicates that the nurse’s teaching was successful? A) “I will be sure to avoid getting pregnant for at least 1 year.” B) “My intake of iron will have to be closely monitored for 6 months.” C) “My blood pressure will continue to be increased for about 6 more months.” D) “I won’t use my birth control pills for at least a year or two.” 2. Which of the following findings on a prenatal visit at 10 weeks might lead the nurse to suspect a hydatidiform mole? A) Complaint of frequent mild nausea B) Blood pressure of 120/84 mm Hg C) History of bright red spotting 6 weeks ago D) Fundal height measurement of 18 cm 3. A client is diagnosed with gestational hypertension and is receiving magnesium sulfate. Which finding would the nurse interpret as indicating a therapeutic level of medication? A) Urinary output of 20 mL per hour B) Respiratory rate of 10 breaths/minute C) Deep tendons reflexes 2+ D) Difficulty in arousing 4. Upon entering the room of a client who has had a spontaneous abortion, the nurse observes the client crying. Which of the following responses by the nurse would be most appropriate? A) “Why are you crying?” B) “Will a pill help your pain?” C) “I’m sorry you lost your baby.” D) “A baby still wasn’t formed in your uterus.” 5. Which of the following data on a client’s health history would the nurse identify as contributing to the client’s risk for an ectopic pregnancy? A) Use of oral contraceptives for 5 years B) Ovarian cyst 2 years ago C) Recurrent pelvic infections D) Heavy, irregular menses 6. In a woman who is suspected of having a ruptured ectopic pregnancy, the nurse would expect to assess for which of the following as a priority? A) Hemorrhage B) Jaundice C) Edema D) Infection 7. Which of the following findings would the nurse interpret as suggesting a diagnosis of gestational trophoblastic disease? A) Elevated hCG levels, enlarged abdomen, quickening B) Vaginal bleeding, absence of FHR, decreased hPL levels C) Visible fetal skeleton on ultrasound, absence of quickening, enlarged abdomen D) Gestational hypertension, hyperemesis gravidarum, absence of FHR 8. It is determined that a client’s blood Rh is negative and her partner’s is positive. To help prevent Rh isoimmunization, the nurse anticipates that the client will receive RhoGAM at which time? A) At 34 weeks’ gestation and immediately before discharge B) 24 hours before delivery and 24 hours after delivery C) In the first trimester and within 2 hours of delivery D) At 28 weeks’ gestation and again within 72 hours after delivery 9. The nurse is developing a plan of care for a woman who is pregnant with twins. The nurse includes interventions focusing on which of the following because of the woman’s increased risk? A) Oligohydramnios B) Preeclampsia C) Post-term labor D) Chorioamnionitis 10. A woman hospitalized with severe preeclampsia is being treated with hydralazine to control blood pressure. Which of the following would the lead the nurse to suspect that the client is having an adverse effect associated with this drug? A) Gastrointestinal bleeding B) Blurred vision C) Tachycardia D) Sweating 11. After reviewing a client’s history, which factor would the nurse identify as placing her at risk for gestational hypertension? A) Mother had gestational hypertension during pregnancy. B) Client has a twin sister. C) Sister-in-law had gestational hypertension. D) This is the client’s second pregnancy. 12. A client with hyperemesis gravidarum is admitted to the facility after being cared for at home without success. Which of the following would the nurse expect to include in the client’s plan of care? A) Clear liquid diet B) Total parenteral nutrition C) Nothing by mouth D) Administration of labetalol 13. The nurse is reviewing the laboratory test results of a pregnant client. Which one of the following findings would alert the nurse to the development of HELLP syndrome? A) Hyperglycemia B) Elevated platelet count C) Leukocytosis D) Elevated liver enzymes 14. Which of the following would the nurse have readily available for a client who is receiving magnesium sulfate to treat severe preeclampsia? A) Calcium gluconate B) Potassium chloride C) Ferrous sulfate D) Calcium carbonate 15. Which assessment finding would lead the nurse to suspect infection as the cause of a client’s PROM? A) Yellow-green fluid B) Blue color on Nitrazine testing C) Ferning D) Foul odor 16. While assessing a pregnant woman, the nurse suspects that the client may be at risk for hydramnios based on which of the following? (Select all that apply.) A) History of diabetes B) Complaints of shortness of breath C) Identifiable fetal parts on abdominal palpation D) Difficulty obtaining fetal heart rate E) Fundal height below that for expected gestataional age 17. After teaching a group of nursing students about the possible causes of spontaneous abortion, the instructor determines that the teaching was successful when the students identify which of the following as the most common cause of first trimester abortions? A) Maternal disease B) Cervical insufficiency C) Fetal genetic abnormalities D) Uterine fibroids 18. A pregnant woman is admitted with premature rupture of the membranes. The nurse is assessing the woman closely for possible infection. Which of the following would lead the nurse to suspect that the woman is developing an infection? (Select all that apply.) A) Fetal bradycardia B) Abdominal tenderness C) Elevated maternal pulse rate D) Decreased C-reactive protein levels E) Cloudy malodorous fluid 19. A nurse is teaching a pregnant woman with preterm premature rupture of membranes who is about to be discharged home about caring for herself. Which statement by the woman indicates a need for additional teaching? A) “I need to keep a close eye on how active my baby is each day.” B) “I need to call my doctor if my temperature increases.” C) “It’s okay for my husband and me to have sexual intercourse.” D) “I can shower but I shouldn’t take a tub bath.” 20. A nurse is assessing a pregnant woman with gestational hypertension. Which of the following would lead the nurse to suspect that the client has developed severe preeclampsia? A) Urine protein 300 mg/24 hours B) Blood pressure 150/96 mm Hg C) Mild facial edema D) Hyperreflexia 21. A nurse suspects that a pregnant client may be experiencing abruption placenta based on assessment of which of the following? (Select all that apply.) A) Dark red vaginal bleeding B) Insidious onset C) Absence of pain D) Rigid uterus E) Absent fetal heart tones 22. The health care provider orders PGE2 for a woman to help evacuate the uterus following a spontaneous abortion. Which of the following would be most important for the nurse to do? A) Use clean technique to administer the drug. B) Keep the gel cool until ready to use. C) Maintain the client for ½ hour after administration. D) Administer intramuscularly into the deltoid area. 23. A nursing student is reviewing an article about preterm premature rupture of membranes. Which of the following would the student expect to find as factor placing a woman at high risk for this condition? (Select all that apply.) A) High body mass index B) Urinary tract infection C) Low socioeconomic status D) Single gestations E) Smoking 24. A woman with placenta previa is being treated with expectant management. The woman and fetus are stable. The nurse is assessing the woman for possible discharge home. Which statement by the woman would suggest to the nurse that home care might be inappropriate? A) “My mother lives next door and can drive me here if necessary.” B) “I have a toddler and preschooler at home who need my attention.” C) “I know to call my health care provider right away if I start to bleed again.” D) “I realize the importance of following the instructions for my care.” 25. A woman with hyperemesis gravidarum asks the nurse about suggestions to minimize nausea and vomiting. Which suggestion would be most appropriate for the nurse to make? A) “Make sure that anything around your waist is quite snug.” B) “Try to eat three large meals a day with less snacking.” C) “Drink fluids in between meals rather than with meals.” D) “Lie down for about an hour after you eat” 26. A woman with gestational hypertension experiences a seizure. Which of the following would be the priority? A) Fluid replacement B) Oxygenation C) Control of hypertension D) Delivery of the fetus 27. A woman is receiving magnesium sulfate as part of her treatment for severe preeclampsia. The nurse is monitoring the woman’s serum magnesium levels. Which level would the nurse identify as therapeutic? A) 3.3 mEq/L B) 6.1 mEq/L C) 8.4 mEq/L D) 10.8 mEq/L Answer Key 1. A 2. D 3. C 4. C 5. C 6. A 7. D 8. D 9. B 10. C 11. A 12. C 13. D 14. A 15. D 16. A, B, D 17. C 18. B, C, E 19. C 20. D 21. A, D, E 22. C 23. B, C, E 24. B 25. C 26. B 27. B Chapter 20- Nursing Management of the Pregnancy at Risk 1. The nurse is teaching a pregnant woman with type 1 diabetes about her diet during pregnancy. Which client statement indicates that the nurse’s teaching was successful? A) “I’ll basically follow the same diet that I was following before I became pregnant.” B) “Because I need extra protein, I’ll have to increase my intake of milk and meat.” C) “Pregnancy affects insulin production, so I’ll need to make adjustments in my diet.” D) “I’ll adjust my diet and insulin based on the results of my urine tests for glucose.” 2. A nurse is developing a program for pregnant women with diabetes about reducing complications. Which factor would the nurse identify as being most important in helping to reduce the maternal/fetal/neonatal complications associated with pregnancy and diabetes? A) Stability of the woman’s emotional and psychological status B) Degree of glycemic control achieved during the pregnancy C) Evaluation of retinopathy by an ophthalmologist D) Blood urea nitrogen level (BUN. within normal limits 3. Because a pregnant client’s diabetes has been poorly controlled throughout her pregnancy, the nurse would be alert for which of the following in the neonate at birth? A) Macrosomia B) Hyperglycemia C) Low birth weight D) Hypobilirubinemia 4. A woman with diabetes is considering becoming pregnant. She asks the nurse whether she will be able to take oral hypoglycemics when she is pregnant. The nurse’s response is based on the understanding that oral hypoglycemics: A) Can be used as long as they control serum glucose levels B) Can be taken until the degeneration of the placenta occurs C) Are usually suggested primarily for women who develop gestational diabetes D) Show promising results but more studies are needed to confirm their effectiveness 5. A 10-week pregnant woman with diabetes has a glycosylated hemoglobin (HbA1C. level of 13%. At this time the nurse should be most concerned about which of the following possible fetal outcomes? A) Congenital anomalies B) Incompetent cervix C) Placenta previa D) Abruptio placentae 6. After teaching a group of students about the use of antiretroviral agents in pregnant women who are HIV-positive, the instructor determines that the teaching was successful when the group identifies which of the following as the underlying rationale? A) Reduction in viral loads in the blood B) Treatment of opportunistic infections C) Adjunct therapy to radiation and chemotherapy D) Can cure acute HIV/AIDS infections 7. Assessment of a pregnant woman and her fetus reveals tachycardia and hypertension. There is also evidence suggesting vasoconstriction. The nurse would question the woman about use of which substance? A) Marijuana B) Alcohol C) Heroin D) Cocaine 8. When teaching a class of pregnant women about the effects of substance abuse during pregnancy, which of the following would the nurse most likely include? A) Low-birth-weight infants B) Excessive weight gain C) Higher pain tolerance D) Longer gestational periods 9. A client who is HIV-positive is in her second trimester and remains asymptomatic. She voices concern about her newborn’s risk for the infection. Which of the following statements by the nurse would be most appropriate? A) “You’ll probably have a cesarean birth to prevent exposing your newborn.” B) “Antibodies cross the placenta and provide immunity to the newborn.” C) “Wait until after the infant is born and then something can be done.” D) “Antiretroviral medications are available to help reduce the risk of transmission.” 10. When assessing a pregnant woman with heart disease throughout the antepartal period, the nurse would be especially alert for signs and symptoms of cardiac decompensation at which time? A) 16 to 20 weeks’ gestation B) 20 to 24 weeks’ gestation C) 24 to 28 weeks’ gestation D) 28 to 32 weeks’ gestation 11. When preparing a schedule of follow-up visits for a pregnant woman with chronic hypertension, which of the following would be most appropriate? A) Monthly visits until 32 weeks, then bi-monthly visits B) Bi-monthly visits until 28 weeks, then weekly visits C) Monthly visits until 20 weeks, then bi-monthly visits D) Bi-monthly visits until 36 weeks, then weekly visits 12. Which medication would the nurse question if ordered to control a pregnant woman’s asthma? A) Budesonide B) Albuterol C) Salmeterol D) Oral prednisone 13. After teaching a pregnant woman with iron deficiency anemia about her prescribed iron supplement, which statement indicates successful teaching? A) “I should take my iron with milk.” B) “I should avoid drinking orange juice.” C) “I need to eat foods high in fiber.” D) “I’ll call the doctor if my stool is black and tarry.” 14. A nurse is providing care to several pregnant women at the clinic. The nurse would screen for group B streptococcus infection in a client at: A) 16 weeks’ gestation B) 28 week’ gestation C) 32 weeks’ gestation D) 36 weeks’ gestation 15. The nurse is assessing a newborn of a woman who is suspected of abusing alcohol. Which newborn finding would provide additional evidence to support this suspicion? A) Wide large eyes B) Thin upper lip C) Protruding jaw D) Elongated nose 16. After teaching a group of nursing students about the impact of pregnancy on the older woman, the instructor determines that the teaching was successful when the students state which of the following? A) “The majority of women who become pregnant over age 35 experience complications.” B) “Women over the age of 35 who become pregnant require a specialized type of assessment.” C) “Women over age 35 and are pregnant have an increased risk for spontaneous abortions.” D) “Women over age 35 are more likely to have substance abuse problems.” 17. A group of students are reviewing information about sexually transmitted infections and their effect on pregnancy. The students demonstrate understanding of the information when they identify which infection as being responsible for ophthalmia neonatorum? A) Syphilis B) Gonorrhea C) Chlamydia D) HPV 18. A nurse is preparing a presentation for a group of young adult pregnant women about common infections and their effect on pregnancy. When describing the infections, which infection would the nurse include as the most common congenital and perinatal viral infection in the world? A) Rubella B) Hepatitis B C) Cytomegalovirus D) Parvovirus B19 19. A pregnant woman asks the nurse, “I’m a big coffee drinker. Will the caffeine in my coffee hurt my baby?” Which response by the nurse would be most appropriate? A) “The caffeine in coffee has been linked to birth defects.” B) “Caffeine has been shown to cause growth restriction in the fetus.” C) “Caffeine is a stimulant and needs to be avoided completely.” D) “If you keep your intake to less than 300 mg/day, you should be okay.” 20. A neonate born to a mother who was abusing heroin is exhibiting signs and symptoms of withdrawal. Which of the following would the nurse assess? (Select all that apply.) A) Low whimpering cry B) Hypertonicity C) Lethargy D) Excessive sneezing E) Overly vigorous sucking F) Tremors 21. A nurse has been invited to speak at a local high school about adolescent pregnancy. When developing the presentation, the nurse would incorporate information related to which of the following? (Select all that apply.) A) Peer pressure to become sexually active B) Rise in teen birth rates over the years. C) Latinas as having the highest teen birth rate D) Loss of self-esteem as a major impact E) Majority of teen pregnancies in the 15–17-year-old age group 22. A nurse is counseling a pregnant woman with rheumatoid arthritis about medications that can be used during pregnancy. Which drug would the nurse emphasize as being contraindicated at this time? A) Hydroxychloroquine B) Nonsteroidal anti-inflammatory drug C) Glucocorticoid D) Methotrexate 23. A nurse is preparing a teaching program for a group of pregnant women about preventing infections during pregnancy. When describing measures for preventing cytomegalovirus infection, which of the following would the nurse most likely include? A) Frequent handwashing B) Immunization C) Prenatal screening D) Antibody titer screening 24. A pregnant woman tests positive for HBV. Which of the following would the nurse expect to administer? A) HBV immune globulin B) HBV vaccine C) Acylcovir D) Valacyclovir 25. After teaching a pregnant woman with iron deficiency anemia about nutrition, the nurse determines that the teaching was successful when the woman identifies which of the following as being good sources of iron in her diet? (Select all that apply.) A) Dried fruits B) Peanut butter C) Meats D) Milk E) White bread 26. A group of nursing students are preparing a presentation for their class about measures to prevent toxoplasmosis. Which of the following would the students be least likely to include? Select all that apply. A) Washing raw fruits and vegetables before eating them B) Cooking all meat to an internal temperature of 140° F C) Wearing gardening gloves when working in the soil D) Avoiding contact with a cat’s litter box. 27. A pregnant woman with gestational diabetes comes to the clinic for a fasting blood glucose level. When reviewing the results, the nurse determines that which result indicates good glucose control? A) 90 mg/dL B) 100 mg/dL C) 110 mg /dL D) 120 mg/dL Answer Key 1. C 2. B 3. A 4. D 5. A 6. A 7. D 8. A 9. D 10. D 11. B 12. D 13. C 14. D 15. B 16. C 17. B 18. C 19. D 20. B, D, F 21. A, C, D 22. D 23. A 24. A 25. A, B, C 26. B 27. A Chapter 21- Nursing Management of Labor and Birth at Risk 1. After spontaneous rupture of membranes, the nurse notices a prolapsed cord. The nurse immediately places the woman in which position? A) Supine B) Side-lying C) Sitting D) Knee–chest 2. A primigravida whose labor was initially progressing normally is now experiencing a decrease in the frequency and intensity of her contractions. The nurse would assess the woman for which condition? A) A low-lying placenta B) Fetopelvic disproportion C) Contraction ring D) Uterine bleeding 3. The nurse would be alert for possible placental abruption during labor when assessment reveals which of the following? A) Macrosomia B) Gestational hypertension C) Gestational diabetes D) Low parity 4. Assessment of a woman in labor who is experiencing hypertonic uterine dysfunction would reveal contractions that are: A) Well coordinated B) Poor in quality C) Rapidly occurring D) Erratic 5. A woman in labor is experiencing hypotonic uterine dysfunction. Assessment reveals no fetopelvic disproportion. Which group of medications would the nurse expect to administer? A) Sedatives B) Tocolytics C) Oxytocins D) Corticosteroids 6. The fetus of a woman in labor is determined to be in persistent occiput posterior position. Which of the following would the nurse identify as the priority intervention? A) Position changes B) Pain relief measures C) Immediate cesarean birth D) Oxytocin administration 7. A woman gave birth to a newborn via vaginal delivery with the use of a vacuum extractor. The nurse would be alert for which of the following in the newborn? A) Asphyxia B) Clavicular fracture C) Caput succedaneum D) Central nervous system injury 8. A pregnant client undergoing labor induction is receiving an oxytocin infusion. Which of the following findings would require immediate intervention? A) Fetal heart rate of 150 beats/minute B) Contractions every 2 minutes, lasting 45 seconds C) Uterine resting tone of 14 mm Hg D) Urine output of 20 mL/hour 9. A woman with a history of crack cocaine abuse is admitted to the labor and birth area. While caring for the client, the nurse notes a sudden onset of fetal bradycardia. Inspection of the abdomen reveals an irregular wall contour. The client also complains of acute abdominal pain that is continuous. Which of the following would the nurse suspect? A) Amniotic fluid embolism B) Shoulder dystocia C) Uterine rupture D) Umbilical cord prolapse 10. When assessing several women for possible VBAC, which woman would the nurse identify as being the best candidate? A) One who has undergone a previous myomectomy B) One who had a previous cesarean birth via a low transverse incision C) One who has a history of a contracted pelvis D) One who has a vertical incision from a previous cesarean birth 11. A woman is to undergo an amnioinfusion. Which statement would be most appropriate to include when teaching the woman about this procedure? A) “You’ll need to stay in bed while you’re having this procedure.” B) “We’ll give you an analgesic to help reduce the pain.” C) “After the infusion, you’ll be scheduled for a cesarean birth.” D) “A suction cup is placed on your baby’s head to help bring it out.” 12. Which finding would indicate to the nurse that a woman’s cervix is ripe in preparation for labor induction? A) Posterior position B) Firm C) Closed D) Shortened 13. A woman with preterm labor is receiving magnesium sulfate. Which finding would require the nurse to intervene immediately? A) Respiratory rate of 16 breaths per minute B) Diminished deep tendon reflexes C) Urine output of 45 mL/hour D) Alert level of consciousness 14. A woman who is 42 weeks pregnant comes to the clinic. Which of the following would be most important? A) Determining an accurate gestational age B) Asking her about the occurrence of contractions C) Checking for spontaneous rupture of membranes D) Measuring the height of the fundus 15. After teaching a couple about what to expect with their planned cesarean birth, which statement indicates the need for additional teaching? A) “Holding a pillow against my incision will help me when I cough.” B) “I’m going to have to wait a few days before I can start breast-feeding.” C) “I guess the nurses will be getting me up and out of bed rather quickly.” D) “I’ll probably have a tube in my bladder for about 24 hours or so.” 16. The nurse is providing care to several pregnant women who may be scheduled for labor induction. The nurse identifies the woman with which Bishop score as having the best chance for a successful induction and vaginal birth? A) 11 B) 8 C) 6 D) 3 17. After teaching a group of nursing students about risk factors associated with dystocia, the instructor determines that the teaching was successful when the students identify which of the following as increasing the risk? (Select all that apply.) A) Pudendal block anesthetic use B) Multiparity C) Short maternal stature D) Maternal age over 35 E) Breech fetal presentation 18. A nurse is preparing an inservice education program for a group of nurses about dystocia involving problems with the passenger. Which of the following would the nurse most likely include as the most common problem? A) Macrosomia B) Breech presentation C) Persistent occiput posterior position D) Multifetal pregnancy 19. After teaching a group of nursing students about tocolytic therapy, the instructor determines that the teaching was successful when they identify which drug as being used for tocolysis? (Select all that apply.) A) Nifedipine B) Terbutaline C) Dinoprostone D) Misoprostol E) Indomethacin 20. A nurse is assessing a pregnant woman who has come to the clinic. The woman reports that she feels some heaviness in her thighs since yesterday. The nurse suspects that the woman may be experiencing preterm labor based on which additional assessment findings? A) Dull low backache B) Malodorous vaginal discharge C) Dysuria D) Constipation 21. A nurse is teaching a pregnant woman at risk for preterm labor about what to do if she experiences signs and symptoms. The nurse determines that the teaching was successful when the woman states that if she experiences any symptoms, she will do which of the following? A) “I’ll sit down to rest for 30 minutes.” B) “I’ll try to move my bowels.” C) “I’ll lie down with my legs raised.” D) “I’ll drink several glasses of water.” 22. A nurse is describing the risks associated with prolonged pregnancies as part of an inservice presentation. Which of the following would the nurse be least likely to incorporate in the discussion as an underlying reason for problems in the fetus? A) Aging of the placenta B) Increased amniotic fluid volume C) Meconium aspiration D) Cord compression 23. A group of nursing students are reviewing information about methods used for cervical ripening. The students demonstrate understanding of the information when they identify which of the following as a mechanical method? A) Herbal agents B) Laminaria C) Membrane stripping D) Amniotomy 24. The nurse notifies the obstetrical team immediately because the nurse suspects that the pregnant woman may be exhibiting signs and symptoms of amniotic fluid embolism. Which findings would the nurse most likely assess? (Select all that apply.) A) Significant difficulty breathing B) Hypertension C) Tachycardia D) Pulmonary edema E) Bleeding with bruising 25. A group of nursing students are reviewing information about cesarean birth. The students demonstrate understanding of the information when they identify which of the following as an appropriate indication? (Select all that apply) A) Active genital herpes infection B) Placenta previa C) Previous cesarean birth D) Prolonged labor E) Fetal distress 26. A pregnant woman is receiving misoprostol to ripen her cervix and induce labor. The nurse assesses the woman closely for which of the following? A) Uterine hyperstimulation B) Headache C) Blurred vision D) Hypotension Answer Key 1. D 2. B 3. B 4. D 5. C 6. B 7. C 8. D 9. C 10. B 11. A 12. D 13. B 14. A 15. B 16. A 17. C, D, E 18. C 19. A, B, E 20. C 21. D 22. B 23. B 24. A, C, D, E 25. A, B, C, E 26. A Chapter 22- Nursing Management of the Postpartum Woman at Risk 1. Review of a primiparous woman’s labor and birth record reveals a prolonged second stage of labor and extended time in the stirrups. Based on an interpretation of these findings, the nurse would be especially alert for which of the following? A) Retained placental fragments B) Hypertension C) Thrombophlebitis D) Uterine subinvolution 2. As part of an inservice program, a nurse is describing a transient, self-limiting mood disorder that affects mothers after childbirth. The nurse correctly identifies this as postpartum: A) Depression B) Psychosis C) Bipolar disorder D) Blues 3. A woman who is 2 weeks postpartum calls the clinic and says, “My left breast hurts.” After further assessment on the phone, the nurse suspects the woman has mastitis. In addition to pain, the nurse would assess for which of the following? A) An inverted nipple on the affected breast B) No breast milk in the affected breast C) An ecchymotic area on the affected breast D) Hardening of an area in the affected breast 4. A group of students are reviewing the causes of postpartum hemorrhage. The students demonstrate understanding of the information when they identify which of the following as the most common cause? A) Labor augmentation B) Uterine atony C) Cervical or vaginal lacerations D) Uterine inversion 5. After presenting a class on measures to prevent postpartum hemorrhage, the presenter determines that the teaching was successful when the class states which of the following as an important measure to prevent postpartum hemorrhage due to retained placental fragments? A) Administering broad-spectrum antibiotics B) Inspecting the placenta after delivery for intactness C) Manually removing the placenta at delivery D) Applying pressure to the umbilical cord to remove the placenta 6. A multipara client develops thrombophlebitis after delivery. Which of the following would alert the nurse to the need for immediate intervention? A) Dyspnea, diaphoresis, hypotension, and chest pain B) Dyspnea, bradycardia, hypertension, and confusion C) Weakness, anorexia, change in level of consciousness, and coma D) Pallor, tachycardia, seizures, and jaundice 7. A client experienced prolonged labor with prolonged premature rupture of membranes. The nurse would be alert for which of the following in the mother and the newborn? A) Infection B) Hemorrhage C) Trauma D) Hypovolemia 8. When assessing the postpartum woman, the nurse uses indicators other than pulse rate and blood pressure for postpartum hemorrhage based on the knowledge that: A) These measurements may not change until after the blood loss is large B) The body’s compensatory mechanisms activate and prevent any changes C) They relate more to change in condition than to the amount of blood lost D) Maternal anxiety adversely affects these vital signs 9. The nurse is assessing a woman with abruption placentae who has just given birth. The nurse would be alert for which of the following? A) Severe uterine pain B) Board-like abdomen C) Appearance of petechiae D) Inversion of the uterus 10. A nurse is assessing a postpartum woman. Which finding would cause the nurse to be most concerned? A) Leg pain on ambulation with mild ankle edema B) Calf pain with dorsiflexion of the foot. C) Perineal pain with swelling along the episiotomy D) Sharp stabbing chest pain with shortness of breath 11. A woman experiencing postpartum hemorrhage is ordered to receive a uterotonic agent. Which of the following would the nurse least expect to administer? A) Oxytocin B) Methylergonovine C) Carboprost D) Terbutaline 12. Which of the following would be most appropriate when massaging a woman’s fundus? A) Place the hands on the sides of the abdomen to grasp the uterus. B) Use an up-and-down motion to massage the uterus. C) Wait until the uterus is firm to express clots. D) Continue massaging the uterus for at least 5 minutes. 13. After teaching a woman with a postpartum infection about care after discharge, which client statement indicates the need for additional teaching? A) “I need to call my doctor if my temperature goes above 100.4° F.” B) “When I put on a new pad, I’ll start at the back and go forward.” C) “If I have chills or my discharge has a strange odor, I’ll call my doctor.” D) “I’ll point the spray of the peribottle so the water flows front to back.” 14. A nurse suspects that a postpartum client is experiencing postpartum psychosis. Which of the following would most likely lead the nurse to suspect this condition? A) Delirium B) Feelings of anxiety C) Sadness D) Insomnia 15. A postpartum woman is diagnosed with metritis. The nurse interprets this as an infection involving which of the following? (Select all that apply.) A) Endometrium B) Decidua C) Myometrium D) Broad ligament E) Ovaries F) Fallopian tubes 16. A group of nursing students are reviewing information about mastitis and its causes. The students demonstrate understanding of the information when they identify which of the following as the most common cause? A) E. coli B) S. aureus C) Proteus D) Klebsiella 17. A home health care nurse is assessing a postpartum woman who was discharged 2 days ago. The woman tells the nurse that she has a low-grade fever and feels “lousy.” Which of the following findings would lead the nurse to suspect metritis? (Select all that apply.) A) Lower abdominal tenderness B) Urgency C) Flank pain D) Breast tenderness E) Anorexia 18. A postpartum client comes to the clinic for her routine 6-week visit. The nurse assesses the client and suspects that she is experiencing subinvolution based on which of the following? A) Nonpalpable fundus B) Moderate lochia serosa C) Bruising on arms and legs D) Fever 19. Assessment of a postpartum client reveals a firm uterus with bright-red bleeding and a localized bluish bulging area just under the skin at the perineum. The woman also is complaining of significant pelvic pain and is experiencing problems with voiding. The nurse suspects which of the following? A) Hematoma B) Laceration C) Bladder distention D) Uterine atony 20. A postpartum woman is ordered to receive oxytocin to stimulate the uterus to contract. Which of the following would be most important for the nurse to do? A) Administer the drug as an IV bolus injection. B) Give as a vaginal or rectal suppository. C) Piggyback the IV infusion into a primary line. D) Withhold the drug if the woman is hypertensive. 21. Assessment of a postpartum woman experiencing postpartum hemorrhage reveals mild shock. Which of the following would the nurse expect to assess? (Select all that apply.) A) Diaphoresis B) Tachycardia C) Oliguria D) Cool extremities E) Confusion 22. A group of students are reviewing risk factors associated with postpartum hemorrhage. The students demonstrate understanding of the information when they identify which of the following as associated with uterine tone? (Select all that apply.) A) Rapid labor B) Retained blood clots C) Hydramnios D) Operative birth E) Fetal malpostion 23. A nurse is massaging a postpartum client’s fundus and places the nondominant hand on the area above the symphysis pubis based on the understanding that this action: A) Determines that the procedure is effective B) Helps support the lower uterine segment C) Aids in expressing accumulated clots D) Prevents uterine muscle fatigue 24. A nurse is developing a plan of care for a woman who is at risk for thromboembolism. Which of the following would the nurse include as the most cost-effective method for prevention? A) Prophylactic heparin administration B) Compression stocking C) Early ambulation D) Warm compresses 25. A postpartum woman who developed deep vein thrombosis is being discharged on anticoagulant therapy. After teaching the woman about this treatment, the nurse determines that additional teaching is needed when the woman states which of the following? A) “I will use a soft toothbrush to brush my teeth.” B) “I can take ibuprofen if I have any pain.” C) “I need to avoid drinking any alcohol.” D) “I will call my health care provider if my stools are black and tarry.” 26. The nurse is developing a discharge teaching plan for a postpartum woman who has developed a postpartum infection. Which of the following would the nurse most likely include in this teaching plan? (Select all that apply.) A) Taking the prescribed antibiotic until it is finished B) Checking temperature once a week C) Washing hands before and after perineal care D) Handling perineal pads by the edges E) Directing peribottle to flow from back to front 27. A nurse is assessing a postpartum client who is at home. Which statement by the client would lead the nurse to suspect that the client may be developing postpartum depression? A) “I just feel so overwhelmed and tired.” B) “I’m feeling so guilty and worthless lately.” C) “It’s strange, one minute I’m happy, the next I’m sad.” D) “I keep hearing voices telling me to take my baby to the river.” Answer Key 1. C 2. D 3. D 4. B 5. B 6. A 7. A 8. A 9. C 10. D 11. D 12. C 13. B 14. A 15. A, B, C 16. B 17. A, E 18. B 19. A 20. C 21. A, D 22. A, C 23. B 24. C 25. B 26. A, C, D 27. B Chapter 23- Nursing Care of the Newborn With Special Needs 1. The nurse is teaching a group of students about the differences between a full-term newborn and a preterm newborn. The nurse determines that the teaching is effective when the students state that the preterm newborn has: A) Fewer visible blood vessels through the skin B) More subcutaneous fat in the neck and abdomen C) Well-developed flexor muscles in the extremities D) Greater surface area in proportion to weight 2. When assessing a postterm newborn, which of the following would the nurse correlate with this gestational age variation? A) Moist, supple, plum skin appearance B) Abundant lanugo and vernix C) Thin umbilical cord D) Absence of sole creases 3. The parents of a preterm newborn being cared for in the neonatal intensive care unit (NICU. are coming to visit for the first time. The newborn is receiving mechanical ventilation and intravenous fluids and medications and is being monitored electronically by various devices. Which action by the nurse would be most appropriate? A) Suggest that the parents stay for just a few minutes to reduce their anxiety. B) Reassure them that their newborn is progressing well. C) Encourage the parents to touch their preterm newborn. D) Discuss the care they will be giving the newborn upon discharge. 4. When performing newborn resuscitation, which action would the nurse do first? A) Intubate with an appropriate-sized endotracheal tube. B) Give chest compressions at a rate of 80 times per minute. C) Administer epinephrine intravenously. D) Suction the mouth and then the nose. 5. The nurse frequently assesses the respiratory status of a preterm newborn based on the understanding that the newborn is at increased risk for respiratory distress syndrome because of which of the following? A) Inability to clear fluids B) Immature respiratory control center C) Deficiency of surfactant D) Smaller respiratory passages 6. The nurse prepares to assess a newborn who is considered to be large for gestational age (LGA). Which of the following would the nurse correlate with this gestational age variation? A) Strong, brisk motor skills B) Difficulty in arousing to a quiet alert state C) Birth weight of 7 lb 14 oz D) Wasted appearance of extremities 7. An LGA newborn has a blood glucose level of 30 mg/dL and is exhibiting symptoms of hypoglycemia. Which of the following would the nurse do next? A) Administer intravenous glucose immediately. B) Feed the newborn 2 ounces of formula. C) Initiate blow-by oxygen therapy. D) Place the newborn under a radiant warmer. 8. A group of pregnant women are discussing high-risk newborn conditions as part of a prenatal class. When describing the complications that can occur in these newborns to the group, which would the nurse include as being at lowest risk? A) Small-for-gestational-age (SGA. newborns B) Large-for-gestational-age (LGA. newborns C) Appropriate-for-gestational-age (AGA. newborns D) Low-birth-weight newborns 9. While caring for a preterm newborn receiving oxygen therapy, the nurse monitors the oxygen therapy duration closely based on the understanding that the newborn is at risk for which of the following? A) Retinopathy of prematurity B) Metabolic acidosis C) Infection D) Cold stress 10. When planning the care for an SGA newborn, which action would the nurse determine as a priority? A) Preventing hypoglycemia with early feedings B) Observing for respiratory distress syndrome C) Promoting bonding between the parents and the newborn D) Monitoring vital signs every 2 hours 11. A woman gives birth to a newborn at 36 weeks’ gestation. She tells the nurse, “I’m so glad that my baby isn’t premature.” Which response by the nurse would be most appropriate? A) “You are lucky to have given birth to a term newborn.” B) “We still need to monitor him closely for problems.” C) “How do you feel about delivering your baby at 36 weeks?” D) “Your baby is premature and needs monitoring in the NICU.” 12. Which of the following would be most appropriate for the nurse to do when assisting parents who have experienced the loss of their preterm newborn? A) Avoid using the terms “death” or “dying.” B) Provide opportunities for them to hold the newborn. C) Refrain from initiating conversations with the parents. D) Quickly refocus the parents to a more pleasant topic. 13. Which of the following, if noted in the maternal history, would the nurse identify as possibly contributing to the birth of an LGA newborn? A) Drug abuse B) Diabetes C) Preeclampsia D) Infection 14. Which of the following would alert the nurse to suspect that a preterm newborn is in pain? A) Bradycardia B) Oxygen saturation level of 94% C) Decreased muscle tone D) Sudden high-pitched cry 15. When describing newborns with birth-weight variations to a group of nursing students, the instructor identifies which variation if the newborn weighs 5.2 lb at any gestational age? A) Small for gestational age B) Low birth weight C) Very low birth weight D) Extremely low birth weight 16. A nurse is assessing a newborn who has been classified as small for gestational age. Which of the following would the nurse expect to find? (Select all that apply.) A) Wasted extremity appearance B) Increased amount of breast tissue C) Sunken abdomen D) Adequate muscle tone over buttocks E) Narrow skull sutures 17. The nurse is reviewing the medical record of a newborn born 2 hours ago. The nurse notes that the newborn was delivered at 35 weeks’ gestation. The nurse would classify this newborn as which of the following? A) Preterm B) Late preterm C) Full term D) Postterm 18. A nursing instructor is describing common problems associated with preterm birth. When describing the preterm newborn’s risk for perinatal asphyxia, the instructor includes which of the following as contributing to the newborn’s risk? (Select all that apply.) A) Surfactant deficiency B) Placental deprivation C) Immaturity of the respiratory control centers D) Decreased amounts of brown fat E) Depleted glycogen stores 19. After determining that a newborn is in need of resuscitation, which of the following would the nurse do first? A) Dry the newborn thoroughly B) Suction the airway C) Administer ventilations D) Give volume expanders 20. A nurse is developing a plan of care for a preterm infant experiencing respiratory distress. Which of the following would the nurse be least likely to include in this plan? A) Stimulate the infant with frequent handling. B) Keep the newborn in a warmed isolette. C) Administer oxygen using a oxygen hood. D) Give gavage or continous tube feedings. 21. A nurse suspects that a preterm newborn is having problems with thermal regulation. Which of the following would support the nurse’s suspicion? (Select all that apply.) A) Shallow, slow respirations B) Cyanotic hands and feet C) Irritability D) Hypertonicity E) Feeble cry 22. The nurse is assessing a preterm newborn’s fluid and hydration status. Which of the following would alert the nurse to possible overhydration? A) Decreased urine output B) Tachypnea C) Bulging fontanels D) Elevated temperature 23. The nurse is assessing a preterm newborn who is in the neonatal intensive care unit (NICU. for signs and symptoms of overstimulation. Which of the following would the nurse be least likely to assess? A) Increased respirations B) Flaying hands C) Periods of apnea D) Decreased heart rate 24. A group of nursing students are reviewing the literature in preparation for a class presentation on newborn pain prevention and management. Which of the following would the students be most likely to find about this topic? A) Newborn pain is frequently recognized and treated B) Newborns rarely experience pain with procedures C) Pain is frequently mistaken for irritability or agitation D) Newborns may be less sensitive to pain than adult. 25. A nurse is developing a plan of care for a preterm newborn to address the nursing diagnosis of risk for delayed development. Which of the following would the nurse include? (Select all that apply.) A) Clustering care to promote rest B) Positioning newborn in extension C) Using kangaroo care D) Loosely covering the newborn with blankets E) Providing nonnutritive sucking 26. A nurse is assisting the anxious parents of a preterm newborn to cope with the situation. Which statement by the nurse would be least appropriate? A) “I’ll be here to help you all along the way.” B) “What has helped you to deal with stressful situations in the past?” C) “Let me tell you about what you will see when you visit your baby.” D) “Forget about what’s happened in the past and focus on the now.” Answer Key 1. D 2. C 3. C 4. D 5. C 6. B 7. A 8. C 9. A 10. A 11. B 12. B 13. B 14. D 15. B 16. A, C, E 17. B 18. A, C 19. A 20. C 21. A, B, E 22. C 23. A 24. C 25. A, C, E 26. D Chapter 24- Nursing Management of the Newborn at Risk 1. A newborn with severe meconium aspiration syndrome (MAS. is not responding to conventional treatment. Which of the following would the nurse anticipate as possibly necessary for this newborn? A) Extracorporeal membrane oxygenation (ECMO) B) Respiratory support with a ventilator C) Insertion of a laryngoscope for deep suctioning D) Replacement of an endotracheal tube via x-ray 2. Which of the following would the nurse expect to assess in a newborn who develops sepsis? A) Increased urinary output B) Interest in feeding C) Hypothermia D) Wakefulness 3. Which of the following would the nurse include in the plan of care for a newborn receiving phototherapy? A) Keeping the newborn in the supine position B) Covering the newborn’s eyes while under the bililights C) Ensuring that the newborn is covered or clothed D) Reducing the amount of fluid intake to 8 ounces daily 4. A newborn has been diagnosed with a Group B streptococcal infection shortly after birth. The nurse understands that the newborn most likely acquired this infection from which of the following? A) Improper handwashing B) Contaminated formula C) Nonsterile catheter insertion D) Mother’s birth canal 5. Which action would be most appropriate for the nurse to take when a newborn has an unexpected anomaly at birth? A) Show the newborn to the parents as soon as possible while explaining the defect. B) Remove the newborn from the birthing area immediately. C) Inform the parents that there is nothing wrong at the moment. D) Tell the parents that the newborn must go to the nursery immediately. 6. The nurse prepares to administer a gavage feeding for a newborn with transient tachypnea based on the understanding that this type of feeding is necessary for which reason? A) Lactase enzymatic activity is not adequate. B) Oxygen demands need to be reduced. C) Renal solute lead must be considered. D) Hyperbilirubinemia is likely to develop. 7. Which of the following would the nurse include when teaching a new mother about the difference between pathologic and physiologic jaundice? A) Physiologic jaundice results in kernicterus. B) Pathologic jaundice appears within 24 hours after birth. C) Both are treated with exchange transfusions of maternal O- blood. D) Physiologic jaundice requires transfer to the NICU. 8. When planning the care of a newborn addicted to cocaine who is experiencing withdrawal, which of the following would be least appropriate to include? A) Wrapping the newborn snugly in a blanket B) Waking the newborn every hour C) Checking the newborn’s fontanels D) Offering a pacifier 9. A newborn is suspected of having fetal alcohol syndrome. Which of the following would the nurse expect to assess? A) Bradypnea B) Hydrocephaly C) Flattened maxilla D) Hypoactivity 10. After teaching the parents of a newborn with periventricular hemorrhage about the disorder and treatment, which statement by the parents indicates that the teaching was successful? A) “We’ll make sure to cover both of his eyes to protect them.” B) “Our newborn could develop a learning disability later on.” C) “Once the bleeding ceases, there won’t be any more worries.” D) “We need to get family members to donate blood for transfusion.” 11. A newborn has an Apgar score of 6 at 5 minutes. Which of the following is the priority? A) Initiating IV fluid therapy B) Beginning resuscitative measures C) Promoting kangaroo care D) Obtaining a blood culture 12. While reviewing a newborn’s medical record, the nurse notes that the chest x-ray shows a ground glass pattern. The nurse interprets this as indicative of: A) Respiratory distress syndrome B) Transient tachypnea of the newborn C) Asphyxia D) Persistent pulmonary hypertension 13. A newborn is suspected of developing persistent pulmonary hypertension. The nurse would expect to prepare the newborn for which of the following to confirm the suspicion? A) Chest x-ray B) Blood cultures C) Echocardiogram D) Stool for occult blood 14. Which of the following would alert the nurse to suspect that a newborn has developed NEC? A) Irritability B) Sunken abdomen C) Clay-colored stools D) Bilious vomiting 15. Which of the following would not be considered a risk factor for bronchopulmonary dysplasia (chronic lung disease)? A) Preterm birth (less than 32 weeks) B) Female gender C) White race D) Sepsis 16. A group of nursing students are reviewing the different types of congenital heart disease in infants. The students demonstrate a need for additional review when they identify which of the following as an example of increased pulmonary blood flow (left-to-right shunting)? A) Atrial septal defect B) Tetralogy of Fallot C) Ventricular septal defect D) Patent ductus arteriosus 17. After teaching the parents of a newborn with retinopathy of prematurity (ROP. about the disorder and treatment, which statement by the parents indicates that the teaching was successful? A) “Can we schedule follow-up eye examinations with the pediatric ophthalmologist now?” B) “We can fix the problem with surgery.” C) “We’ll make sure to administer eye drops each day for the next few weeks.” D) “I’m sure the baby will grow out of it.” 18. The nurse is assessing the newborn of a mother who had gestational diabetes. Which of the following would the nurse expect to find? (Select all that apply.) A) Pale skin color B) Buffalo hump C) Distended upper abdomen D) Excessive subcutaneous fat E) Long slender neck 19. The nurse is assessing a newborn who is large for gestational age. The newborn was born breech. The nurse suspects that the newborn may have experienced trauma to the upper brachial plexus based on which assessment findings? A) Absent grasp reflex B) Hand weakness C) Absent Moro reflex D) Facial asymmetry 20. The nurse is assessing a newborn and suspects that the newborn was exposed to drugs in utero because the newborn is exhibiting signs of neonatal abstinence syndrome. Which of the following would the nurse expect to assess? (Select all that apply.) A) Tremors B) Diminished sucking C) Regurgitation D) Shrill, high-pitched cry E) Hypothermia F) Frequent sneezing 21. A nurse is developing a plan of care for a newborn with omphalocele. Which of the following would the nurse include? A) Placing the newborn into a sterile drawstring bowel bag B) Using clean technique for dressing changes C) Preparing the newborn for incision and drainage D) Instituting gavage feedings 22. A nurse is explaining to the parents of a child with bladder exstrophy about the care their infant requires. Which of the following would the nurse include in the explanation? (Select all that apply.) A) Covering the area with a sterile, clear, nonadherent dressing B) Irrigating the surface with sterile saline twice a day C) Monitoring drainage through the suprapubic catheter D) Administering prescribed antibiotic therapy E) Preparing for surgical intervention in about 2 weeks 23. A nursing student is preparing a presentation for the class on clubfoot. The student determines that the presentation was successful when the class states which of the following? A) Clubfoot is a common genetic disorder. B) The condition affects girls more often than boys. C) The exact cause of clubfoot is not known. D) The intrinsic form can be manually reduced. 24. Assessment of newborn reveals a large protruding tongue, slow reflexes, distended abdomen, poor feeding, hoarse cry, goiter and dry skin. Which of the following would the nurse suspect? A) Phenylketonuria B) Galactosemia C) Congenital hypothyroidism D) Maple syrup urine disease 25. A group of students are reviewing information about the effects of substances on the newborn. The students demonstrate understanding of the information when they identify which drug as not being associated with teratogenic effects on the fetus? A) Alcohol B) Nicotine C) Marijuana D) Cocaine 26. A nurse is teaching the mother of a newborn diagnosed with galactosemia about dietary restrictions. The nurse determines that the mother has understood the teaching when she identifies which of the following as needing to be restricted? A) Phenylalanine B) Protein C) Lactose D) Iodine 27. A newborn was diagnosed with a congenital heart defect and will undergo surgery at a later time. The nurse is teaching the parents about signs and symptoms that need to be reported. The nurse determines that the parents have understood the instructions when they state that they will report which of the following? (Select all that apply.) A) Weight loss B) Pale skin C) Fever D) Absence of edema E) Increased respiratory rate 28. When developing the plan of care for a newborn with an acquired condition, which of the following would the nurse include to promote participation by the parents? A) Use verbal instructions primarily for explanations B) Assist with decision making process C) Provide personal views about their decisions D) Encourage them to refrain from showing emotions 29. A nurse is assisting in the resuscitation of a newborn. The nurse would expect to stop resuscitation efforts when the newborn has no heartbeat and respiratory effort after which time frame? A) 5 minutes B) 10 minutes C) 15 minutes D) 20 minutes 30. A newborn is diagnosed with meconium aspiration syndrome. When assessing this newborn, which of the following would the nurse expect to find? (Select all that apply.) A) Pigeon chest B) Prolonged tachypnea C) Intercostal retractions D) High blood pH level E) Coarse crackles on auscultation Answer Key 1. A 2. C 3. B 4. D 5. A 6. B 7. B 8. B 9. C 10. B 11. B 12. A 13. C 14. D 15. B 16. B 17. A 18. B, C, D 19. C 20. A, C, D, F 21. A 22. A, C, D 23. C 24. C 25. C 26. C 27. A, C, E 28. B 29. B 30. B, C, E Chapter 25- Growth and Development of the Newborn and Infant 1. The nurse is examining a 10-month-old boy who was born 10 weeks early. Which of the following findings is cause for concern? A) The child has doubled his birthweight. B) The child exhibits plantar grasp reflex. C) The child's head circumference is 19.5 inches. D) No primary teeth have erupted yet. 2. The nurse is teaching a new mother about the drastic growth and developmental changes her infant will experience in the first year of life. Which of the following describes a developmental milestone occurring in infancy? A) By 6 months of age the infant's brain weighs half that of the adult brain; at age 12 months, the brain weighs 2.5 times what it did at birth. B) Most infants triple their birthweight by 4 to 6 months of age and quadruple their birthweight by the time they are 1 year old. C) The head circumference increases rapidly during the first 6 months: the average increase is about 1 inch per month. D) The heart triples in size over the first year of life; the average pulse rate decreases from 120 to 140 in the newborn to about 100 in the 1-year-old. 3. The nurse measures the head circumference of a 6-month-old infant. Which measurement would the nurse interpret as most appropriate? A) 33 cm B) 35 cm C) 43.5 cm D) 47 cm 4. The nurse is helping a new mother prepare for breastfeeding her infant. During which of the following newborn states of consciousness would the nurse recommended attempting the feeding? A) Light sleep B) Drowsiness C) Quiet alert state D) Active alert state 5. The nurse is assessing the respiratory system of a newborn. Which of the following anatomic differences place the infant at risk for respiratory compromise? Select all answers that apply. A) The nasal passages are narrower. B) The trachea and chest wall are less compliant. C) The bronchi and bronchioles are shorter and wider. D) The larynx is more funnel shaped. E) The tongue is smaller. F) There are significantly fewer alveoli. 6. A new mother shows the nurse that her baby grasps her finger when she touches the baby's palm. How might the nurse respond to this information? A) "This is a primitive reflex known as the plantar grasp." B) "This is a primitive reflex known as the palmar grasp." C) "This is a protective reflex known as rooting." D) "This is a protective reflex known as the Moro reflex." 7. Which reflex, if found in a 4-month-old infant, would cause the nurse to be concerned? A) Plantar grasp B) Step C) Babinski D) Neck righting 8. A new mother expresses concern to the nurse that her baby is crying and grunting when passing stool. What is the nurse's best response to this observation? A) "This is normal behavior for infants unless the stool passed is hard and dry." B) "This is normal behavior for infants due to the immaturity of the gastrointestinal system." C) "This indicates a blockage in the intestine and must be reported to the physician." D) "This is normal behavior for infants unless the stool passed is black or green." 9. The neonatal nurse assesses newborns for iron-deficiency anemia. Which of the following newborns is at highest risk for this disorder? A) A postterm newborn B) A term newborn with jaundice C) A newborn born to a diabetic mother D) A premature newborn 10. The nurse caring for newborns knows that infants exhibit phenomenal increases in their gross motor skills over the first 12 months of life. Which of the following statements accurately describe the typical infant's achievement of these milestones? Select all answers that apply. A) At 1 month the infant lifts and turns the head to the side in the prone position. B) At 2 months the infant lifts head and looks around. C) At 6 months the infant pulls to stand up. D) At 7 months the infant sits alone with some use of hands for support. E) At 9 months the infant crawls with the abdomen off the floor. F) At 12 months the infant walks independently. 11. At which age would the nurse expect to find the beginning of object permanence? A) 1 month B) 4 months C) 8 months D) 12 months 12. The nurse is teaching a new mother about the development of sensory skills in her newborn. Which of the following would alert the mother to a sensory deficit in her child? A) The newborn's eyes wander and occasionally are crossed. B) The newborn does not respond to a loud noise. C) The newborn's eyes focus on near objects. D) The newborn becomes more alert with stroking when drowsy. 13. The nurse is assessing a 4-month-old boy during a scheduled visit. Which of the following findings might suggest a developmental problem? A) The child does not coo or gurgle. B) The child does not babble or laugh. C) The child never squeals or yells. D) The child does not say dada or mama. 14. The nurse observes an infant interacting with his parents. Which of the following are normal social behavioral developments for this age group? Select all answers that apply. A) Around 5 months the infant may develop stranger anxiety. B) Around 2 months the infant exhibits a first real smile. C) Around 3 months the infant smiles widely and gurgles when interacting with the caregiver. D) Around 3 months the infant will mimic the parent's facial movements, such as sticking out the tongue. E) Around 3 to 6 months of age the infant may enjoy socially interactive games such as pattycake and peek-a-boo. F) Separation anxiety may also start in the last few months of infancy. 15. The nurse is performing a health assessment of a 3-month-old Black American boy. For what condition should this infant be monitored based on his race? A) Jaundice B) Iron deficiency C) Lactose intolerance D) Gastroesophageal reflux disease (GERD) 16. The nurse is promoting a healthy diet to guide a mother when feeding her 2-week-old girl. Which of the following is the most effective anticipatory guidance? A) Encouraging breastfeeding until the sixth month B) Advocating iron supplements with bottle-feeding C) Advising fluid intake per feeding of 5 or 6 ounces D) Discouraging the addition of fruit juice to the diet 17. The nurse is teaching a new mother the proper techniques for breastfeeding her newborn. Which of the following is a recommended guideline that should be implemented? A) Wash the hands and breasts thoroughly prior to breastfeeding. B) Stroke the nipple against the baby's chin to stimulate wide opening of the baby's mouth. C) Bring the baby's wide-open mouth to the breast to form a seal around all of the nipple and areola. D) When finished the mother can break the suction by firmly pulling the baby's mouth away from the nipple. 18. The nurse is providing discharge teaching regarding formula preparation for a new mother. Which of the following guidelines would the nurse include in the teaching plan? A) Always wash bottles and nipples in hot soapy water and rinse well; do not wash them in the dishwasher. B) Store tightly covered ready-to-feed formula can after opening in refrigerator for up to 24 hours. C) Warm bottle of formula by placing bottle in a container of hot water, or microwaving formula. D) Do not add cereal to the formula in the bottle or sweeten the formula with honey. 19. The nurse is caring for a 4-week-old girl and her mother. Which of the following is the most appropriate subject for anticipatory guidance? A) Promoting the digestibility of breast milk B) Telling how and when to introduce rice cereal C) Describing root reflex and latching on D) Advising how to choose a good formula 20. The nurse is providing anticipatory guidance to a mother of a 5-month-old boy about introducing solid foods. Which statement by the mother indicates that effective teaching has occurred? A) “I'll start with baby oatmeal cereal mixed with low-fat milk.” B) “The cereal should be a fairly thin consistency at first.” C) “I can puree the meat that we are eating to give to my baby.” D) “Once he gets used to the cereal, then we'll try giving him a cup.” 21. The nurse is providing anticipatory guidance to the mother of a 9-month-old girl during a well-baby visit. Which of the following topics would be most appropriate? A) Advising how to create a toddler-safe home B) Warning about small objects left on the floor C) Cautioning about putting the baby in a walker D) Telling about safety procedures during baths 22. The nurse in a community clinic is caring for a 6-month-old boy and his mother. Which of the following is the priority intervention to promote adequate growth? A) Monitoring the child's weight and height B) Encouraging a more frequent feeding schedule C) Assessing the child's current feeding pattern D) Recommending higher-calorie solid foods 23. The nurse is caring for a 7-month-old girl during a well-child visit. Which of the following interventions is most appropriate for this child? A) Discussing the type of sippy cup to use B) Advising about increased caloric needs C) Explaining how to prepare table meats D) Describing the tongue extrusion reflex 24. The nurse is assessing a 12-month-old boy with an English-speaking father and a Spanishspeaking mother. The boy does not say mama or dada yet. Which of the following is the priority intervention? A) Performing a developmental evaluation of the child B) Encouraging the parents to speak English to the child C) Asking the mother if the child uses Spanish words D) Referring the child to a developmental specialist 25. A 6-month-old girl weighs 14.7 pounds during a scheduled check-up. Her birth weight was 8 pounds. Which of the following is the priority nursing intervention? A) Talking about solid food consumption B) Discouraging daily fruit juice intake C) Increasing the number of breastfeedings D) Discussing the child's feeding patterns 26. The nurse is educating a first-time mother who has a 1-week-old boy. Which of the following is the most accurate anticipatory guidance? A) Describing the effect of neonatal teeth on breastfeeding B) Explaining that the stomach holds less than 1 ounce C) Informing that fontanels will close by 6 months D) Telling that the step reflex persists until the child walks 27. A mother is concerned about her infant's spitting up. Which suggestion would be most appropriate? A) “Put the infant in an infant seat after eating.” B) “Limit burping to once during a feeding.” C) “Feed the same amount but space out the feedings.” D) “Keep the baby sitting up for about 30 minutes afterward.” 28. The nurse is providing anticipatory guidance to a mother to help promote healthy sleep for her 3-week old baby. Which of the following recommended guidelines might be included in the teaching plan? A) Place the baby on a soft mattress with a firm flat pillow for the head. B) Place the head of the bed near the window to provide fresh air, weather permitting. C) Place the baby on his or her back when sleeping. D) If the baby sleeps through the night, wake him or her up for the night feeding. 29. The nurse is counseling the mother of a newborn who is concerned about her baby's constant crying. What teaching would be appropriate for this mother? A) Carrying the baby may increase the length of crying. B) Reducing stimulation may decrease the length of crying. C) Using vibration, white noise, or swaddling may increase crying. D) Using a swing or car ride may increase the incidence of crying episodes. 30. The parent of a 6-month old infant asks the nurse for advice about his son's thumb sucking. What would be the nurse's best response to this parent? A) "Thumb sucking is a healthy self-comforting activity." B) "Thumb sucking leads to the need for orthodontic braces." C) "Caregivers should pay special attention to the thumb sucking to stop it." D) "Thumb sucking should be replaced with the use of a pacifier." Answer Key 1. C 2. A 3. C 4. C 5. A, D, F 6. B 7. B 8. A 9. D 10. A, D, E, F 11. B 12. B 13. B 14. B, C, D, F 15. C 16. D 17. C 18. D 19. B 20. B 21. A 22. A 23. A 24. C 25. D 26. B 27. D 28. C 29. B 30. A Chapter 26- Growth and Development of the Toddler 1. The nurse is performing a physical assessment of a 3-year-old girl. Which of the following would be a concern for the nurse? A) The toddler gained 4 pounds in weight since last year. B) The toddler gained 3 inches in height since last year. C) The toddler's anterior fontanel is not fully closed. D) The circumference of the child's head increased 1 inch since last year. 2. The nurse is describing the maturation of various organ systems during toddlerhood to the parents. Which would the nurse correctly include in this description? A) Myelinization of the brain and spinal cord is complete at about 24 months. B) Alveoli reach adult numbers by 3 years of age. C) Urine output in a toddler typically averages approximately 30 mL/hour. D) Toddlers typically have strong abdominal muscles by the age of 2. 3. The nurse is teaching the parents of a 2-year-old toddler methods of dealing with their child's "negativism." Based on Erokson's theory of development, which of the following would be an appropriate intervention for this child? A) Discourage solitary play; encourage playing with other children. B) Encourage the child to pick out his own clothes. C) Use "time-outs" whenever the child says "no" inappropriately. D) Encourage the child to take turns when playing games. 4. The nurse is caring for a toddler who is in Piaget's sensorimotor stage of cognitive development. Which of the following tasks would the nurse expect the toddler to be able to perform? A) Completing puzzles with four pieces B) Winding up a mechanical toy C) Playing make-believe with dolls D) Knowing which are his or her toys 5. The nurse is observing a 24-month-old boy in a day care center. Which finding suggests delayed motor development? A) The child has trouble undressing himself. B) The child is unable to push a toy lawnmower. C) The child is unable to unscrew a jar lid. D) The child falls when he bends over. 6. Which of the following would the nurse expect to find in an 18-month-old? A) Standing on tiptoes B) Pedaling a tricycle C) Climbing stairs with assistance D) Carrying a large toy while walking 7. The pediatric nurse is planning quiet activities for hospitalized 18-month-olds. Which of the following would be an appropriate activity for this age group? A) Painting by number B) Putting shapes into appropriate holes C) Stacking blocks D) Using crayons to color in a coloring book 8. The nurse is performing a cognitive assessment of a 2-year-old. Which of the following behaviors would alert the nurse to a developmental delay in this area? A) The child cannot say name, age, and gender. B) The child cannot follow a series of two independent commands. C) The child has a vocabulary of 40 to 50 words. D) The child does not point to named body parts. 9. The nurse is interviewing a 3-year-old girl who tells the nurse: "Want go potty." The parents tell the nurse that their daughter often speaks in this type of broken speech. What would be the nurse's appropriate response to this concern? A) "This is a normal, common speech pattern in the 3-year-old and is called telegraphic speech." B) "This is considered a developmental delay in the 3-year-old and we should consult a speech therapist." C) "This is a condition known as echolalia and can be corrected if you work with your daughter on language skills." D) "This is a condition known as stuttering and it is a normal pattern of speech development in the toddler." 10. After teaching a group of parents about language development in toddlers, which of the following if stated by a member of the group indicates successful teaching? A) “When my 3-year-old asks 'why?' all the time, this is completely normal.” B) “A 15-month-old should be able to point to his eyes when asked to do so.” C) “At age 2 years, my son should be able to understand things like under or on.” D) “An 18-month-old would most likely use words and gestures to communicate.” 11. The nurse is testing the sensory development of a toddler brought to the clinic for a well visit. Which of the following might alert the nurse to a potential problem with the child's sensory development? A) The toddler places the nurse's stethoscope in his mouth. B) The toddler's vision tests at 20/50 in both eyes C) The toddler does not respond to commands whispered in his ear. D) The toddler's taste discrimination is not at adult levels yet. 12. The nurse observing toddlers in a day care center notes that they may be happy and pleasant one moment and overreact to limit setting the next minute by throwing a tantrum. What is the focus of the toddler's developmental task that is driving this behavior? A) The need for separation and control B) The need for love and belonging C) The need for safety and security D) The need for peer approval 13. The nurse is teaching parents interventions appropriate to the emotional development of their toddlers. Which of the following is a recommended intervention for this age group? A) Remove children's security blankets at this stage to help them assert their autonomy. B) Distract toddlers from exploring their own body parts, particularly their genitals. C) Do not blame toddlers for aggressive behavior; instead, point out the results of their behavior. D) Offer toddlers many choices to foster control over their environment. 14. The nurse is assessing a 3-year-old boy's development during a well-child visit. Which response by the child indicates the need for further assessment? A) He says a swear word when he hurts himself playing. B) He says “pew” when his sister has soiled her diaper. C) He laughs when his brother cries getting vaccinated. D) He constantly asks “why?” whenever he is told a fact. 15. The nurse is designing a nursing care plan for a toddler with lymphoma, who is hospitalized for treatment. Which of the following is a priority intervention that the nurse should include in this child's nursing plan? A) Limiting visitors to scheduled visiting hours B) Planning physical therapy for the child C) Introducing the toddler to other toddlers in the unit D) Monitoring the toddler for developmental delays 16. The nurse is watching toddlers at play. Which of the following normal behaviors would the nurse observe? A) Toddlers engage in parallel play. B) Toddlers engage in solitary play. C) Toddlers engage in cooperative play. D) Toddlers do not engage in play outside the home. 17. The nurse is developing a teaching plan for toddler safety to present at a parenting seminar. Which of the following are safety interventions that the nurse should address? A) Encourage parents to enroll toddlers in swimming classes to avoid the need for constant supervision around water. B) Advise parents to keep pot handles on stoves turned outward to avoid accidental burns. C) Encourage parents to smoke only in designated rooms in the house or outside the house. D) Advise parents to use a forward-facing car seat with harness straps and a clip, placed in the backseat of the car. 18. During a health history, the nurse explores the sleeping habits of a 3-yearold boy by interviewing his parents. Which of the following statements from the parents reflects a recommended guideline for promoting healthy sleep in this age group? A) "Our son sleeps through the night, and we insist that he takes two naps a day." B) "We keep a strict bedtime ritual for our son, which includes a bath and bedtime story." C) "Our son still sleeps in a crib because we feel it is the safest place for him at night." D) "Our son occasionally experiences night walking so we allow him to stay up later when this happens." 19. The nurse is teaching good sleep habits for toddlers to the mother of a 2-year-old boy. Which response indicates the mother understands sleep requirements for her son? A) “I'll put him to bed at 7 p.m., except Friday and Saturday.” B) “He needs 13 hours of sleep per day including his nap.” C) “I need to put the side down on the crib so he can get out.” D) “His father can give him a horseback ride into his bed.” 20. The parents of a 1-year-old girl, both of whom have perfect teeth, are concerned about their child getting dental caries. Which is the best advice the nurse can provide? A) Tell the parents to limit the child's eating to meal and snack times. B) Urge the parents to take the child to a dentist for a check-up. C) Advise the parents to reduce carbohydrates in the child's diet. D) Advise the parents to use fluoride toothpaste. 21. The nurse is helping parents prepare a healthy meal plan for their toddler. Which of the following guidelines for promoting nutrition should be followed when planning meals? Select all answers that apply. A) The child younger than 2 years of age should have his or her fat intake restricted. B) Extending breastfeeding into toddlerhood is believed to be beneficial to the child. C) Weaning from the bottle should occur by 6 to 12 months of age. D) Adequate calcium intake and appropriate exercise lay the foundation for proper bone mineralization. E) The toddler requires an average intake of 500 mg calcium per day. F) Toddlers tend to have the highest daily iron intake of any age group. 22. The nurse is choosing foods for a toddler's diet that are high in vitamin A. Which of the following could be added to the menu? Select all answers that apply. A) Applesauce B) Avocados C) Broccoli D) Sweet potatoes E) Spinach F) Carrots 23. When instructing the parents of a toddler about appropriate nutrition, which of the following would the nurse recommend? A) About 12 to 16 ounces of fruit juice per day B) Approximately 16 to 24 ounces of milk per day C) Fat intake of 30% to 40% of total calories D) An average of 10 to 12 grams of fiber per day 24. The nurse is teaching the parents of an overweight 18-month-old girl about diet. Which intervention will be most effective for promoting proportionate growth? A) Remove highcalorie, low-nutrient foods from the diet. B) Ensure 30 minutes of unstructured activity per day. C) Avoid sharing your snacks and candy with the child. D) Reduce the amount of high-fat food the child eats. 25. The nurse is providing anticipatory guidance to the parents of an 18-month-old girl. Which guidance will be most helpful for toilet teaching? A) Telling them either one may demonstrate toilet use B) Assuring them that bladder control occurs first C) Telling them that curiosity is a sure sign of readiness D) Advising them to use praise, not scolding 26. The parents of a 2-year-old girl are frustrated by the frequent confrontations they have with their child. Which is the best anticipatory guidance the nurse can offer them? A) “Respond in a calm but firm manner.” B) “You need to adhere to various routines.” C) “Put her in time-out when she misbehaves.” D) “It's important to toddler-proof your home.” 27. The nurse is providing guidance after observing a mother interact with her negative 2- yearold boy. For which interaction will the nurse advise the mother that she is handling the negativism properly? A) Telling the child to stop tearing pages from magazines B) Asking the child if he would please quit throwing toys C) Telling the child firmly that we don't scream in the office D) Saying, “Please come over here and sit in this chair. OK?” 28. The nurse is teaching the parents of a 2-year-old girl how to deal with common toddler situations. Which is the best advice? A) Discipline the child for regressive behavior. B) Scold the child for public thumb sucking. C) Tell the older sibling to not act like a baby. D) Have the child help clean up a bowel accident. 29. The nurse is assessing a 2-year-old boy who has missed some developmental milestones. Which finding will point to the cause of motor skill delays? A) The mother is suffering from depression. B) The child is homeless and has no toys. C) The mother describes an inadequate diet. D) The child is unperturbed by a loud noise. 30. The nurse emphasizes that a toddler younger than the age of 18 months should never be spanked primarily for which reason? A) Spanking in a child this age predisposes the child to a pro-violence attitude. B) The child will become resentful and angry, leading to more outbursts. C) Spanking demonstrates a poor model for problem-solving skills. D) There is an increased risk for physical injury in this age group. Answer Key 1. C 2. A 3. B 4. D 5. B 6. C 7. C 8. D 9. A 10. A 11. C 12. A 13. C 14. C 15. D 16. A 17. D 18. B 19. B 20. A 21. B, D, E 22. D, E, F 23. B 24. A 25. D 26. B 27. C 28. D 29. B 30. D Chapter 27- Growth and Development of the Preschooler 1. The nurse is conducting a well-child examination of a 5-year-old girl, who was 40 inches tall at her last examination at age 4. Which of the following height measurements would be within the normal range of growth expected for a preschooler? A) 41 inches B) 43 inches C) 45 inches D) 47 inches 2. The nurse is teaching the parents of a 4-year-old boy about the normal maturations of the child's organs during the preschool years and their effect on body functions. Which of the following statements accurately describe these changes? Select all answers that apply. A) Myelination of the spinal cord allows for bowel and bladder control to be complete in most children by age 3 years. B) The respiratory structures are continuing to grow in size, and the number of alveoli continues to increase, reaching the adult number at about 7 years of age. C) Heart rate increases and blood pressure decreases slightly during the preschool years; an innocent heart murmur may be heard upon auscultation. D) The bones continue to increase in length and the muscles continue to strengthen and mature; however, the musculoskeletal system is still not fully mature. E) The small intestine is continuing to grow in length, and stool passage usually occurs once or twice per day in the average preschooler. F) The urethra remains long in both boys and girls, making them more susceptible to urinary tract infections than adults. 3. The nurse is assessing the psychosocial development of a preschooler. Which of the following are normal activities characteristic of the preschooler? Select all answers that apply. A) Plans activities and makes up games B) Initiates activities with others C) Acts out roles of other people D) Engages in parallel play with peers E) Classifies or groups objects by their common elements F) Understands relationships among objects 4. The nurse is caring for a 5-year-old girl posttonsillectomy. The girl looks out the window and tells the nurse that it is raining and says, “The sky is crying because it is sad that my throat hurts.” The nurse understands that the girl is demonstrating which mental process? A) Magical thinking B) Centration C) Transduction D) Animism 5. The parents of a 4-year-old ask the nurse when their child will be able to differentiate right from wrong and develop morals. What would be the best response of the nurse? A) "The preschooler has no sense of right and wrong." B) "The preschooler is developing a conscience." C) "The preschooler sees morality as internal to self." D) "The preschooler's morals are their own, right or wrong." 6. Which activity would the nurse least likely include as exemplifying the preconceptual phase of Piaget's preoperational stage? A) Displays of animism B) Use of active imaginations C) Understanding of opposites D) Beginning questioning of parents' values 7. The nurse is assessing the motor skills of a 5-year-old girl. Which finding would cause the nurse to be concerned? A) Can copy a square on another piece of paper B) Can dress and undress herself without help C) Draws a person with three body parts D) Is beginning to tie her own shoelaces 8. The school nurse is helping parents choose books for their preschoolers. What literacy skills present in the preschooler would the nurse consider when making choices? Select all answers that apply. A) Preschoolers enjoy books with pictures that tell stories. B) Preschoolers like stories with repeated phrases as they help keep their attention. C) Preschoolers like stories that describe experiences different from their own. D) Preschoolers demonstrate early literacy skills by reciting stories or portions of books. E) Preschoolers may retell the story from the book, pretend to read books, and ask questions about the story. F) Preschoolers do not have enough focus and expanded attention to notice when a page is skipped during reading. 9. The parents of a preschooler ask the nurse to help them choose a preschool for their child. Which of the following are recommended guidelines and goals for choosing a preschool? Select all answers that apply. A) The main goal of preschool is to improve reading and writing skills and readiness for entering into grade school. B) When selecting a preschool the parent may want to consider the accreditation of the school and the teachers' qualifications. C) The teachers should decide how focused on curriculum the school should be for each individual student. D) The parent should observe the classroom, evaluating the environment, noise level, and sanitary practices. E) The type of discipline used in the school is also an important factor. Parents should choose a preschool that uses corporal punishment. F) The parent should observe the classroom to determine how the children interact with each other and how the teachers interact with the children. 10. The mother of a 4-year-old boy tells the nurse that her son occasionally wets his pants during the day. How should the nurse respond? A) “Is there a family history of diabetes?” B) "Suddenly having accidents can be a sign of diabetes.” C) “That's normal; don't worry about it.” D) “Tell me about the circumstances when this occurs.” 11. The nurse is explaining to parents that the preschooler's developmental task is focused on the development of initiative rather than guilt. Which of the following is a priority intervention the nurse might recommend for parents of preschoolers to stimulate initiative? A) Reward the child for initiative in order to build self-esteem. B) Change the routine of the preschooler often to stimulate initiative. C) Do not set limits on the preschooler's behavior as this results in low self-esteem. D) As a parent, decide how and with whom the child will play. 12. The parents of a 5-year-old boy tell the nurse that their son is having frequent episodes of night terrors. Which of the following statements would indicate that the boy is having nightmares instead of night terrors? A) “It usually happens about an hour after he falls asleep.” B) “He will tell us about what happened in his dream.” C) “He is completely unaware that we are there.” D) “When we try to comfort him, he screams even more.” 13. The mother of a 5-year-old boy calls the nurse and seeks advice on how to assist the child with the recent death of his paternal grandfather. The boy keeps asking when his grandpa is coming back. How should the nurse respond? A) “It is best to just ignore this and to not respond to his questions.” B) “This is normal; children his age do not understand the permanence of death.” C) “You have to keep repeating that his grandfather is never coming back.” D) “He will eventually figure this out on his own.” 14. The nurse is developing a nursing care plan for a hospitalized 6-year-old. Which of the following behaviors would warrant nursing intervention? A) The child pretends he is talking to an imaginary friend when the nurse addresses the child. B) The child states that her fairy godmother is going to come and take her home. C) The child starts talking about his grandmother and then quickly changes the subject to a new toy he received. D) The child does not want to play games with other children on the hospital ward. 15. The nurse is caring for preschoolers in a day care center. Of which of the following developmental milestones of this age group should the nurse be aware? Select all answers that apply. A) Counting 10 or more objects B) Correctly naming at least four colors C) Understanding the concept of time D) Knowing everyday objects E) Understanding the differences of others F) Forming concepts as logical as an adult's 16. When observing a group of preschoolers at play in the clinic waiting room, which type of play would the nurse be least likely to note? A) Parallel play B) Cooperative play C) Dramatic play D) Fantasy play 17. The nurse is supervising lunch time for children on a pediatric ward. Which of the following observations is considered abnormal for this age group? A) The child has a full set of primary teeth. B) The child has no difficulty chewing and swallowing meat. C) The child uses his fingers and refuses to use a fork. D) The child is a picky eater. 18. The nurse of a preschool child is helping parents develop a healthy meal plan for their child. Which of the following nutritional requirements for this age group should the nurse consider? A) The 3- to 5-year-old requires 300 to 500 mg calcium and 10 mg iron daily. B) The 3-year-old should consume 10 mg dietary fiber daily. C) The 4- to 8-year-old requires 15 mg dietary fiber per day. D) The typical preschooler requires about 85 kcal/kg of body weight. 19. The parents of a 4-year-old who is a picky eater ask the nurse what foods to include in their child's diet to provide adequate iron consumption. Which of the following foods would the nurse recommend? A) Cooked lentils B) Whole milk C) Oranges D) Sweet potatoes 20. The nurse is counseling parents of a picky eater on how to promote healthy eating habits in their child. Which of the following interventions would be appropriate advice? A) Allow the child to pick out his or her own foods for meals. B) Present the food matter-of-factly and allow the child to choose what to eat. C) Offer high-fat snacks if the child does not eat to get him or her to eat something. D) Offer the child a special treat if he or she eats all the food on the plate. 21. Which food suggestion would be most appropriate for the mother of a preschooler to ensure an adequate intake of calcium? A) Spinach B) White beans C) Enriched bread D) Fortified cereal 22. The nurse is providing teaching about accidental poisoning to the family of a 3-year-old. The nurse understands that a child of this age is at increased risk of accidental ingestion due to which sensory alteration? A) A less discriminating sense of taste B) A lack of fully developed hearing C) Visual acuity that has not fully developed D) A less discriminating sense of touch 23. The nurse is conducting a well-child assessment for a 5-year-old boy in preparation for kindergarten. The boy's grandmother is his primary caregiver because the boy's mother has suffered from depression and substance abuse issues. The nurse understands that the child is at increased risk for which developmental problem? A) Lack of social and emotional readiness for school B) Stuttering C) Speech and language delays D) Fine motor skills delay 24. A nurse is caring for a 4-year-old girl. The mother says that the girl is afraid of cats and dogs and does not like to go to the playground anymore because she wants to avoid the dogs that are often being walked at the park. What should the nurse tell the mother? A) “It is best to avoid the playground until she outgrows the fear.” B) “She needs to face her fears head-on; take her to the park as much as possible.” C) “Acknowledge her fear and help her develop a strategy for dealing with it.” D) “Try to minimize her fears and insist that she go to the park.” 25. The nurse is providing teaching about car safety to the parents of a 5-year-old girl who weighs 45 pounds. What should the nurse instruct the parents to do? A) “Place her in a booster seat with lap and shoulder belts in the front seat.” B) “Place her in the back seat with the lap and shoulder belts in place.” C) “Place her in a forward-facing car seat with a harness and top tether.” D) “Place her in a booster seat with lap and shoulder belts in the back seat.” 26. The nurse is caring for a premature baby in the NICU. The mother reports that the infant's normally happy and outgoing 5-year-old sister is acting sad and withdrawn. The nurse understands that due to her developmental stage, the girl is at risk of which of the following? A) Viewing her baby sister's illness as her fault B) Harming the baby C) Experiencing clinical depression D) Creating an imaginary friend to cope with the situation 27. When providing anticipatory guidance to parents about lying during the preschool period, which of the following would the nurse emphasize? A) “You need to determine the reason for lying before punishing the child.” B) “Lying typically occurs because the child is afraid of being punished.” C) “The misbehavior is usually more serious than the lying itself.” D) “It is okay to become angry when dealing with the child's lying.” 28. The nurse is providing anticipatory guidance for parents of a preschooler regarding sex education. Which of the following is a recommended guideline when dealing with this issue? A) Be prepared to thoroughly cover a topic before the child asks about it. B) Before answering questions, find out what the child thinks about the subject. C) Expand upon the topic when answering questions to prevent further confusion. D) Provide a less than honest response to shelter the child from knowledge that is too advanced. 29. The parents of a preschooler express concern to the nurse about their son's new habit of masturbating. Which of the following is an appropriate response to this concern? A) Tell the child in a firm manner that this behavior is not acceptable. B) When the child displays this behavior, place him in a "time-out." C) Treat the action in a matter-of-fact manner emphasizing safety. D) Consult a psychotherapist to determine the reason for this behavior.   Answer Key 1. B 2. A, B, D, E 3. A, B, C 4. A 5. B 6. D 7. C 8. A, B, D, E 9. B, D, F 10. D 11. A 12. B 13. B 14. D 15. A, B, C, D 16. A 17. C 18. D 19. A 20. B 21. B 22. A 23. A 24. C 25. D 26. A 27. A 28. B 29. C Chapter 28- Growth and Development of the School-Age Child 1. The nurse is performing a physical assessment of a 10-year-old boy. The nurse notes that during last year's check-up the child weighed 80 pounds. According to average growth for this age group, what would be his expected current weight? A) 83 pounds B) 85 pounds C) 87 pounds D) 89 pounds 2. The nurse is performing an annual check-up for an 8-year-old child. Compared to the previous assessment of this child, which of the following characteristics would most likely be observed? A) Breathing is diaphragmatic. B) Pulse rate is increased. C) Secondary sex characteristics are present. D) Blood pressure has reached adult level. 3. The nurse is preparing a presentation for a local parent–teacher organization about the growth and development of school-age children. Which of the following would the nurse include? A) Boys mature much more quickly than girls of the same age during this time. B) From 6 to 12 years of age, children grow an average of 4 inches per year. C) The child's body size is in direct correlation with his or her maturity level. D) Secondary sex characteristics are often embarrassing for both sexes. 4. The pediatric nurse is aware of the maturation of organ systems in the school-age child. Which of the following accurately describe these changes? Select all answers that apply. A) The brain grows very slowly during the school-age years and growth is complete by the time the child is 12 years of age. B) Respiratory rates decrease, abdominal breathing disappears, and respirations become diaphragmatic in nature. C) The school-age child's blood pressure increases and the pulse rate decreases, and the heart grows more slowly during the middle years. D) The school-age child experiences more gastrointestinal upsets compared with earlier years since the stomach capacity increases. E) Bladder capacity increases, but varies among individual children, and girls generally have a greater bladder capacity than boys. F) Prepubescence typically occurs in the 2 years before the beginning of puberty and is characterized by the development of secondary sexual characteristics. 5. The nurse is using the formula for bladder capacity to measure the bladder capacity of a 9year-old girl. What number would the nurse document for this measurement? A) 9 ounces B) 10 ounces C) 11 ounces D) 12 ounces 6. The nurse knows that the school-age child is in Erikson's stage of industry versus inferiority. Which of the following is the best example of a school-ager working toward accomplishing this developmental task? A) The child signs up for after-school activities. B) The child performs his bedtime preparations autonomously. C) The child becomes aware of the opposite sex. D) The child is developing a conscience. 7. The school nurse providing school health screenings knows that the 7- to 11-year-old is in Piaget's stage of concrete operational thoughts. Which of the following should this age group accomplish when developing operations? Select all answers that apply. A) Ability to assimilate and coordinate information about the world from different dimensions B) Ability to see things from another person's point of view and think through an action C) Ability to use stored memories of past experiences to evaluate and interpret present situations D) Ability to think about a problem from all points of view, ranking the possible solutions while solving the problem E) Ability to think outside of the present and incorporate into thinking concepts that do exist as well as concepts that might exist F) Ability to understand the principle of conservation—that matter does not change when its form changes 8. The nurse explains to parents of school-age children that according to Kohlberg's theory of moral development, their child is at the conventional stage of moral development. What is the motivation for school-age children to follow rules? A) They follow rules out of a sense of being a "good person." B) They follow rules out of fear of being punished. C) They follow rules in order to receive praise from caretakers. D) They follow rules because it is in their nature to do so. 9. The nurse is talking with a chatty 7-year-old girl during her regular check-up. Which of the following behaviors would the child also be expected to exhibit? A) Showing no interest in what the nurse sees in her ears B) Explaining what is right and what is wrong C) Demonstrating independence from her mother D) Showing no concern when the nurse hurts her own finger 10. The nurse is assessing the gross motor skills of an 8-year-old boy. Which of the following interview questions would facilitate this assessment? A) "Do you like to do puzzles?" B) "Do play any instruments?" C) "Do you participate in any sports?" D) "Do you like to construct models?" 11. The school nurse is conducting vision screening for a 7-year-old girl and documents the condition "amblyopia." What would the nurse tell the parents about this condition? A) "Amblyopia is an uncorrected refractive error of the eye." B) "Amblyopia is reduced vision in an eye that has not been adequately used during early development." C) "Amblyopia is a malalignment of the eye, which occurs at birth." D) "Amblyopia is a clouding of the lens of the eye caused by trauma to the eye." 12. The school nurse knows that school-age children are developing metalinguistic awareness. Which of the following is an example of this skill? A) The child enjoys reading books. B) The child enjoys conversations with peers. C) The child enjoys speaking on the phone. D) The child enjoys telling jokes. 13. A mother brings her 6-year-old son in for a check-up because the child is complaining of stomachaches. It is the beginning of the school year. Which of the following might the mother also mention? A) The child cries before going to school. B) The child made friends the first day of school. C) The child fights with siblings more often. D) The child loves the crowds in the lunchroom. 14. The nurse is teaching the parents of a 9-year-old girl about the socialization that is occurring in their child through school contacts. Which of the following information would the nurse include in her teaching plan? A) Teachers are the most influential people in the development of the school-age child's social network. B) Continuous peer relationships provide the most important social interaction for school-age children. C) Parents should establish norms and standards that signify acceptance or rejection. D) A characteristic of school-age children is their formation of groups with no rules and values involved. 15. During a well-child check-up, the parents of a 9-year-old boy tell the nurse that their son's friends told him that soccer is a stupid game, and now he wants to play baseball. Which comment by the nurse best explains the effects of peer groups? A) “The child's best friends will continue playing soccer.” B) “The children will cheer for each other regardless of the sport being played.” C) “Your child will rarely talk to you about his friends.” D) “Acceptance by friends, especially of the same sex, is very important at this age.” 16. The nurse is performing a physical examination of an 11-year-old girl. Which of the following observations would be expected? A) The child has not gained weight since last year. B) The child has grown 3 inches since last year. C) The child breathes abdominally. D) The child's third molars are about to erupt. 17. Which of the following would the nurse most likely find in a 10-year-old child in the period of concrete operational thought? A) Participation in abstract thinking B) Ability to classify similar objects C) Problem solving via the scientific method D) Ability to make independent decisions 18. After teaching the parents of a 9-year-old girl about safety, which statement indicates the need for additional teaching? A) “She can ride in the front seat of the car once she is 10 years old.” B) “We need to buy her a helmet so she can ride her scooter.” C) “She should ride her bike with the traffic on the side of the road.” D) “We signed her up for swim lesions at the local community center.” 19. The nurse is teaching parents to plan nutritional meals for their son who is overweight. Which of the following guidelines might the nurse include in the teaching plan? A) School-age children with an average body weight of 20 to 35 kg need approximately 90 calories per kilogram daily. B) The average water requirement for a school-age child per 24 hours ranges from 2,000 to 2,500 mL per day. C) The school-age child needs 28 g of protein and 800 mg of calcium for maintenance of growth and good nutrition. D) In the school-age child, calories needed to sustain weight increase, while the appetite decreases. 20. The nurse has determined that an 8-year-old girl is at risk for being overweight. Which of the following interventions would be a priority prior to developing the care plan? A) Determining the need for additional caloric intake B) Asking the parents who they want to work with the child C) Interviewing the parents about their eating habits D) Discussing the influence of peers on the child's diet 21. The parents of an 11-year-old child ask the nurse for suggestions to promote good nutrition for their child. Which response by the nurse would be most appropriate? A) “Be sure to limit protein to one meal every day.” B) “Use whole-grain or enriched breads and cereals.” C) “Have eggs on the average of once a week.” D) “Eat dark green leafy vegetables about twice a week.” 22. The school nurse is preparing a talk on the influence of the media on school-age children to present at the next PTO meeting. Which of the following facts might the nurse include in the introduction? A) Children in the United States spend about 6 hours a day either watching TV or playing video games. B) A child will see 2,000 murders by the end of grade school and 20,000 commercials a year. C) A school-age child cannot determine what is real from what is fantasy; therefore, TV and video games can lead to aggressive behavior. D) Parents should limit television watching and video-game playing to 2 hours per day. 23. The mother of a 7-year-old girl tells the school nurse that her child is deathly afraid of going to school. What would be the best intervention the nurse could suggest in this situation? A) Return the child to school and investigate the cause of the fear. B) Have the child stay home from school until any issues causing this fear are resolved. C) Investigate a new school for the child to attend that the child will not be afraid of. D) Tell the child that privileges will be taken away if she does not return to school. 24. Two working parents are discussing with the school nurse the possibility of their 12- yearold girl going home alone after school. Which of the following suggestions should the nurse make? A) Provide entertainment until the parents come home. B) Allow the child to go to a friend's house. C) Teach her how to take a message if someone calls. D) Purchase caller ID for the phone. 25. The parents of an 8-year-old boy are interested in promoting learning through reading to their son. Which of the following suggestions by the nurse would best promote this goal? A) Have the parents choose what he should read initially. B) Tell the child to read instead of watching TV with his parents. C) Tell the parents that reading is for the child to do by himself. D) Take the child to the library to check out some books. 26. The nurse is caring for a 7-year-old girl who is scheduled for a hernia repair and is very scared. Which of the following fears would she also most likely have at this age? A) Fear of being kidnapped B) Fear of cutting her finger C) Fear of sudden loud noises D) Fear of the neighbor's dog 27. The nurse is counseling the parents of a 10-year-old child who was caught stealing at school. Which of the following topics should the nurse cover? A) Having the child return the property in front of his or her class B) Discussing ways for the child to save face C) Finding out what is currently going on at home D) Reminding the child daily that stealing is wrong 28. When providing anticipatory guidance to a group of parents with school-aged children, which of the following would the nurse describe as the most important aspect of social interaction? A) School B) Peer relationships C) Family D) Temperament 29. The school nurse is teaching parents about the effects of bullying on school children. Which of the following accurately describes this developmental concern? A) Children who bully are those who report themselves as being lonely and having difficulty in forming friendships. B) Children who are bullied are reported to have low self-esteem, poor grades, and poor interpersonal skills. C) In general, about 20% of all children attending school are frightened and afraid most of the day. D) Both boys and girls are bullied; boys usually bully boys and use force more often. 30. The nurse is providing anticipatory guidance for parents of a school-age child on teaching the dangers of drugs and alcohol. Which of the following advice might be helpful for these parents? A) School-age children are not ready to absorb information that deals with drugs and alcohol. B) School-age children can think critically to interpret messages seen in advertising, media, and sports. C) Parents must prevent their child from being exposed to messages that are in conflict with their values. D) Discussions with children need to be based on facts and focused on the past and future. Answer Key 1. B 2. A 3. D 4. B, C, E, F 5. C 6. A 7. A, B, C, F 8. A 9. B 10. C 11. B 12. D 13. A 14. B 15. D 16. B 17. B 18. A 19. C 20. C 21. B 22. D 23. A 24. D 25. D 26. A 27. C 28. B 29. D 30. B Chapter 29- Growth and Development of the Adolescent 1. The nurse teaches parents of adolescents that adolescents need the support of parents and nurses to facilitate healthy lifestyles. Which of the following should be a priority focus of this guidance? A) Reducing risk-taking behavior B) Promoting adequate physical growth C) Maximizing learning potential D) Teaching personal hygiene routines 2. The nurse has seen a 15-year-old girl and a 16-year-old boy during health surveillance visits. Which of the following physical characteristics would be seen in both teenagers? A) Decreased respiratory rates of 15 to 20 breaths per minute B) Eruption of last four molars C) Increased shoulder, chest, and hip widths D) Fully functioning sweat and sebaceous glands 3. The nurse is performing an assessment of the reproductive system of a 17-year-old girl. Which of the following would alert the nurse to a developmental delay in this girl? A) Areola and papilla separate from the contour of the breast B) Mature distribution and coarseness of pubic hair C) Developed breast tissue D) Occurrence of first menstrual period 4. The school nurse is performing health assessments on students in middle school. Of which of the following developmental milestones should the nurse be aware? A) Height in girls increases rapidly after menarche and usually ceases immediately after menarche. B) Boys' growth spurt usually begins between the ages of 8 and 14 years and ends between the ages of 131/2 and 171/2 years. C) Peak height velocity (PHV) occurs at approximately 12 years of age in girls or about 6 to 12 months after menarche. D) Boys reach PHV and peak weight velocity (PWV) at about 16 years of age. 5. After assessing a 10-year-old girl, the nurse documents the appearance of breast buds, identifying this as which of the following? A) Menarche B) Thelarche C) Puberty D) Tanner stage 5 6. When describing the various changes that occur in organ systems during adolescence, which of the following would the nurse include? A) Significant increase in brain size B) Ossification completed later in girls C) Decrease in heart rate D) Decrease in activity of sebaceous glands 7. The school nurse is performing a physical examination on a 13-year-old boy who is on the soccer team. Which of the following is a physical quality that develops during these early adolescent years? A) Coordination B) Endurance C) Speed D) Accuracy 8. Based on Erikson's developmental theory, which of the following is the major developmental task of the adolescent? A) Gaining independence B) Finding an identity C) Coordinating information D) Mastering motor skills 9. The nurse assesses the spirituality of an adolescent. Which of the following are normal moral and spiritual milestones in this age group? Select all answers that apply. A) Adolescents will base their actions on the avoidance of punishment and the attainment of pleasure. B) Adolescents develop their own set of morals and values and question the status quo. C) Adolescents undergo the process of developing their own set of morals at different rates. D) Adolescents are more interested in the spiritualism of their religion than in the actual practices of their religion. E) Adolescents can understand the concepts of right and wrong and are developing a conscience. F) Adolescents are able to understand and incorporate into their behavior the concept of the “golden rule.” 10. The school nurse is conducting a seminar for parents of adolescents on how to communicate with teenagers. Which of the following guidelines might the nurse recommend? Select all answers that apply. A) Talk face to face and be aware of body language. B) Ask questions to see why he or she feels that way. C) Do not give praise unless the adolescent deserves it. D) Speak to your child as an authority figure, not an equal. E) Don't admit that you make mistakes. F) Don't pretend you know all the answers. 11. The nurse is teaching the parents of a 12-year-old boy about appropriate approaches when raising an adolescent. Which of the following comments should be included in the discussion? A) “Find out if his friends are worthy of him.” B) “Try to be open to his views.” C) “Maintain a firm set of rules.” D) “Remind him that he is still your little boy.” 12. The mother of a 14-year-old girl complains to the nurse that her daughter is moody, shuts herself in her room, and fights with her younger sister. Which of the following comments is most valuable to the mother? A) “Calmly talk to her about your concerns.” B) “This is normal for her age.” C) “She may be hanging with a bad crowd.” D) “Set some rules for family etiquette.” 13. The adolescent continues to develop self-concept and self-esteem. Which of the following is most important to a teen's self-esteem? A) Strong authority figures B) Spirituality C) Morals and values D) Body image 14. The nurse is performing risk assessments on adolescents in the school setting. Which one of the following teens should the nurse screen for hypertension? A) An Asian female B) A white male C) An African American male D) A Jewish male 15. The nurse knows that barriers to the adolescent's health and successful achievement of the tasks of adolescence exist. Which of the following is the major barrier to health for this population? A) Cultural B) Socioeconomic C) Marital status D) Racial 16. The nurse teaching safety to teens knows that which of the following is the leading cause of death among adolescents? A) Drowning B) Poisoning C) Diseases D) Unintentional injuries 17. When assessing adolescents for health risks, the nurse must keep in mind the factors related to the prevalence of adolescent injuries. Which of the following accurately describe these factors? Select all answers that apply. A) Increased physical growth B) Insufficient psychomotor coordination C) Tiredness, lack of energy D) Lack of impulsivity E) Peer pressure F) Inexperience 18. The nurse is helping the parents and their underweight adolescent collaborate on planning a healthy menu. Of which of the following nutritional requirements of adolescents should the nurse be aware? A) Teenagers have a need for increased calories, zinc, calcium, and iron for growth. B) Teenage girls who are active require about 1,800 calories per day. C) Teenage boys who are active require between 2,000 and 2,500 calories per day. D) Adolescents require about 1,000 to 1,200 mg of calcium each day. 19. The nurse is promoting nutrition to a 13-year-old boy who is overweight. Which of the following comments should the nurse expect to include in the discussion? A) “You need to go on a low-fat diet.” B) “Eat what your parents eat.” C) “Go out for a sport at school.” D) “Keep a food diary.” 20. The nurse is providing suggestions to a female adolescent about foods to help meet her nutritional requirements for iron. Which food would the nurse suggest as a good source of iron? A) Broccoli B) Yogurt C) Peanut butter D) White beans 21. During a health maintenance visit, a 15-year-old girl mentions that she is not happy with how overweight she is. Which of the following approaches is best for the nurse to take? A) “Good observation. Let's talk about diet and exercise.” B) “Don't worry; you are within the weight and height guidelines.” C) “What specifically have you been noticing?” D) “Tell me about your parents. Are they overweight?” 22. The school nurse knows that dating is a milestone for adolescents. Which of the following statements accurately describes a trend in teen dating? A) Most late adolescents spend more time in activities with mixed-sex groups, such as dances and parties, than they do dating as a couple. B) Most teens have been involved in at least one romantic relationship by middle adolescence. C) Teens that date frequently report slightly lower levels of self-esteem and decreased autonomy. D) Homosexual behavior as a teen usually indicates that the adolescent will maintain a homosexual orientation. 23. A mother calls the school nurse and is concerned because her 13-year-old daughter's friends wear heavy makeup and black clothes. Which of the following is the best advice for the mother? A) “This can lead to piercings and tattoos.” B) “The teen years are a time for experimenting.” C) “Encourage her to socialize with the kids at church.” D) “Teen appearance might not accurately reflect their actual values.” 24. During a health check-up without his parents, a 17-year-old tells the nurse he is gay. Which of the following approaches should the nurse take? A) “Tell me what makes you think you are gay.” B) “This puts you in an at-risk category.” C) “We need to talk about safe sex.” D) “You're not gay; you're confused.” 25. The nurse is promoting learning and school attendance to an 11-year-old girl. Which of the following factors will affect the child's attitude most? A) Her parents' values and desires B) The dramatic changes to her body C) Peer group behaviors and attitudes D) Desire for attention from boys 26. The school nurse is planning to teach a segment on smoking during the freshman health classes. The nurse is aware that this needs to be a forum rather than a lecture. Which of the following techniques will also help deliver a “don't smoke” message? A) Showing a command of the facts on smoking B) Speaking with a tone of authority C) Keeping your personal experiences out of it D) Listening to all comments nonjudgmentally 27. The school nurse is preparing a program on sexuality and birth control for a class of 14- to 16-year-olds. Which of the following behaviors will have the most influence on how the information is presented? A) Teens are adjusting to new body images. B) Adolescents tend to take risks. C) Teenagers are able to think in the abstract. D) Adolescents understand that actions have consequences. 28. The nurse is preparing a class for a group of adolescents about promoting safety. Which of the following would the nurse plan to include as the leading cause of adolescent injuries? A) Car accidents B) Firearms C) Water D) Fires 29. The nurse is discussing ways to promote discipline with parents who are becoming increasingly frustrated with their teenager. Which of the following would the nurse identify as most important? A) Establish rules and expectations. B) Collaborate to determine consequence. C) Make your responses consistent. D) Explain the rules to the adolescent. 30. The school nurse is teaching parents risk factors for suicide in adolescents. Which of the following would the nurse discuss? Select all answers that apply. A) Mental health changes B) History of previous suicide attempt C) Higher socioeconomic status D) Greatly improved school performance E) Family disorganization F) Substance abuse Answer Key 1. A 2. C 3. D 4. C 5. B 6. C 7. B 8. B 9. B, C, D 10. A, B, F 11. B 12. A 13. D 14. C 15. B 16. D 17. A, B, E, F 18. A 19. D 20. C 21. C 22. B 23. D 24. A 25. C 26. D 27. B 28. A 29. C 30. A, B, E, F Chapter 30- Atraumatic Care of Children and Families 1. The nurse is providing atraumatic care to children in a hospital setting. Which of the following are principles of this philosophy of care? Select all answers that apply. A) Avoid or reduce painful procedures B) Avoid or reduce physical distress C) Minimize parent–child interactions D) Provide child-centered care E) Minimize child control F) Use core primary nursing 2. The nurse is consulting with a child life specialist (CLS) to help minimize the stress of hospitalization for a child. Which of the following services would the CLS provide? Select all answers that apply. A) Medical preparation for tests, surgeries, and other medical procedures B) Support before and after, but not during, medical procedures C) Activities to support normal growth and development D) Grief and bereavement support E) Emergency room interventions for children and families F) Only inpatient consultations with families 3. The nurse is implementing interventions to prevent physical stressors for a 9-year-old child receiving chemotherapy in the hospital. Which of the following is an example of using atraumatic care for this child? A) Use restraint or "holding down" of the child during the procedure to prevent injury. B) Have the parent stand near and/or rub the child's feet during the procedure. C) Insert a saline lock if the child will require multiple doses of parenteral medications. D) Avoid using numbing techniques for multiple blood draws or IV insertion. 4. The nurse contacts a child life specialist (CLS) to work with children on a pediatric ward. What is the primary goal of the CLS? A) Decrease anxiety and fear during hospitalization and painful procedures B) Keep children who are hospitalized distracted from pain C) Perform medical procedures using atraumatic principles D) Act as a liaison between the nurse and the child 5. The nurse is preparing a child and his family for a lumbar puncture. Which of the following would be a primary intervention instituted by the CLS to keep the child safe? A) Distraction methods B) Stimulation methods C) Therapeutic hugging D) Therapeutic touch 6. The child life specialist (CLS) is preparing a 6-year-old child for a magnetic resonance imaging (MRI) scan. Which of the following statements reflects the use of atraumatic principles when explaining the procedure? A) "You will be taken to a magnetic resonance imaging machine for an x-ray of your liver." B) "You may hear some loud noises when you are lying in the machine, but they won't hurt you." C) "You have nothing to worry about; the MRI machine is safe and will not cause you any pain." D) "Let's just get you to the x-ray department for your test and you'll see how simple it is." 7. The nurse uses family-centered care to care for children in a pediatric office. Upon what concept is family-centered care based? A) The family is the constant in the child's life and the primary source of strength. B) The care provider is the constant in the child's life and the primary source of strength. C) The child must be prepared to be his or her own source of strength during times of crisis. D) The wishes of the family should direct the nursing care plan for the child. 8. A nurse is promoting the use of family-centered care in a local community clinic. Which of the following are advantages or disadvantages of this type of care provision? Select all answers that apply. A) Recovery times are longer. B) Anxiety is decreased. C) Communication is improved. D) Health care costs are increased. E) Pain management is enhanced. F) More health care resources are utilized. 9. The nurse knows that effective communication with children and their parents is critical to providing atraumatic quality nursing care. Which of the following statements accurately describes the communication patterns of children? A) Communication patterns are similar from one child to the next. B) Children often use more words than adults to describe their fears. C) Children rely more on nonverbal communication and silence. D) Parents more often require affective communication rather than neutral communication. 10. The nurse is using verbal skills to explain the nursing care plan to parents of a 10-year- old child with cancer. Which of the following describes a guideline the nurse should follow to provide appropriate verbal communication? A) Use closed-ended questions that do not restrict the child's or parent's answers. B) Allow the focus to change without redirecting the conversation. C) Restate the child's and parents comments in your own words. D) Paraphrase the child's or parent's feelings to demonstrate empathy. 11. The nurse is incorporating nonverbal communication with verbal communication when explaining the treatment plan for a child with juvenile diabetes. Which of the following should the nurse do to communicate effectively with this family? A) Relax; maintain an open posture, with the arms crossed. B) Sit opposite the family and lean forward slightly. C) Use eye contact sparingly to avoid embarrassment. D) Speak a verbal yes or no; do not use head nods. 12. The nurse is teaching the student nurse how to communicate effectively with children. Which one of the following methods would the nurse recommend? A) Position self above the child's level to denote authority. B) If possible, communicate with the child apart from the parent. C) Direct questions and explanations to the child. D) Use the medical terms for body parts and medical care. 13. The nurse is implementing care for a hospitalized toddler. What communication technique would the nurse use with the child to reflect the child's developmental level? A) Allow the child extra time to complete thoughts. B) Communicate solely through play. C) Provide simple but honest and straightforward responses. D) Remain nonjudgmental to avoid alienation. 14. The nurse is caring for a 4-year-old boy with Ewing sarcoma who is scheduled for a computed axial tomography (CAT) scan tomorrow. Which of the following is the best example of therapeutic communication? A) Telling him he will get a shot when he wakes up tomorrow morning B) Telling him how cool he looks in his baseball cap and pajamas C) Using family-familiar words and soft words when possible D) Describing what it is like to get a CAT scan using words he understands 15. The nurse is caring for a 14-year-old boy with an osteosarcoma. Which of the following communication techniques would be least effective for him? A) Letting him choose juice or soda to take pills B) Seeking the teenager's input on all decisions C) Discussing the benefits of chemotherapy with him D) Avoiding undue criticism of noncompliance 16. The nurse is educating a 16-year-old girl who has just been diagnosed with acute myelogenous leukemia. Which of the following statements best demonstrates therapeutic communication? A) Discussing the treatment plan in detail for the next few weeks B) Using medical terms when describing the disease C) Assessing the adolescent's emotional status in private D) Talking about clothing and the stores where she shops 17. The nurse is explaining a discharge plan to the parents of an infant being discharged from the hospital. Which of the following characteristics regarding adult learning should the nurse incorporate into her plan? A) Adults are dependent learners. B) Adults are problem focused. C) Adults are future focused. D) Adults do not value past learning. Answer Key 1. A, B, F 2. C, D, E 3. C 4. A 5. C 6. B 7. A 8. B, C, E 9. C 10. D 11. B 12. C 13. A 14. D 15. A 16. C 17. B Chapter 31- Health Supervision 1. The nurse is caring for children in a physician's office where health supervision is practiced. Which of the following is a key focus of health supervision? Select all answers that apply. A) Making referrals for all health care needs B) Monitoring disease incidence C) Optimizing level of functioning D) Monitoring quality of care provided E) Teaching parents to prevent injury F) Providing care developed from national guidelines 2. The nurse is providing care for children in a pediatric medical home. Which of the following is a characteristic of care in these types of facilities? A) All insurance except Medicaid is accepted. B) Ambulatory care is not provided C) A centralized database contains all child information. D) Continuity of care is provided from infancy through adulthood. 3. The nurse is preparing a presentation to a local parent group about pediatric health supervision. Which of the following would the nurse emphasize as the focus? A) Injury prevention B) Wellness C) Health maintenance D) Developmental surveillance 4. A large portion of the nurse's efforts is dedicated to health supervision for children who use the facility as their primary medical contact. At which of the following facilities does the nurse work? A) An urgent care center B) A pediatric practice C) A mobile outreach immunization program D) A dermatology practice 5. The nurse strives to provide culturally competent care for children in a health clinic that follows the principles of health supervision. Which of the following nursing actions reflects this type of care? A) The nurse treats all children the same regardless of their culture. B) The nurse negotiates a care plan with the child and family. C) The nurse researches the child's culture and provides care based on the findings. D) The nurse provides future-based care for culturally diverse children. 6. The nurse is aware that the community affects the health of its members. Which of the following statements accurately reflect a community influence of health care? Select all answers that apply. A) A community can be a contributor to a child's health or be the cause of his or her illnesses. B) The child's health should be separated from the health of the surrounding community. C) Community support and resources are necessary for children with significant problems. D) Poverty has not been linked to an increase in health problems in communities. E) The breakdown of community and family support systems can lead to depression and violence. F) Ideally, the child's medical home is located outside the community. 7. The nurse is conducting a psychosocial assessment of a child with asthma brought to the physician's office for a check-up. Which of the following are psychosocial issues that might be assessed? Select all answers that apply. A) Health insurance coverage B) Transportation to health care facilities C) School's response to the chronic illness D) Past medical history E) Future treatment plans F) Health maintenance needs 8. The nurse is examining a 2-year-old child who was adopted from Guatemala. Which of the following would be a priority screening for this child? A) Screening for congenital defects B) Screening for abuse C) Screening for childhood illnesses D) Screening for infectious diseases 9. The mother of a 5-year-old child with eczema is getting a check-up for her child before school starts. Which of the following will the nurse do during the visit? A) Change the bandage on a cut on the child's hand B) Assess the compliance with treatment regimens C) Discuss systemic corticosteroid therapy D) Assess the child's fluid volume 10. During the health history, the mother of a 4-month-old child tells the nurse she is concerned that her baby is not doing what he should be at this age. What is the nurse's best response? A) “I'll be able to tell you more after I do his physical.” B) “Fill out the questionnaire and then I can let you know.” C) “Tell me what concerns you.” D) “All mothers worry about their babies. I'm sure he's doing well.” 11. A 3-year-old child is scheduled for a hearing screening. The nurse would prepare the child for screening by which method? A) Auditory brain stem response B) Evoked otoacoustic emissions C) Visual reinforcement audiometry D) Conditioned play audiometry 12. A 2-week-old child responds to a bell during an initial health supervision examination. The child's records do not show that a newborn hearing screening was done. Which of the following is the best action for the nurse to take? A) Do nothing because responding to the bell proves he does not have a hearing deficit. B) Immediately schedule the infant for a newborn hearing screening. C) Ask the mother to observe for signs that the infant is not hearing well. D) Screen again with the bell at the 2-month-old health supervision visit. 13. The nurse is performing developmental surveillance for children at a medical home. Which of the following infants are most at risk for developmental delays? Select all answers that apply. A) A child whose birthweight was 1,600 g B) A child whose parent has a mental illness C) A child raised by a single parent D) A child with a lead level above 10 mg/dL E) A child with hypertonia or hypotonia F) A child with gestational age more than 33 weeks 14. The nurse is examining a 15-month-old child who was able to walk at the last visit and now can no longer walk. What would be the nurse's best intervention in this case? A) Schedule a full evaluation since this may indicate a neurologic disorder. B) Note the regression in the child's chart and recheck in another month. C) Document the findings as a developmental delay since this is a normal occurrence. D) Ask the parents if they have changed the child's schedule to a less active one. 15. During a physical assessment of a 5-month-old child, the nurse observes the first tooth has just erupted and uses the opportunity to advise the mother to schedule a dental examination for her baby. Which of the following is the correct time for the dentist visit? A) By the first birthday B) By the second birthday C) By entry into kindergarten D) By entry into first grade 16. A mother and her 4-week-old infant have arrived for a health maintenance visit. Which of the following activities will the nurse perform? A) Assess the child for an upper respiratory infection B) Take a health history for a minor injury C) Administer a varicella injection D) Plot the child's head circumference on a growth chart 17. The nurse is screening a 6-year-old child for mental ability. Which of the following tests would the nurse use to assess intelligence? A) Denver Articulation Screening B) Denver PRQ C) Goodenough-Harris Drawing Test D) Parents' Evaluation of Developmental Status (PEDS. 18. When assessing the vision of a 2-month-old, the nurse would use which of the following? A) Black-and-white checkerboard B) Red and blue circles C) Gray and blue animal drawings D) Green and yellow letters 19. The nurse is performing a risk assessment of a 5-year-old and determines the child has a risk factor for cystic fibrosis. What type of screening would the nurse perform to confirm or rule out this disease? A) Universal screening B) Selective screening C) Hyperlipidemia screening D) Developmental screening 20. The nurse is caring for an infant who had hyperbilirubinemia requiring exchange transfusion. Based on this information, this infant is at risk for what type of disorder? A) Vision loss B) Hearing loss C) Hypertension D) Hyperlipidemia 21. The nurse is performing a vision screening for a 4-year-old child. Which of the following screening charts would be best for determining the child's visual acuity? A) Snellen B) Ishihara C) Allen figures D) Color Vision Testing Made Easy (CVTME) 22. The nurse is explaining the difference between active and passive immunity to the student nurse. Which of the following statements accurately describes a characteristic of the process of immunity? A) Active immunity is produced when the immunoglobulins of one person are transferred to another. B) Passive immunity can be obtained by injection of exogenous immunoglobulins. C) Active immunity can be transferred from mothers to infants via colostrum or the placenta. D) Passive immunity is acquired when a person's own immune system generates the immune response. 23. The nurse is administering a hepatitis B vaccine to a child. What is the classification of this type of vaccine? A) Killed vaccines B) Toxoid vaccines C) Conjugate vaccines D) Recombinant vaccines 24. A 15-month-old girl is having her first health visit at a clinic. The mother has no immunization record but says the child was immunized 3 months ago at the local health department. Which of the following is the best action for the nurse to take? A) Ask the mother to bring the records to the next health maintenance visit. B) Start the catch-up schedule since there are no immunization records. C) Keep the child at the facility while the mother returns home for the records. D) Call the local health department and verify the child's immunization status. 25. The nurse is discussing vaccination for Haemophilus influenzae type B (Hib) with the mother of a 6-month-old child. Which of the following comments provides the most compelling reason to get the vaccination? A) “These bacteria live in every human.” B) “Young children are especially susceptible to these bacteria.” C) “You have a choice of two excellent vaccines.” D) “Your child needs this final dose for protection.” 26. The mother of a 15-month-old child is questioning the nurse about the need for the hepatitis B vaccination. Which of the following comments provides the most compelling reason for the vaccine? A) “The most common side effect is injection site soreness.” B) “This is a recombinant or genetically engineered vaccine.” C) “Immunizations are needed to protect the general population.” D) “This protects your child from infection that can cause liver disease.” 27. When preparing to administer the polio vaccine to an infant, the nurse would expect to administer the vaccine by which route? A) Intramuscular B) Subcutaneous C) Oral D) Intradermal 28. After teaching the mother about follow-up immunizations for her daughter, who received the varicella vaccine at age 14 months, the nurse determines that the teaching was successful when the mother states that a follow-up dose should be given at which time? A) When the child is 20 to 36 months of age B) When the child is 4 to 6 years of age C) When the child is 11 to 12 years of age D) When the child is 13 to 15 years of age 29. The nurse working in a community clinic attempts to establish a free vaccination program to refer low-income families. What is the key strategy for success when implementing a health promotion activity? A) Partnership development B) Funding for projects C) Finding an audience D) Adequate staffing 30. The nurse is providing anticipatory guidance to an obese teenager. Which of the following interventions would be most likely to promote healthy weight in teenagers? A) Make the focus of the program weight centered. B) Begin directly advising children about their weight at age 6. C) Focus physical activity on competitive sports and activities. D) Obtain nutritional histories directly from the school-age child and adolescent. Answer Key 1. C, E, F 2. C 3. B 4. B 5. B 6. A, C, E 7. A, B, C 8. D 9. B 10. C 11. D 12. B 13. B, C, E 14. A 15. A 16. D 17. C 18. A 19. B 20. B 21. C 22. B 23. D 24. D 25. B 26. D 27. B 28. B 29. A 30. D Chapter 32- Health Assessment of Children 1. The nurse is conducting a health history for a 9-year-old child with stomach pains. Which of the following is a recommended guideline when approaching the child for information? A) Wear a white examination coat when conducting the interview. B) Allow the child to control the pace and order of the health history. C) Use quick deliberate gestures to get your point across. D) Do not make physical contact with the child during the interview. 2. For which of the following children would the nurse conduct an immediate comprehensive health history? A) A child who is brought to the emergency room with lacerations B) A child who is a new client in a pediatric office C) A child who is a routine client and presents with signs of a sinus infection D) A child whose condition is improving 3. The nurse is performing a health history on a 6-year-old boy who is having trouble adjusting to school. Which of the following questions would be most likely to elicit valuable information? A) "Do you like your new school?" B) "Are you happy with your teacher?" C) "Do you enjoy reading a book?" D) "What are your new classmates like?" 4. The nurse performing a health history on a child asks the parents if their child has experienced increased appetite or thirst. What body system is the nurse assessing with this question? A) Endocrine B) Genitourinary C) Hematologic D) Neurologic 5. The nurse is questioning the parents of a 2-year-old child to obtain a functional history. Which of the following topics might the nurse include? Select all answers that apply. A) The child's toileting habits B) Use of car seats and other safety measures C) Problems with growth and development D) Prenatal and perinatal history E) The child's race and ethnicity F) Use of supplements and vitamins 6. The nurse is conducting a physical examination of a child following a comprehensive health history. What should be the focus of the physical examination? A) The child B) The parents C) Chief complaint D) Developmental age 7. The nurse is teaching the student nurse how to perform a physical assessment based on the child's developmental stage. Which of the following statements accurately describes a recommended guideline for setting the tone of the examination for a school-age child? A) Keep up a running dialogue with the caregiver, explaining each step as you do it. B) Include the child in all parts of the examination; speak to the caregiver before and after the examination. C) Speak to the child using mature language and appeal to his or her desire for self- care. D) Address the child by name; speak to the caregiver and do the most invasive parts last. 8. Which of the following would be least effective in gaining the cooperation of a toddler during a physical examination? A) Tell the child that another child the same age wasn't afraid. B) Allow the child to touch and hold the equipment when possible. C) Permit the child to sit on the parent's lap during the examination. D) Offer immediate praise for holding still or doing what was asked. 9. The nurse is performing a physical examination on a sleeping newborn. Which of the following body systems should the nurse examine last? A) Heart B) Abdomen C) Lungs D) Throat 10. The nurse is teaching the student nurse the sequence for performing the assessment techniques during a physical examination. What is the appropriate order? A) Inspection, palpation, percussion, auscultation B) Inspection, percussion, palpation, auscultation C) Palpation, percussion, inspection, auscultation D) Inspection, auscultation, palpation, percussion 11. The nurse is examining the posture of a male toddler and notes the condition "lordosis." What would be the appropriate reaction of the nurse to this finding? A) Explain that the child will need a back brace. B) Refer the toddler to a physical therapist. C) Do nothing; this is a normal condition for toddlers. D) Notify the primary care physician about the condition. 12. The nurse is assessing the temperature of a diaphoretic toddler who is crying and being uncooperative. What would be the best method to assess temperature in this child? A) Oral thermometer B) Axillary method C) Temporal scanning D) Rectal route 13. The nurse is preparing to take a tympanic temperature reading of a 4-year-old. In order to get an accurate reading, what does the nurse need to do? A) Pull the earlobe back and down B) Direct the infrared sensor at the tympanic membrane C) Pull the earlobe down and forward D) Remove any visible cerumen from inside the ear canal 14. A mother brings her 31/2-year-old daughter to the emergency department because the child has been vomiting and having diarrhea for the past 36 hours. When assessing this child's temperature, which method would be least appropriate? A) Oral B) Tympanic C) Rectal D) Axillary 15. The nurse is assessing heart rate for children on the pediatric ward. Which of the following is a normal finding based on developmental age? A) An infant's rate is 90 bpm. B) A toddler's rate is 150 bpm. C) A preschooler's rate is 130 bpm. D) A school-age child's rate is 50 bpm. 16. The nurse is assessing the heart rate of a healthy 13-month-old child. The nurse knows to auscultate which of the following sites to obtain an accurate assessment? A) Radial pulse B) Brachial pulse C) Apical pulse at the third or fourth intercostal space D) Apical pulse at the fourth or fifth intercostal space at the midclavicular line 17. The nurse is assessing the heart rate of a healthy school-age child. The nurse expects that the child's heart rate will be in which of the following ranges? A) 80 to 150 bpm B) 70 to 120 bpm C) 65 to 110 bpm D) 60 to 100 bpm 18. The nurse is preparing to assess the pulse of an 18-month-old child. Which pulse would be most difficult for the nurse to palpate? A) Radial B) Brachial C) Pedal D) Femoral 19. While auscultating the heart of a 5-year-old child, the nurse notes a murmur that is soft and quiet and heard each time the heart is auscultated. The nurse documents this finding as which of the following? A) Grade 1 B) Grade 2 C) Grade 3 D) Grade 4 20. The parents of a 2-day-old girl are concerned because her feet and hands are slightly blue. How should the nurse respond? A) “Your daughter has acrocyanosis; this is causing her blue hands and feet.” B) “Let's watch her carefully to make sure she does not have a circulatory problem.” C) “This is normal; her circulatory system will take a few days to adjust.” D) “This is a vasomotor response caused by cooling or warming.” 21. A nurse is assessing the fontanels of a crying newborn and notes that the posterior fontanel pulsates and briefly bulges. What do these findings indicate? A) Increased intracranial pressure B) Overhydration C) Dehydration D) These are normal findings. 22. The nurse is assessing the neck of an 8-year-old child with Down syndrome. Which of the following findings would the nurse expect during the examination? A) Webbing B) Excessive neck skin C) Lax neck skin D) Shortened neck 23. The nurse is conducting a routine health assessment of a 3-month-old boy and notices a flat occiput. The nurse provides teaching and emphasizes the importance of tummy time. Which of the following responses by the mother indicates a need for further teaching? A) “He must be positioned on his tummy as much as possible.” B) “I need to watch him during his tummy time.” C) “I need to change his head position while he is in an upright chair.” D) “His head has flattened due to the pressure of his head position.” 24. The nurse is measuring the blood pressure of a 12-year-old boy with an oscillometric device. The boy's reading is greater than the 90th percentile for gender and height. What is the appropriate nursing action? A) Repeat the reading with the oscillometric device. B) Repeat the blood pressure reading using auscultation. C) Measure the blood pressure in all four extremities. D) Measure the blood pressure with a Doppler. 25. The nurse is inspecting the fingernails of an 18-month-old girl. Which of the following findings indicates chronic hypoxemia? A) Nails that curve inward B) Clubbing of the nails C) Nails that curve outward D) Dry, brittle nails 26. The nurse is using pulse oximetry to measure oxygen saturation in a 3-year-old girl. The nurse understands that falsely high readings may be associated with which situation or condition? A) A nonsecure connection B) Cold extremities C) Hypovolemia D) Anemia 27. Assessment reveals that a child weighs 73 lb and is 4 ft 1 in. tall. The nurse calculates this child's body mass index as: A) 19.1 B) 20.7 C) 21.4 D) 24.5 28. The nurse is teaching the student nurse about abnormal findings when assessing the breasts of children. Which of the following may be associated with renal disorders? A) Swollen nipples upon inspection of a newborn's breasts B) Tender nodule palpated under the nipple of a 10-year-old C) Observation of enlarged breast tissue in a male adolescent D) Observation of a supernumerary nipple along the mammary ridge 29. The nurse is inspecting the genitals of a prepubescent girl. Which of the following are normal signs of the onset of puberty? A) Appearance of pubic hair around 11 to 13 years old B) Swelling or redness of the labia minora C) Presence of a small amount of downy pubic hair D) Lesions on the external genitalia   Answer Key 1. B 2. B 3. D 4. A 5. A, B, F 6. C 7. B 8. A 9. D 10. A 11. C 12. B 13. B 14. C 15. A 16. C 17. D 18. A 19. B 20. C 21. D 22. C 23. A 24. B 25. B 26. D 27. C 28. D 29. A Chapter 33- Caring for Children in Diverse Settings 1. The nurse working in the emergency room monitors the admission of children. Statistically, for which one of the following disorders would children younger than 5 years most commonly be admitted? A) Mental health problems B) Injuries C) Respiratory disorders D) Gastrointestinal disorders 2. The nurse is caring for a 7-year-old girl hospitalized in isolation. The nurse notices that she has begun sucking her thumb and changing her speech patterns to those of a toddler. What condition is the girl manifesting? A) Regression B) Suppression C) Repression D) Denial 3. The nurse is caring for an 8-year-old boy hospitalized for a bone marrow transplant. His parents are in and out of his room throughout the day. Which of the following behaviors of the child would alert the nurse that he is in the second stage of separation anxiety? A) He ignores his parents when they return to his room. B) He cries uncontrollably whenever they leave. C) He forms superficial relationships with his caregivers. D) He sits quietly and is uninterested in playing and eating. 4. The nurse is caring for a 4-year-old girl who has been hospitalized for over a week with severe burns. Which of the following would be a priority intervention to help satisfy this preschool child's basic needs? A) Encourage friends to visit as often as possible. B) Suggest that a family member be present with her 24 hours a day. C) Explain necessary procedures in simple language that she will understand. D) Allow her to make choices about her meals and activities as much as permitted. 5. The nurse is caring for a hospitalized 13-year-old girl, who is questioning everything the medical staff is doing and is resistant to treatment. How should the nurse respond? A) “Let's work together to plan your day along with your treatments.” B) “The sooner you cooperate, the sooner you are going to leave.” C) “If you are more cooperative, perhaps we can arrange a visit from friends.” D) “Please don't make me call your parents about this.” 6. The nurse is caring for a 10-year-old boy who is in traction. The boy has a nursing diagnosis of deficient diversional activity related to confinement in bed that is evidenced by verbalization of boredom and lack of participation in play, reading, and schoolwork. Which of the following would be the best intervention? A) Offer the child reading materials. B) Enlist the aid of a child life specialist. C) Encourage the child to complete his homework. D) Ask for the parents' assistance. 7. The nurse is caring for a 13-year-old girl hospitalized for complications from type 1 diabetes. The girl has a nursing diagnosis of powerlessness related to lack of control of multiple demands associated with hospitalization, procedures, treatments, and changes in usual routine. How can the nurse help promote control? A) Ask the child to identify her areas of concern. B) Encourage participation of parents in care activities. C) Offer the girl as many choices as possible. D) Enlist the family's assistance in creating a time schedule. 8. The nurse is caring for an 8-year-old girl who requires numerous venipunctures and injections daily. The nurse understands that the child is exhibiting signs of sensory overload and enlists the assistance of the child life specialist. What should the therapeutic play involve to best deal with the child's stressors? A) Puppets and dolls B) Drawing paper and crayons C) Wooden hammer and pegs D) Sewing puppets with needles 9. After teaching a group of students about therapeutic play, the instructor determines that additional teaching is needed when the students identify which of the following as a characteristic? A) Focus on coping B) Use of a highly structured format C) Dramatization of emotions D) Expression of feelings 10. The mother of a hospitalized child reports that her daughter, who is having some difficulty eating, just had a 4-ounce cup of ice chips. The nurse documents this on the child's intake flow sheet as which of the following? A) 2 ounces B) 4 ounces C) 6 ounces D) 8 ounces 11. The nurse is preparing a nursing care plan for a child hospitalized for cardiac surgery. Which of the following are examples of interventions that nurses perform in the "building a trusting relationship" stage? Select all answers that apply. A) Gathering information about the child using the child's own toys B) Preparing the child for a procedure by playing games C) Explaining in simple terms what will happen during surgery D) Allowing the child to devise an exercise plan following surgery E) Praising the child for how well he is doing following instructions F) Giving the child a favorite toy to cuddle following a painful procedure 12. The nurse is preparing a hospitalized 7-year-old girl for a lumbar puncture. Which of the following actions would help reduce her stress related to the procedure? Select all answers that apply. A) Pretend to perform the procedure on her doll. B) Explain the procedure to her in medical terms. C) Do not allow her to see or touch the equipment. D) Teach her the steps of the procedure. E) Tell her not to pay attention to any sounds she might hear. F) Introduce her to the health care personnel. 13. The nurse is performing an admission of a 10-year-old boy. Which of the following actions will help the nurse establish a trusting and caring relationship with the child and his family? Select all answers that apply. A) The nurse should not minimize the child's fears by smiling. B) The nurse should initiate introductions. C) The nurse should not use formal titles at the introduction. D) The nurse should maintain eye contact at the appropriate level. E) The nurse should start communication with the child first and then move on to the family. F) The nurse should use age-appropriate communication with the child. 14. The nurse is caring for an 11-year-old girl preparing to undergo a magnetic resonance imaging (MRI) scan. Which of the following statements would best help prepare the girl for the test and decrease anxiety? A) “You won't hear a sound if you wear your headphones.” B) “The machine makes a very loud rattle; however, headphones will help.” C) “There are a variety of loud sounds you will hear.” D) “The MRI scanner sounds like a machine gun.” 15. The nurse is caring for a 10-year-old girl who is in an isolation room. Which of the following interventions would be a priority intervention for this child? A) Reduce noise as much as possible. B) Provide age-appropriate toys and games. C) Discourage visits from family members. D) Put on mask prior to entering the room. 16. An adolescent is scheduled for outpatient arthroscopic surgery on his knee next week. As part of preparing him for the procedure, which action would be most appropriate? A) Discussing the events with the adolescent and his mother upon arrival the morning of the procedure B) Providing detailed explanations of the procedure at least a week in advance of the procedure C) Encouraging the parent to stay with the adolescent as much as possible before the procedure D) Answering the adolescent's questions with simple answers, encouraging him to ask the surgeon 17. The nurse is providing developmentally appropriate care for a toddler hospitalized for observation following a fall down the steps. Which of the following measures might the nurse consider when caring for this child? Select all answers that apply. A) Use the en face position when holding the toddler. B) Use a bed for toddlers who have an adult present. C) Avoid leaving small objects that can be swallowed in the bed. D) Explain activities in concrete, simple terms. E) Allow the child to select meals and activities. F) Encourage parents to stay to prevent separation anxiety. 18. The nurse is ordered to apply restraints to a toddler who keeps pulling at the tubes in his arm. Which of the following criteria must occur to ensure proper use of these restraints? Select all answers that apply. A) The nurse must check the restraints every 15 minutes while they are in place. B) Secure the restraints with ties to the side rails, not the bed or crib frame. C) Assess the temperature of the affected extremities, pulses, and capillary refill every 15 minutes after placement. D) Use a clove-hitch type of knot to secure the restraints with ties. E) Remove the restraint every 2 hours to allow for range of motion and repositioning. F) Encourage parent participation, providing continuous explanations about the reasons and time frame for restraints. 19. The nurse is enlisting the parents' assistance for therapeutic hugging prior to an otoscopic examination. What should the nurse emphasize to the parents? A) “You will need to keep his hands down and his head still.” B) “If this does not work, we will have to apply restraints.” C) “If you are not capable of this, let me know so I can get some assistance.” D) “I may need you to leave the room if your son will not remain still.” 20. The nurse is caring for a 7-year-old boy who needs his left leg immobilized. What is the priority nursing intervention? A) Enlist the assistance of a child life specialist. B) Explain to the boy that he must keep his leg very still. C) Apply a clove-hitch restraint to the boy's left leg. D) Explain that a restraint will be applied if he cannot hold still. 21. The nurse is transporting a 6-month-old with a suspected blood disorder to the nursery. What is the most appropriate method of transporting the child by the nurse? A) A wagon with rails B) Cradle hold C) Football hold D) Over the shoulder 22. The nurse is caring for an immunosuppressed 3-year-old girl and is providing teaching to the mother about proper oral hygiene. Which of the following responses from the mother indicates a need for further teaching? A) “I really need to carefully check for skin breakdown.” B) “I must really scrub her teeth and gums well.” C) “I must use a soft toothbrush.” D) “I can use a soft gauze sponge to care for her gums.” 23. When preparing to apply a restraint to a child, which of the following would be most important for the nurse to do? A) Expect to keep the restraint on for at least 8 hours. B) Explain that safety, not punishment, is the reason for the restraint. C) Plan to use a square knot to secure the restraint to the side rails. D) Use a limb restraint rather than a jacket restraint for most issues. 24. The nurse is providing discharge planning for a 12-year-old boy with multiple medical conditions. Which of the following would be the best teaching method for this child and his family? A) Demonstrate the care and ask for a return demonstration. B) Provide and review educational booklets and materials. C) Provide a written schedule for the child's care. D) Provide a trial period of home care. 25. Community-based nursing provides opportunities that are quite different from acute care nursing. Which of the following job characteristics is unique to home care nursing? A) Experiencing a greater amount of independence B) Building a close relationship with the family C) Coordinating therapy services and reimbursements D) Focusing teaching on child independence 26. The nurse is working as a community health care nurse. What would be the nurse's focus when providing care of the child? A) Providing care to the individual and family in acute care settings B) Providing care to the indigent in family care settings C) Providing care in geographically and culturally diverse settings D) Providing care for particular age groups or particular diagnoses 27. The nurse working in community nursing uses epidemiology as a tool. What information can be obtained using this process? A) Health needs of a population B) Cultural needs of a population C) Income levels of a population D) Mortality rates of a population 28. When speaking to a group of parents at a local elementary school, the nurse describes school nursing as a specialized practice of nursing based on the fact that a healthy child has a better chance to succeed in school. Which of the following best describes the strategy school nurses use to achieve student success? A) They coordinate all school health programs. B) They link community health services. C) They work to minimize health-related barriers to learning. D) They promote student health and safety. 29. The nurse referring a child to home care discusses the advantages and disadvantages with the child's family. Which of the following are disadvantages of this method of health care? Select all answers that apply. A) The nurse is performing care of the child in the family's home. B) The home care nurse is not always equipped to perform technical care. C) The out-of-pocket cost of home care is more expensive. D) The technical procedures may be overwhelming for the family. E) The financial burden may cause more stress for the family. F) The child does not receive continuity of care provided in the hospital setting. 30. The nurse working with children in a hospital setting notes that they are being discharged earlier and earlier. Which of the following is a primary reason for this trend? A) Nursing shortages B) Increased funding for home care C) National health care initiatives D) Cost containment Answer Key 1. C 2. A 3. D 4. C 5. A 6. B 7. C 8. D 9. B 10. A 11. B, C 12. A, D, F 13. B, D, F 14. B 15. B 16. B 17. C, F 18. D, E, F 19. A 20. B 21. D 22. B 23. B 24. D 25. A 26. C 27. A 28. C 29. A, C, D, E 30. D Chapter 34- Caring for the Special Needs Child 1. The nurse is caring for medically fragile children in a hospital setting. What nursing role has the greatest impact on the child and family when caring for this population? A) Teacher B) Advocate C) Coordinator D) Caregiver 2. The nurse is teaching a group of parents with premature infants about the various medical and developmental problems that may occur. The nurse determines that additional teaching is needed when the group identifies which of the following as a problem? A) Sudden infant death syndrome B) Hydrocephalus C) Peptic ulcer D) Bronchopulmonary dysplasia 3. The nurse is caring for a toddler with special needs. Which of the following developmental tasks related to toddlerhood might be delayed in the child with special needs? A) Developing body image B) Developing peer relationships C) Developing language and motor skills D) Learning through sensorimotor exploration 4. The nurse is providing home care for a 1-year-old girl who is technologically dependent. Which intervention will best support the family process? A) Finding an integrated health program for the family B) Teaching modifications of the medical regimen for vacation C) Assessing family expectations for the special needs child D) Creating schedules for therapies and interventions 5. The nurse is caring for families with vulnerable child syndrome. Which of the following situations would be most likely to predispose the family to this condition? A) Having a postterm infant B) Having an infant who is reluctant to feed properly C) Having a child diagnosed with leukemia at age 10 D) Having a child with juvenile diabetes 6. A 7-year-old boy has reentered the hospital for the second time in a month. Which intervention is particularly important at this time? A) Assessing his parents' coping abilities B) Seeking his parents' input about their child's needs C) Educating his family about the procedure D) Notifying the care team about his hospitalization 7. The nurse is caring for a special needs infant. Which intervention will be most important in helping the child reach her maximum developmental potential? A) Directing her parents to an early intervention program B) Monitoring her progress in elementary school C) Serving on an individualized education program committee D) Preparing a plan for her to transition to college 8. The nurse is caring for a 4-year-old girl with special care needs in the hospital. Which intervention would have the most positive effect on this child? A) Taking her on an adventure down the hall B) Helping her do a simple craft project C) Introducing her to children in the playroom D) Limiting the staff providing care for her 9. The nurse is caring for infants having the condition failure to thrive (FTT). Which of the following infants would be at high risk for this condition? Select all answers that apply. A) A newborn baby with tetralogy of Fallot B) An infant with a cleft palate C) An infant born to a diabetic mother D) An infant born to an impoverished mother E) An infant with bronchopulmonary dysplasia F) An infant born to a teenage mother 10. The nurse is weighing an underweight infant diagnosed with failure to thrive (FTT) and notes that the baby does not make eye contact and is less active than the other infants. What would be a probable cause for the FTT related to the infant's body language? A) Congenital heart defect B) Cleft palate C) Gastroesophageal reflux disease D) Maternal abuse 11. Which of the following would be least appropriate to include in the discharge plan for a medically fragile child? A) Assisting with referrals for financial support B) Arranging for necessary care equipment and supplies C) Assessing the family's home environment D) Encouraging passive caregiving 12. The nurse is looking into the Individuals with Disabilities Education Improvement Act (IDEA) of 2004 to help provide resources for a client with multiple chronic diseases. Which of the following are mandates of this legislation? Select all answers that apply. A) The law mandates government-funded care coordination and special education for children up to 8 years of age. B) This early intervention program is a state-funded program run at the federal level. C) This federal law allows each state to define “developmental disability” differently. D) An evaluation of the child's physical, language, emotional, and social capabilities is performed to determine eligibility. E) The primary care nurse manages the developmental services and special education that the child requires. F) The goal is to maintain a natural environment, so most services occur in the home or day care center. 13. The nurse is reviewing the Adolescent Health Transition Project (AHTP)-recommended schedule for transition planning. According to the schedule, at what age should the nurse explore health care financing for young adults? A) 12 years old B) 14 years old C) 17 years old D) 19 years old 14. The nurse is caring for a 14-year-old girl with special health needs. What is the priority intervention for this child? A) Encouraging the parents to promote the child's self-care B) Assessing the child for signs of depression C) Discussing how her care will change as she grows D) Monitoring for compliance with treatment 15. The nurse is helping a 20-year-old woman transition to adult care. Which of the following would be the most important role of the nurse following a successful transition? A) Teacher B) Consultant C) Care provider D) Advocate 16. The nurse is coordinating home care for a 3-year-old girl with special care needs. Which approach provides the greatest benefit to the family of this child? A) Asking the father for his observations on his daughter's progress B) The nurse adjusting her office schedule to be available C) Urging parents to arrange respite care whenever possible D) Monitoring the mother for depression 17. The nurse caring for young children in a hospice setting is aware of the following statistics related to the occurrence of death in children. Which one of the following statements accurately reflects one of these statistics? A) Each year, about 50,000 children die in the United States; of those, about 15,000 are infants. B) It is unusual for a child's chronic illness to progress to the point of becoming a terminal illness. C) Despite strides made, diabetes remains the leading cause of death from disease in all children older than the age of 1 year. D) Congenital defects and traumatic injuries are the most common causes of diseases leading to death. 18. The nurse is providing home care for the family of an 8-year-old boy who is dying of leukemia. Which action will be most supportive to the parents of the child? A) Encouraging organ and tissue donation B) Being patient with parental indecision C) Getting prior authorization for treatments D) Explaining how anorexia is a natural process 19. The nurse is providing palliative care for a 9-year-old boy in hospice. Which is unique to hospice care for children? A) Encouraging visits from friends and family B) Educating parents about terminal dehydration C) Prolonging treatment that might possibly help D) Treating constipation to relieve abdominal pain 20. When providing care to a dying child and his family, which of the following would be most important? A) Focusing on the family as the unit of care B) Teaching the family appropriate care measures C) Offering the child support and encouragement D) Assisting the parents in decision making 21. The parents of an 11-year-old boy who is dying from cancer are concerned that he is not eating. Which intervention would serve both the parents' and child's needs? A) Urging the child to eat one good meal per day B) Serving small meals of things the child likes C) Straightening up around the child before meals D) Administering antiemetics as ordered for nausea 22. The nurse is caring for terminally ill children in a hospital setting. With which of the following children would the nurse consult regarding the continuation or withdrawal of treatment? A) A 4-year-old with an inoperable brain tumor B) A 5-year-old with kidney failure C) A 6-year-old with life-threatening injuries D) A 7-year-old with end-stage leukemia 23. The nurse is caring for a child involved in an automobile accident whose family has been informed that the child is brain dead. Which of the following teachings might the nurse provide the family regarding organ donation? A) The nurse should ask about organ donation when the family is informed of their child's condition. B) The nurse should explain that written consent is necessary for the organ donation. C) The nurse should make sure the parents know that procurement of organs may mar their child's appearance. D) The nurse should make sure the parents know that they will be responsible for expenses related to organ procurement. 24. The nurse is caring for a preschool child who is receiving palliative care for end-stage cancer. Which of the following would be the focus of age-appropriate interventions for this child? A) Providing unconditional love and trust B) Providing a familiar and consistent routine C) Teaching the child that death is not punishment D) Providing specific, honest details of death 25. The nurse is caring for a 5-year-old boy who is terminally ill. Which intervention would best meet the needs of this dying child? A) Offer the child decision-making opportunities. B) Provide the child with specific details. C) Assure the child that he did nothing wrong. D) Act as a confidant for the child's concerns. 26. Which of the following would the nurse include in the plan of care for a dying child with pain? A) Administering analgesics as needed B) Using measures the nurse finds comforting C) Playing the television or radio so the child can hear it D) Changing the child's position frequently but gently 27. When describing organ donation to the family of a dying child, which of the following would the nurse include in the discussion? A) Telling them that further harm may occur to the child through the process B) Tell them that their cultural and religious beliefs will be considered C) Including this topic in the discussion of impending death D) Informing the family that organ donation will delay the funeral Answer Key 1. A 2. C 3. C 4. D 5. B 6. A 7. A 8. A 9. A, B, D, E 10. D 11. D 12. C, D, F 13. C 14. C 15. B 16. B 17. D 18. B 19. C 20. A 21. B 22. D 23. B 24. C 25. C 26. D 27. B Chapter 35- Key Pediatric Nursing Interventions 1. The nurse is administering Tylenol PRN to a 9-year-old child on the pediatric ward of the hospital. Which of the following reflect nursing actions that follow the rules of the "eight rights" of pediatric medication administration? Select all answers that apply. A) The nurse identifies the child by checking the name on the child's chart. B) The nurse makes sure the medication is given within the hour of the ordered time. C) The nurse checks the documented time of the last dosage administered. D) The nurse calculates the dosage according to the child's weight. E) The nurse explains the therapeutic effects of the medication to the child and parents. F) The nurse administers the medication even though the child is adamant about not taking it. 2. The nurse is teaching the student nurse the factors that affect the pharmacodynamics of the drugs they are administering. Which of the following is a factor affecting this property of drugs? A) Immature body systems B) Weight C) Body surface D) Body composition 3. When describing the differences affecting the pharmacokinetics of drugs administered to children, which of the following would the nurse include? A) Oral drugs are absorbed more quickly in children than adults. B) Absorption of intramuscularly administered drugs is fairly constant. C) Topical drugs are absorbed more quickly in young children than adults. D) Absorption of drugs administered by subcutaneous injection is increased. 4. The nurse is helping a 14-year old boy who has asthma to administer medication via an inhaler. Which of the following describes a developmentally appropriate nursing intervention for this child? A) Involve the adolescent's parents in the administration of the medication. B) Allow the adolescent to handle a demo inhaler prior to administering the medication. C) Offer the adolescent a special treat if he uses his inhaler correctly. D) Treat the adolescent as an adult when explaining the use of the inhaler. 5. The nurse is providing teaching for the mother of an infant who receives all of his nutrition through a tube. The nurse is reviewing interventions to promote growth and development. Which of the following responses from the mother indicates a need for further teaching? A) “I will give him a pacifier during feeding time.” B) “We need to keep feeding time very quiet.” C) “We need to make sure he doesn't lose the desire to eat by mouth.” D) “Sucking produces saliva, which aids in digestion.” 6. The nurse is preparing to administer oral ampicillin to a child who weighs 40 kg. The safe dose for children is 50 to 100 mg/kg/day divided in doses administered every 6 hours. What would be the low single safe dose and high single safe dose per day for this child? A) 50 to 100 mg per dose B) 100 to 500 mg per dose C) 500 to 1,000 mg per dose D) 1,000 to 5,000 mg per dose 7. The nurse is preparing to administer a medication to a 5-year-old who weighs 35 pounds. The prescribed single dose is 1 to 2 mg/kg/day. Which of the following is the appropriate dose range for this child? A) 8 to 16 mg B) 16 to 32 mg C) 35 to 70 mg D) 70 to 140 mg 8. The nurse is administering a crushed tablet to an 18-month-old infant. Which of the following is a recommended guideline for this intervention? A) Mix the crushed tablet with a small amount of applesauce. B) Place the crushed tablet in the infant's formula. C) Mix the crushed tablet with the infant's cereal. D) Crushed tablets should only be mixed with water. 9. The nurse is preparing to administer medication to a child with a gastrostomy tube in place. Which of the following is a recommended guideline for this procedure? Select all answers that apply. A) Verify proper tube placement prior to instilling medication. B) Mix liquid medications with a small amount of water and add directly into the tube. C) Mix powdered medications well with cold water first. D) Crush tablets and mix with warm water to prevent tube occlusion. E) Open up capsules and mix the contents with warm water. F) Flush the tube with water after administering medications. 10. The nurse is administering immunizations to children in a neighborhood clinic. Which of the following is the most frequent route of administration? A) Oral B) Intradermal C) Intramuscular D) Topical 11. The nurse is preparing to administer insulin to a diabetic child. Which of the following would be the recommended route for this administration? A) Subcutaneous B) Intradermal C) Intramuscular D) Oral 12. The nurse is caring for an 8-year-old girl who requires medication that is only available in an enteric tablet form. The nurse is teaching the mother how to help the girl swallow the medication. Which of the following statements indicates a need for further teaching? A) “I can encourage her to place it on the back of her tongue.” B) “I can pinch her nose to make it easier to swallow.” C) “We cannot crush this type of pill as it will affect the delivery of the medication.” D) “We can place the tablet in a spoonful of applesauce.” 13. The nurse is providing teaching on how to administer nasal drops. Which of the following responses by the parents indicates a need for further teaching? A) “We need to be careful not to stimulate a sneeze.” B) “She needs to remain still for at least 10 minutes after administration.” C) “Our daughter should lie on her back with her head hyperextended.” D) “We must not let the dropper make contact with the nasal membranes.” 14. The nurse is preparing to administer ear drops to a 6-year-old. To ensure that the medication is instilled properly, the nurse does which of the following? A) Pulls the pinna downward B) Pulls the pinna downward and back C) Pulls the pinna upward D) Pulls the pinna upward and back 15. The nurse is administering a liquid medication to a 3-year-old using an oral syringe. Which action would be most appropriate? A) Direct the liquid toward the anterior side of the mouth. B) Keep the child's hand away from the oral syringe when squirting the medication. C) Give all of the drug in the syringe at one time with one squirt. D) Allow the child time to swallow the medication in between amounts. 16. After administering eye drops to a child, the nurse applies gentle pressure to the inside corner of the eye at the nose for which reason? A) To promote dispersion over the cornea B) To enhance systemic absorption C) To ensure the medication stays in the eye D) To stabilize the eyelid 17. The nurse is preparing to administer an intramuscular injection to an 8-month-old infant. Which site would the nurse select? A) Rectus femoris B) Vastus lateralis C) Dorsogluteal muscle D) Deltoid 18. The nurse is providing care for a 10-year-old girl who has required multiple venipunctures and a computed tomography (CT) scan in a single day. The girl has expressed no fear or need for comfort. How should the nurse respond? A) “Tell me about your day today.” B) “Are you doing okay?” C) “Are you feeling okay?” D) “You have done really well today.” 19. The nurse is preparing a 5-year-old for a radiograph. Which of the following would be the best communication to prepare the child for the procedure? A) “We are going to take some x-rays of your body.” B) “We need to look inside at some of your organs.” C) “X-rays are not painful; you won't feel a thing.” D) “We are going to use a big camera to take pictures inside your body.” 20. The nurse is supporting an 8-year-old child who is having blood specimens drawn. Which method would be least appropriate to use for distraction? A) “Squeeze my hand as tight as you can.” B) “Look at how many dots there are on the ceiling.” C) “Count with me slowly from 1 to 20.” D) “It's okay to scream if it hurts.” 21. The nurse is choosing a vein to insert a peripheral IV for a 2-year-old child. Which of the following sites would be appropriate? Select all answers that apply. A) Hand veins B) Feet veins C) Jugular vein D) Forearm veins E) Scalp veins F) Vena cava 22. The nurse is caring for children who are receiving IV therapy in the hospital setting. For which of the following children would a central venous device be indicated? A) A child who is receiving an IV push B) A child who is receiving chemotherapy for leukemia C) A child who is receiving IV fluids for dehydration D) A child who is receiving a one-time dose of a medication 23. The nurse is determining the amount of IV fluids to administer in a 24-hour period to a child who weighs 40 kg. How many milliliters should the nurse administer? A) 1,000 mL B) 1,500 mL C) 1,750 mL D) 1,900 mL 24. A nurse has just administered medication via an orogastric tube. What is the priority nursing action following administration? A) Check tube placement. B) Retape the tube. C) Flush the tube. D) Remove the tube. 25. A nurse is caring for a 14-year-old with a gastrostomy tube. The girl has skin breakdown and irritation at the insertion site. Which of the following would be the most appropriate method to clean and secure the gastrostomy tube? A) Make sure the tube cannot be moved in and out of the child's stomach. B) Use adhesive tape to tape the tube in place and prevent movement. C) Place a transparent dressing over the site whether there is drainage or not. D) If any drainage is present, use a presplit 2 × 2 and place it loosely around the site. 26. The nurse is explaining to the student nurse the therapeutic effects of total parenteral nutrition (TPN). Which of the following accurately describes the use of TPN? A) It is used short term to supply additional calories and nutrients as needed. B) It is delivered via the peripheral vein to allow rapid dilution of hypertonic solution. C) It is a highly concentrated solution of carbohydrates, electrolytes, vitamins, and minerals. D) It is usually used when the child's nutritional status is within acceptable parameters. 27. The nurse is caring for a 6-year-old child who has multisystem trauma due to a motor vehicle accident. The child is receiving total parenteral nutrition (TPN). Which of the following is a recommended nursing intervention for children on TPN? A) Initially, check blood glucose levels frequently, such as every 4 to 6 hours, to evaluate for hyperglycemia. B) Be vigilant in monitoring the infusion rate, change the rate as necessary, and report any changes to the physician or nurse practitioner. C) If for any reason the TPN infusion is interrupted or stops, begin an infusion of a 10% saline at the same infusion rate as the TPN. D) Administer TPN continuously over an 8-hour period, or after initiation it may be given on a cyclic basis, such as over a 12-hour period during the night. 28. The nurse is caring for a child who is receiving total parenteral nutrition (TPN) for failure to thrive. Which of the following nursing actions might the nurse take to prevent complications from this therapy? A) Adhere to clean technique when caring for the catheter and administering TPN. B) Ensure that the system remains an open system at all times. C) Secure all connections and open the catheter during tubing and cap changes. D) Use occlusive dressings and chlorhexidine-impregnated sponge (Biopatch) dressings.   Answer Key 1. C, D, E 2. A 3. C 4. D 5. B 6. C 7. B 8. A 9. A, D, E, F 10. C 11. A 12. B 13. B 14. D 15. D 16. C 17. B 18. A 19. D 20. D 21. A, B, D, E 22. B 23. D 24. C 25. D 26. C 27. A 28. D Chapter 36- Pain Management in Children 1. The nurse is teaching the student nurse the physiology involved in pain transmission. Which of the following statements accurately describes a physiologic event in the nervous system related to pain transmission? Select all answers that apply. A) Thermal stimulation may involve the release of mediators, such as histamine, prostaglandins, leukotrienes, or bradykinin. B) When nociceptors are activated by noxious stimuli, the stimuli are converted to electrical impulses that are relayed to the spinal cord and brain. C) Myelinated A-delta fibers are large fibers that conduct the impulse at very rapid rates; unmyelinated small C fibers transmit the impulse slowly. D) Once in the dorsal horn of the spinal cord, the nerve fibers divide and then cross to the opposite side and rise upward to the thalamus. E) The point at which the person first feels the highest intensity of the painful stimulus is termed the pain threshold. F) Peripheral sensitization allows the nerve fibers to react to a stimulus that is of lower intensity than would be needed to cause pain. 2. The nurse is managing children who have chronic diseases in a neighborhood clinic. Which of the following are examples of chronic conditions? Select all answers that apply. A) Diabetes mellitus B) Myocardial infarction C) Rheumatoid arthritis D) Compound fracture E) Acute asthma F) Bronchopneumonia 3. The nurse is caring for a child who is recovering from an appendectomy. Which of the following is the appropriate term for the pain this child is experiencing? A) Nociceptive pain B) Neuropathic pain C) Chronic pain D) Superficial somatic pain 4. The nurse is conducting an assessment of a high school track athlete. He tells the nurse he is experiencing pain along his outer thigh. He describes it as tight, achy, and tender, particularly after he runs. The nurse understands that he is most likely experiencing what kind of pain? A) Cutaneous B) Neuropathic C) Visceral D) Deep somatic 5. The nurse is caring for a child who is experiencing pain related to chemotherapy treatment. Which of the following is a behavioral factor that might affect the child's pain experience? A) Knowledge of the therapy B) Fear about the outcome of therapy C) Participation in normal routine activities D) Ability to identify pain triggers 6. The nurse caring for infants in the neonatal intensive care unit (NICU) relies on the use of behavioral and physiologic indicators for determining pain. Which of the following examples are behavioral indicators? Select all answers that apply. A) The infant grimaces. B) The infant's heart rate is elevated. C) The infant flails his arms and legs. D) The infant's respiratory rate is elevated. E) The infant is crying uncontrollably. F) The infant's oxygen saturation is low. 7. A nurse is providing teaching to the mother of an adolescent girl about how to manage menstrual pain nonpharmacologically. Which of the following statements by the mother indicates a need for further teaching? A) “I need to help her learn techniques to distract her, card games, for example.” B) “I need to be able to identify the subtle ways she shows pain.” C) “I need to follow these instructions exactly for them to work properly.” D) “I need to encourage her to practice and utilize these techniques.” 8. The nurse is counseling the parents of a 9-year-old boy who is receiving morphine for postoperative pain. Which of the following statements from the nurse accurately reflects the pain experience in children? A) "You can expect that your child will tell you when he is experiencing pain." B) "Your child will learn to adapt to the pain he is experiencing." C) "Your child will experience more adverse effects to narcotics than adults." D) "It is very rare that children become addicted to narcotics." 9. The nurse is using the acronym QUESTT to assess the pain of a child. Which of the following is an accurate descriptor of this process? A) Question the child's parents. B) Understand the child's pain level. C) Establish a caring relationship with the child. D) Take the cause of pain into account when intervening. 10. When the nurse is assessing a child's pain, which of the following is most important? A) Obtaining a pain rating from the child with each assessment B) Using the same tool to assess the child's pain each time C) Documenting the child's pain assessment D) Asking the parents about the child's pain tolerance 11. Which tool would be the least appropriate scale for the nurse to use when assessing a 4- year-old child's pain? A) FACES pain rating scale B) Oucher pain rating scale C) Poker chip tool D) Numeric pain intensity scale 12. The nurse uses the FLACC behavioral scale to assess a 6-year-old's level of postoperative pain and obtains a score of 9. The nurse interprets this to indicate that the child is experiencing: A) Little to no pain B) Mild pain C) Moderate pain D) Severe pain 13. The nurse is assessing the pain of a postoperative newborn. The nurse measures the infant's facial expression, body movement, sleep, verbal or vocal ability, consolability, and response to movements and touch. Which behavioral assessment tool is being used by the nurse? A) Riley Infant Pain Scale B) Pain Observation Scale for Young Children C) CRIES Scale for Neonatal Postoperative Pain Assessment D) FLACC Behavioral Scale for Postoperative Pain in Young Children 14. The nurse is researching behavioral-cognitive pain relief strategies to use on a 5-year- old child with unrelieved pain. Which of the following methods might the nurse choose? Select all answers that apply. A) Relaxation B) Distraction C) Biofeedback D) Thought stopping E) Massage F) Sucking 15. The nurse tells a joke to a 12-year-old to distract him from a painful procedure. What pain management technique is the nurse using? A) Relaxation B) Distraction C) Imagery D) Thought stopping 16. The nurse is providing instructions to a mother on how to use thought stopping to help her child deal with anxiety and fear associated with frequent painful injections. Which of the following statements indicates the mother understands the technique? A) “We will imagine that we are on the beach in Florida.” B) “We can talk about our favorite funny movie and laugh.” C) “She can let her body parts go limp, working from head to toe.” D) “We'll repeat 'quick stick, feel better, go home soon' several times.” 17. The student nurse is learning about the effects of heat and cold when used in a pain management plan. Which of the following accurately describes one of these effects? A) Cold results in vasodilation. B) Cold alters capillary permeability. C) Heat results in vasoconstriction. D) Heat decreases blood flow to the area. 18. The nurse is teaching an 8-year-old child and his family how to manage cancer pain using nonpharmacologic methods. Which of the following parent statements signifies successful child teaching? A) "I will avoid using descriptive words like pinching, pulling, or heat." B) "I will not use positive reinforcement until the technique is perfected." C) "I will begin using the technique before he experiences pain." D) "I will be honest and tell him that the procedure will hurt a lot." 19. For which of the following children would nonopioid analgesics be recommended? A) A child with juvenile arthritis B) A child with end-stage cancer C) A child with a broken arm D) A child with severe postoperative pain 20. Prior to administering morphine to a 10-year-old child, the nurse reviews the adverse effects of the drug. Which system is primarily affected by the drug causing most of the adverse effects? A) Central nervous system B) Peripheral nervous system C) Digestive system D) Musculoskeletal system 21. The nurse is administering pain medication for a child with continuous pain from internal injuries. Which of the following methods would be ordered to dispense the medication? A) Administer the medication PRN (as needed). B) Administer the mediation when pain has peaked. C) Administer the medication around the clock at timed intervals. D) Administer the medication when the child complains of pain. 22. The nurse has applied EMLA cream as ordered. How does the nurse assess that the cream has achieved its purpose? A) Assess the skin for redness. B) Note any blanching of skin. C) Lightly tap the area where the cream is. D) Gently poke the child with a needle. 23. The nurse is preparing to administer a topical anesthetic for a 10-year-old girl with a chin laceration. The nurse would expect to apply which of the following as ordered in preparation for sutures? A) TAC (tetracaine, epinephrine, cocaine B) Iontophoretic lidocaine C) EMLA D) Vapocoolant spray 24. The nurse is caring for a child who is complaining of chronic pain. Which of the following is the priority nursing assessment? A) How the pain impacts the child's and family's stress level B) The pain's history, onset, intensity, duration, and location C) The child's and parents' feeling of anxiety and depression D) The child's cognitive level and emotional response 25. The nurse is monitoring a child who has received epidural analgesia with morphine. The nurse is careful to monitor for which of the following adverse effects of the medication? A) Epidural hematoma B) Arachnoiditis C) Spinal headache D) Respiratory depression 26. The nurse is providing teaching to the parents of a newborn prior to a heelstick. The nurse is describing the procedure and recommending various methods for the parents to help comfort their baby. Which of the following statements by the parents indicates a need for further teaching? A) “It's better if we are not in the room for this.” B) “We can use kangaroo care before and after.” C) “We hope you are using a very tiny needle.” D) “We can offer him nonnutritive sucking to calm him.” 27. The nurse is conducting a pain assessment of a 10-year-old boy who has been taking acetaminophen for chronic knee pain. The assessment indicates that the recommended dose is no longer providing adequate relief. What is the appropriate nursing action? A) Increase the dosage of the acetaminophen. B) Tell the child he is experiencing the ceiling effect. C) Use guided imagery to help his pain. D) Obtain an order for a different medication. 28. The nurse is preparing a child for a lumbar puncture. How far ahead of the procedure should the nurse apply the EMLA cream? A) 30 minutes B) 1 hour C) 3 hours D) 4 hours 29. The nurse is explaining the effects of heat application for pain relief. Which of the following would the nurse be likely to include as an effect? A) Increased blood flow to the area B) Increased pressure on nociceptive fibers C) Possible release of endogenous opioids D) Altered capillary permeability 30. Pentazocine is prescribed for a child with moderate pain. The nurse identifies this drug as an example of which type? A) Nonsteroidal anti-inflammatory drug (NSAID B) Prostaglandin inhibitor C) Opioid D) Mixed opioid agonist–antagonist Answer Key 1. B, C, D, F 2. A, C, E 3. A 4. D 5. C 6. A, C, E 7. C 8. D 9. D 10. B 11. D 12. D 13. A 14. A, B, C, D 15. B 16. D 17. B 18. C 19. A 20. A 21. C 22. C 23. A 24. B 25. D 26. A 27. D 28. C 29. B 30. D Chapter 37- Nursing Care of the Child With an Infectious or Communicable Disorder 1. A 3-year-old boy has been on chemotherapy for cancer. He complains about a sore throat, is experiencing malaise, and has a temperature of 99.8°F orally. His mother calls the child's physician. What is the appropriate information for the nurse to tell his mother at this time? A) “Continue to watch the child, giving him aspirin and cool fluids for the fever.” B) “Plan to bring the child into the physician's office today.” C) “Monitor the temperature, but not to worry unless it gets above 104°F.” D) “Keep the child warm and as comfortable as possible.” 2. A nurse is conducting a health history for a 1-month-old with an infectious disorder. Which segment of the health history would be most helpful for the nurse when determining if the infant developed the infection from the mother? A) Family history B) Past medical history C) Home treatments D) Present illness history 3. While reviewing various studies about the use of antipyretics possibly prolonging illness, the nurse notes that there are benefits to their use for the child with fever. Which of the following would the nurse identify as the best explanation related to the benefit of antipyretics? A) They slow the growth of bacteria. B) They increase neutrophil production. C) They encourage T-cell proliferation. D) They help decrease fluid requirements. 4. The nurse is caring for a neonate who is suspected of having sepsis. Which of the following assessment findings would the nurse interpret as most indicative of sepsis? A) Rash on face B) Edematous neck C) Hypothermia D) Coughing 5. The nurse is preparing a plan of care for a 5-year-old boy with chickenpox. Which of the following nursing interventions should be questioned? A) Administer antipyretics as ordered. B) Keep the child's fingernails short. C) Monitor fluid intake and output. D) Provide alcohol baths as needed. 6. The nurse is developing a teaching plan for the mother of a 4-year-old girl with cold and fever. Which of the following would the nurse include in this teaching plan? A) Keeping the child covered and warm B) Calling the doctor if the child's fever lasts more than 36 hours C) Ensuring fluid intake to prevent dehydration D) Observing for changes in alertness resulting from brain damage 7. After teaching a mother how to remove a tick from her 6-year-old boy's arm, the nurse determines that additional teaching is needed when the mother states which of the following? A) “I'll protect my fingers with a paper towel.” B) “I'll grasp the tick and pull it away quickly.” C) “I should put the tick in a plastic bag in the freezer.” D) “I need to grasp the tick close to the child's skin.” 8. The nurse is performing a physical examination of an 8-year-old girl who was bitten by her kitten. Which of the following assessments would lead the nurse to suspect cat- scratch disease? A) Swollen lymph nodes B) Strawberry tongue C) Infected tonsils D) Swollen neck 9. A group of nursing students are reviewing the six links in the chain of infection and the nursing implications for each. The students demonstrate understanding of the information when they identify which precaution as helping to break the chain of infection to the susceptible host? A) Keeping linens dry and clean B) Maintaining skin integrity C) Washing hands frequently D) Coughing into a handkerchief 10. The nurse is performing a physical examination on a 9-year-old boy who has experienced a tick bite on his lower leg and is suspected of having Lyme disease. Which assessment finding would the nurse expect to find? A) Swelling in the neck B) Confusion and anxiety C) Ring-like rash on lower leg D) Hypersalivation 11. The nurse determines that it is necessary to implement airborne precautions for children with which of the following infections? A) Measles B) Streptococcus group A C) Rubella D) Scarlet fever 12. A child is diagnosed with scarlet fever. The nurse is reviewing the child's medical record, expecting which medication to be prescribed for this child? A) Ibuprofen B) Acyclovir C) Penicillin V D) Doxycycline 13. A mother brings her 8-year-old son for evaluation because of a rash on his lower leg. Which finding would support the suspicion that the child has Lyme disease? A) Playing in the woods about a week ago B) Rash is papular and vesicular C) High fever occurring about 4 days before the rash D) Complaints of extreme pruritus with visible nits 14. Which of the following would be most important to include in the teaching plan for parents of a child with pinworm? A) “Seal the child's clothing in a plastic bag for at least 10 days.” B) “Be sure your child wears shoes at all times.” C) “Make sure the child washes his hands after using the bathroom.” D) “After applying this special cream, leave it on for about 8 to 10 hours.” 15. After teaching the parents of a child with varicella zoster, the nurse determines that the parents have understood the teaching when they state that their child can return to school at which time? A) After day 5 of the rash B) When the rash is completely healed C) Once the rash appears D) After the lesions have crusted 16. After teaching a class on the role of white blood cells in infection, the instructor determines that the teaching was successful when the class identifies which type of white blood cells as important in combating bacterial infections? A) Neutrophils B) Eosinophils C) Basophils D) Lymphocytes 17. A woman gives birth to a healthy newborn. As part of the newborn's care, the nurse instills erythromycin ophthalmic ointment as a preventive measure for which sexually transmitted infection (STI)? A) Genital herpes B) Hepatitis B C) Syphilis D) Gonorrhea 18. A pregnant woman diagnosed with syphilis comes to the clinic for a visit. The nurse discusses the risk of transmitting the infection to her newborn, explaining that this infection is transmitted to the newborn through which of the following? A) Amniotic fluid B) Placenta C) Birth canal D) Breast milk 19. A nursing instructor is teaching a group of students about the action of antipyretic agents in children. The instructor determines that the teaching has been successful when the students identify which of the following as the primary action? A) Cause vasodilation to promote heat loss B) Decrease the temperature set point C) Block release of histamine D) Promote prostaglandin production 20. A nurse is instructing a parent on how to obtain a stool culture for ova and parasites from a child with diarrhea. Which of the following would the nurse include in the teaching plan? A) “Give the child bismuth and then collect the next specimen.” B) “Obtain the specimen from the toilet after the child has a bowel movement.” C) “Keep the specimen from coming into contact with any urine.” D) “Bring the specimen to the laboratory on the third day.” 21. The nurse is preparing to obtain a blood specimen via capillary heel puncture. Which of the following would be most appropriate for the nurse to do? A) Apply a cool compress for several minutes before collection B) Elevate the extremity used after puncturing it C) Squeeze the area to facilitate specimen collection D) Wipe away the first drop of blood with dry gauze 22. The nurse is assessing the tympanic temperature of several children. The nurse documents that the child with which temperature reading has a fever? A) 99.5°F B) 99.2°F C) 100.0°F D) 100.8°F 23. A school-aged child with an infectious disease is placed on transmission-based precautions. Which nursing diagnosis would most likely be a priority? A) Impaired skin integrity related to trauma secondary to pruritus and scratching B) Fluid volume deficit related to increased metabolic demands and insensible losses C) Social isolation related to infectivity and inability to go to the playroom D) Deficient knowledge related to how infection is transmitted 24. A group of nursing students are reviewing information about childhood infectious diseases. The students demonstrate understanding of this information when they identify which of the following as a common childhood exanthema? A) Mumps B) Rabies C) Rubella D) West Nile virus 25. The parents of a 5-year-old have just found out that their child has head lice. Which statement by the parents would support the nursing diagnosis of deficient knowledge? A) “I can't believe it. We're not unclean, poor people.” B) “We'll have to get that special shampoo.” C) “Everybody in the house will need to be checked.” D) “That explains his complaints of itching on his neck.” 26. While obtaining a health history from a male adolescent during a well check-up, the nurse assesses his sexual behavior and risk for sexually transmitted infections. Based on the information, the nurse plans to teach the adolescent about using a condom. Which of the following would the nurse include in the teaching plan? A) “You can reuse a condom if it's within 3 hours.” B) “Store your condoms in your wallet so they are ready for use.” C) “Put the condom on before engaging in any genital contact.” D) “Use Vaseline with a latex condom for extra lubrication.” 27. A nurse suspects that an adolescent may have community-acquired methicillin-resistant Staphylococcus aureus (CAMRSA). Which of the following would the nurse expect to assess? Select all answers that apply. A) Participation in contact sport B) Recent cut on the lower leg C) History of a recent sort throat D) Raised fluctuant lesions E) Erythematous rash over the trunk and face 28. A child is diagnosed with a helminthic infection. Which of the following would the nurse expect to be prescribed? Select all answers that apply. A) Erythromycin B) Albendazole C) Pyrantel pamoate D) Acyclovir E) Metronidazole F) Permethrin 29. The nurse is providing instructions to parents of a 2-year-old child with a fever. The child weighs 33 pounds. Based on the recommended dose for ibuprofen, how much would the nurse instruct the parents to give as the lowest amount per dose? Answer: ____mg   Answer Key 1. B 2. B 3. D 4. C 5. D 6. C 7. B 8. A 9. B 10. C 11. A 12. C 13. A 14. C 15. D 16. A 17. D 18. B 19. B 20. C 21. D 22. D 23. C 24. C 25. A 26. C 27. A, B, D 28. B, C 29. 75 mg Chapter 38- Nursing Care of the Child With an Alteration in Intracranial Regulation / Neurologic Disorder 1. When providing care to a newborn infant who was born at 29 weeks' gestation, the nurse integrates knowledge of potential complications, being alert for signs and symptoms of which of the following? A) Neonatal conjunctivitis B) Facial deformities C) Intracranial hemorrhage D) Incomplete myelinization 2. The nurse knows that children have larger heads in relation to the body and a higher center of gravity. When developing a teaching plan for parents, the nurse includes information about an increased risk for which of the following problems? A) Febrile seizures B) Head trauma C) Caput succedaneum D) Posterior plagiocephaly 3. The nurse is caring for a child hospitalized with Reye syndrome who is in the acute stage of the illness. The nurse would assess the child most carefully for which of the following? A) Indications of increased intracranial pressure B) An increase in the blood glucose level C) A decrease in the liver enzymes D) A presence of protein in the urine 4. The physician has ordered rectal diazepam (Valium) for a 2-year-old boy with status epilepticus. Which of the following instructions is essential for the nurse to teach the parents? A) Monitor their child's level of sedation. B) Watch for fever indicating infection. C) Gradually reduce the dosage as seizures stop. D) Monitor for an allergic reaction to the medication. 5. As a result of seizure activity, a computed tomography (CT) scan was performed and showed that an 18-month-old child has intracranial arteriovenous malformation. When developing the child's plan of care, the nurse would expect to implement actions to prevent which of the following? A) Drug interactions B) Developmental disabilities C) Hemorrhagic stroke D) Respiratory paralysis 6. A 16-year-old boy complains to the school nurse of headaches and a stiff neck. Which of the following signs and symptoms would alert the nurse that the child may have bacterial meningitis? A) Fixed and dilated pupils B) Frequent urination C) Sunset eyes D) Sunlight is “too bright” 7. A 4-year-old boy has a febrile seizure during a well-child visit. Which of the following would be a priority? A) Hyperextending the child's head while placing him on his side B) Using a tongue blade to pry open the child's jaw C) Loosening the child's clothing to ensure a patent airway D) Protecting the child from harm during the seizure 8. The nurse has developed a teaching plan for the family of a 2-year-old boy who holds his breath when he gets frustrated. Which of the following will be most important to include in this plan? A) Provide cuddle time whenever the child begins to act out. B) Explain the child's behavior to the parents. C) Encourage the parents to interact more with the child. D) Stay close to prevent injury when he gets frustrated. 9. The nurse is caring for an 8-year-old boy who has chronic epilepsy. Which of the following would be most important to address when teaching the child and parents about living with this condition? A) Multiple corrective surgeries to slowly remove diseased parts of his brain B) Physical, occupational, and speech therapy to maximize his potential C) Support for maintaining self-esteem because of his altered lifestyle D) Hyperventilation therapy to counteract the periods of decreased oxygenation 10. A 4-year-old boy has a history of seizures and has been started on a ketogenic diet. Which of the following food selections would be most appropriate for his lunch? A) Fried eggs, bacon, and iced tea B) A hamburger on a bun, French fries, and milk C) Spaghetti with meatballs, garlic bread, and a cola drink D) A grilled cheese sandwich, potato chips, and a milkshake 11. A child with increased intracranial pressure is being treated with hyperventilation. The nurse understands the need for this treatment is based on which of the following? A) PaCO2 levels decrease, causing vasoconstriction. B) Drainage of cerebrospinal fluid occurs. C) Activity is controlled via a stimulator. D) Hyperexcitability of the nerves is reduced. 12. The nurse assesses a child's level of consciousness, noting that the child falls asleep unless he is stimulated. The nurse documents this finding as: A) Confusion B) Obtunded C) Stupor D) Coma 13. During a well-child visit, the nurse assesses an infant's ability to suck on a pacifier. The nurse is assessing which cranial nerve? A) Olfactory B) Trigeminal C) Facial D) Accessory 14. The nurse inspects the eyes of a child and observes that the sclera is showing over the top of the iris. The nurse documents this finding as: A) Decorticate posturing B) Nystagmus C) Doll's eye D) Sunsetting 15. Which of the following would lead the nurse to suspect that a child is beginning to develop increased intracranial pressure? A) Bradycardia B) Cheyne-Stokes respirations C) Fixed, dilated pupils D) Projectile vomiting 16. A nurse is talking with the parents of a child who has had a febrile seizure. The nurse would integrate an understanding of which of the following into the discussion? A) The child's risk for cognitive problems is greatly increased. B) Structural damage occurs with febrile seizure. C) The child's risk for epilepsy is now increased. D) Febrile seizures are benign in nature. 17. A nurse is preparing a school-aged child for a lumbar puncture. The nurse would expect to position the child in which manner? A) On her side with the head flexed forward and knees flexed to the abdomen B) Sitting upright with the head flexed forward to the chest C) Supine with arms and legs pronated and extended D) Prone with the arms flexed under the chest 18. A group of nursing students are reviewing information related to seizures that occur in infants and children. The students demonstrate a need for additional review when they identify which type as common in neonates? A) Tonic B) Focal clonic C) Multifocal clonic D) Myoclonic 19. Hydrocephalus is suspected in a 4-month-old infant. Which of the following would the nurse expect to assess? A) Sunken fontanels B) Diminished reflexes C) Lower extremity spasticity D) Skull symmetry 20. A nurse is providing teaching to the parents of a child who has had a shunt inserted as treatment for hydrocephalus. The parents demonstrate understanding of the teaching when they state which of the following? A) “Having the shunt put in decreases his risk for developmental problems.” B) “If he doesn't get an infection in the first week, the risk is greatly reduced.” C) “He will need more surgeries to replace the shunt as he grows.” D) “The shunt will help to prevent any further complications from his disease.” 21. A nurse is preparing a presentation for an expectant parent group about neural tube defects and prevention. Which of the following would the nurse emphasize? A) Smoking cessation B) Aerobic exercise C) Increased calcium intake D) Folic acid supplementation 22. A 15-year-old adolescent is brought to the emergency department by his parents. The adolescent is febrile with chills that started suddenly. He states, “I had a sinus infection and sore throat a couple of days ago.” The nurse suspects bacterial meningitis based on which of the following? Select all answers that apply. A) Complaints of stiff neck B) Photophobia C) Absent headache D) Negative Brudzinski sign E) Vomiting 23. A child is brought to the emergency department after sustaining a concussion. The child is to be discharged home with his parents. Which of the following would the nurse include in the child's discharge instructions? A) “Expect his headache to get worse initially and then disappear.” B) “Wake him every 2 hours to check his movement and responses.” C) “Call your medical provider if he vomits more than five times.” D) “Any watery fluid draining from his ears is normal.” 24. A nurse is preparing a presentation for a local health fair about meningitis and has developed a display that lists the following causes: Streptococcus group B Haemophilus influenzae type B Streptococcus pneumoniae Neisseria meningitidis Which of these would the nurse highlight as the most common cause of meningitis in newborns? A) Streptococcus group B B) Haemophilus influenzae type B C) Streptococcus pneumoniae D) Neisseria meningitides 25. A group of students are reviewing information about head injuries in children. The students demonstrate understanding of this information when they identify which of the following as the most common type of skull fracture in children? A) Linear B) Depressed C) Diastatic D) Basilar 26. During class, a student states, “I didn't think children could have strokes. I thought this only occurred in older adults.” When responding to the student, which of the following would be most important for the instructor to integrate into the response? A) Strokes in children often have an identifiable cause. B) The signs and symptoms in children are different from an adult. C) Research has identified specific treatments for children. D) Ischemic strokes are more common than hemorrhagic strokes. 27. A 10-month-old is brought to the emergency department by her parents after they found her face down in the bathtub. The mother said, “I just left the bathroom to answer the phone. When I came back, I found her.” Which of the following assessments would be the priority? A) Airway, breathing, and circulation B) Level of consciousness C) Vital signs D) Pupillary response   Answer Key 1. C 2. B 3. A 4. A 5. C 6. D 7. D 8. D 9. C 10. A 11. A 12. B 13. B 14. D 15. D 16. D 17. A 18. D 19. C 20. C 21. D 22. A, B, E 23. B 24. A 25. A 26. D 27. A Chapter 39- Nursing Care of the Child With an Alteration in Sensory Perception / Disorder of the Eyes or Ears 1. The nurse is developing a plan of care for a 5-year-old child with a severe hearing impairment focusing on psychosocial interventions based on assessment findings. Which behavior would the nurse have most likely assessed? A) Immature emotional behavior B) Self-stimulatory actions C) Inattention and vacant stare D) Head tilt or forward thrust 2. The nurse is caring for a 3-month-old with nasolacrimal duct obstruction. Which intervention would be most appropriate for the nurse to implement? A) Being careful to prevent spread of infection B) Teaching the parents how to gently massage the duct C) Applying hot, moist compresses to the affected eye D) Referring the child to an ophthalmologist 3. The nurse is caring for a 10-year-old with allergic conjunctivitis. The nurse would be alert to the child's increased risk for which of the following? A) Atopic dermatitis B) Insect bite sensitivity C) Acute otitis media D) Frequent sore throats 4. The nurse is caring for a 6-year-old visually impaired boy and is about to begin the physical examination. Which intervention would be most appropriate to promote effective communication with the child? A) Show him the stethoscope. B) Describe the examination room. C) Use his name before touching him. D) Allow him to explore the exam room. 5. The nurse is instructing the parents of a school-age child with an eye disorder how to care for her eye. Which of the following conditions would the nurse explain as resolving by itself without the use of antibiotics? A) Blepharitis B) Hordeolum C) Corneal abrasion D) Chalazion 6. The nurse is caring for a 4-year-old boy with infectious conjunctivitis. Which intervention would be least appropriate to include in the child's plan of care? A) Rinsing the eye with cool water B) Educating the family about the disease C) Encouraging frequent hand washing D) Promoting eye safety 7. The nurse is examining a 7-year-old boy with blepharitis. Which of the following would the nurse least likely expect to assess? A) Redness B) Scaling C) Pain D) Edema 8. The nurse is caring for a newborn and knows that his vision, unlike his hearing, is not fully developed. Which aspect of the child's vision would the nurse expect to be similar to his father's vision? A) Adequate color detection B) Visual acuity of 20/100 C) Nearsightedness D) Monocular vision 9. The nurse is taking a health history for a 9-year-old girl. Which finding would alert the nurse to a possible risk factor specifically associated with visual impairment? A) Being born at 39 weeks' gestation B) Having several hours of homework daily C) Being of African American heritage D) Being active in sports 10. The nurse is examining a 3-year-old boy with acute otitis media who has a mild earache and a temperature of 38.5º C. Which of the following actions will be taken? A) Obtain a culture of the middle ear fluid. B) Instruct the parents to watch for worsening symptoms. C) Administer antibiotics. D) Administer antivirals. 11. A nurse is examining a 7-year-old boy with hordeolum. Which of the following would the nurse expect to find? A) Redness B) Scaling C) Pain D) Edema 12. After teaching a group of parents about ear infections in children, which statement indicates that the teaching was successful? A) Infants with congenital deformities have an increased risk for ear infections. B) Ear infections typically increase as the child gets older. C) The shorter and wider eustachian tubes of an infant increase the risk. D) Adenoids shrink as the child grows, allowing more bacteria to enter. 13. A nurse develops a plan of care for a child that includes patching the eye. This plan of care would be most appropriate for which condition? A) Astigmatism B) Hyperopia C) Myopia D) Amblyopia 14. After teaching a group of new parents about their newborns' eyes and vision, which statement by the group indicates effective teaching? A) “Our newborn can see at distances of about 1 to 2 feet.” B) “We won't know the baby's eye color until he's at least 6 months old.” C) “A baby can easily distinguish colors, but they must be bright colors.” D) “A newborn can focus with both eyes at the same time shortly after birth.” 15. Assessment of a child leads the nurse to suspect viral conjunctivitis based on which of the following? A) Mild pain B) Photophobia C) Itching D) Watery discharge 16. The mother of a school-age child brings the child to the clinic for evaluation because he is having difficulty reading. His last visual screening was normal. He also complains of headaches and dizziness. Which of the following would the nurse suspect? A) Astigmatism B) Myopia C) Hyperopia D) Nystagmus 17. Which of the following would the nurse include when teaching parents how to prevent otitis externa? A) Daily ear cleaning with cotton swabs B) Wearing ear plugs when swimming C) Using a hair dryer on high to dry the ear canals D) Using hydrogen peroxide to dry the canal skin 18. A nurse is reviewing the medical record of a child with hearing loss and notes that the child's hearing loss is in the range 40 to 60 decibels (dB). The nurse interprets this as indicating which of the following? A) Mild loss B) Moderate loss C) Severe loss D) Profound loss 19. A nurse is examining a child who has sustained blunt trauma to the eye area. The nurse suspects a simple contusion based on which of the following? A) Pain in the eye B) Impaired visual acuity C) Blurred vision D) Intact extraocular movements 20. The parents of a 10-year-old girl with a refractive error ask the nurse about the possibility of laser surgery to correct the vision. Which statement by the nurse would be most appropriate? A) “As she gets older, her vision will begin to correct itself.” B) “Laser surgery typically is not done until she's 18 years old.” C) “She looks so cute in her glasses; why put her through surgery?” D) “She can use contact lenses soon, so surgery isn't necessary.” 21. A nurse is developing a plan of care for a child who is admitted to the hospital for surgery. The child is visually impaired. Which of the following would be most appropriate for the nurse to include in the child's plan of care? Select all answers that apply. A) Explaining instructions using simple and specific terms the child understands B) Allowing the child to explore the postoperative equipment with his hands C) Touching the child on his shoulder before letting the child know someone is there D) Using the child's body parts to refer to the area where he may have postoperative pain E) Speaking to the child in a voice that is slightly louder than the usual tone of voice 22. A child with persistent otitis media with effusion is to undergo insertion of pressure- equalizing tubes via a myringotomy. The child is to be discharged later that day. After teaching the parents about caring for their child after discharge, which statement indicates that the teaching was successful? A) “The tubes will stay in place for about a month and then fall out on their own.” B) “His chances for ear infections now have dramatically decreased.” C) “He should wear earplugs when swimming in a pool or a lake.” D) “We should keep the ears protected with cotton balls for the first 24 hours.” 23. An infant is diagnosed with a congenital cataract. Which of the following would the nurse expect to assess? A) Absent red reflex B) Rapid irregular eye movement C) Misalignment of the eyes D) Enlarged eye appearance 24. A mother brings her child to the health care clinic because she thinks that the child has conjunctivitis. Which assessment finding would lead the nurse to suspect bacterial conjunctivitis? Select all answers that apply. A) Itching of the eyes B) Inflamed conjunctiva C) Stringy discharge D) Photophobia E) Mild pain F) Tearing 25. The nurse is instructing a 7-year-old child and his parents about using his prescribed corrective lenses. Which of the following would the nurse include in these instructions? A) “Make sure to take your glasses off from time to time to allow your eyes to rest.” B) “Remove your glasses with both hands and lay them with the lens upright on the surface.” C) “Clean the glasses every day with a mild soap and water or commercial cleaning agent.” D) “Use paper towels or tissues to dry and periodically clean the lenses. 26. A group of students are reviewing information about the anatomic differences in the eyes and ears of a child in comparison to an adult. The students demonstrate a need for additional study when they identify which of the following? A) Hearing is completely developed at the time of birth. B) Visual acuity develops from birth throughout childhood. C) Binocular vision is usually achieved by 2 months of age. D) The ability to discriminate colors is completed by birth. 27. A child is diagnosed with bacterial conjunctivitis and is prescribed topical antibiotic therapy. The child's mother asks when he can return to school. Which response by the nurse would be most appropriate? A) “You need to wait until you finish the entire prescription of antibiotic.” B) “Once the drainage is gone, he can go back to school.” C) “You can send him to school this afternoon after his first dose of antibiotic.” D) “He needs to be symptom-free for at least 72 hours.” 28. The parents of a 5-year-old bring their son to the emergency department because of significant eyelid edema. The mother states, “He scratched himself near his eye a couple of days ago while playing outside in the yard.” The nurse suspects periorbital cellulitis based on which of the following? A) Evidence of discharge B) Reddened conjunctiva C) Purplish discoloration of eyelid D) Altered visual acuity 29. After teaching a group of students about visual disorders, the instructor determines that the teaching was successful when the students identify which of the following as the most common cause of visual difficulties in children? A) Astigmatism B) Strabismus C) Refractive errors D) Nystagmus 30. An infant is diagnosed with infantile glaucoma. When developing the plan of care for the infant, the nurse would expect to prepare the infant and family for which of the following? A) Goniotomy B) Antibiotic therapy C) Contact lenses D) Patching of affected eye Answer Key 1. A 2. B 3. A 4. C 5. D 6. D 7. C 8. B 9. C 10. B 11. C 12. C 13. D 14. B 15. B 16. A 17. B 18. B 19. D 20. B 21. A, B, D 22. C 23. A 24. B, E 25. C 26. D 27. B 28. C 29. C 30. A Chapter 40- Nursing Care of the Child With an Alteration in Gas Exchange / Respiratory Disorder 1. The nurse is assessing a 7-year-old boy with pharyngitis. The nurse would least likely expect to assess which of the following? A) Working hard to breathe B) Difficulty swallowing C) Rash on the abdomen D) Sore throat and headache 2. The nurse hears wheezing when auscultating a 4-year-old. Which of the following conditions would the nurse most likely rule out based on the assessment findings? A) Bronchiolitis B) Asthma C) Influenza D) Cystic fibrosis 3. The nurse is examining an 8-year-old boy with tachycardia and tachypnea. The nurse anticipates which of the following as most helpful in determining the extent of the child's hypoxia? A) Pulmonary function test B) Pulse oximetry C) Peak expiratory flow D) Chest radiograph 4. A rapid strep test has confirmed that a 5-year-old girl has a group A Streptococcus infection. When teaching the parents about measures to implement, which of the following would be the least immediate concern? A) Using a cool mist humidifier B) Encouraging the child to drink liquids C) Discarding the child's toothbrush D) Administering antibiotic therapy 5. A nurse is administering 100% oxygen to a child with a pneumothorax based on the understanding that this treatment is used primarily for which reason? A) Improve gas exchange B) Bypass the obstruction C) Hasten air reabsorption D) Prevent hypoxemia 6. Bacterial pneumonia is suspected in a 4-year-old boy with fever, headache, and chest pain. Which assessment finding would most likely indicate the need for this child to be hospitalized? A) Fever B) Oxygen saturation level of 96% C) Tachypnea with retractions D) Pale skin color 7. The nurse is assessing a 5-year-old girl who is anxious, has a high fever, speaks in a whisper, and sits up with her neck thrust forward. Based on these findings, which of the following would be least appropriate for the nurse to perform? A) Providing 100% oxygen B) Visualizing the throat C) Having the child sit forward D) Auscultating for lung sounds 8. The nurse is educating the parents of a 7-year-old boy with asthma about the medications that have been prescribed. Which of the following drugs would the nurse identify as an adjunct to a b2-adrenergic agonist for treatment of bronchospasm? A) Ipratropium B) Montelukast C) Cromolyn D) Theophylline 9. The nurse is caring for a 3-year-old girl with a respiratory disorder. The nurse anticipates the need for providing supplemental oxygen to the child when performing which of the following? A) Suctioning a tracheostomy tube B) Administering drugs with a nebulizer C) Providing tracheostomy care D) Suctioning with a bulb syringe 10. The nurse is examining a 5-year-old boy. Which of the following signs or symptoms is a reliable first indication of respiratory illness in children? A) Slow, irregular breathing B) A bluish tinge to the lips C) Increasing lethargy D) Rapid, shallow breathing 11. A child requires supplemental oxygen therapy at 8 liters per minute. Which delivery device would the nurse most likely expect to be used? A) Simple mask B) Venturi mask C) Nasal cannula D) Oxygen hood 12. A group of nursing students are reviewing information about the variations in respiratory anatomy and physiology in children in comparison to adults. The students demonstrate understanding of the information when they identify which of the following? A) Children's demand for oxygen is lower than that of adults. B) Children develop hypoxemia more rapidly than adults do. C) An increase in oxygen saturation leads to a much larger decrease in pO2. D) Children's bronchi are wider in diameter than those of an adult. 13. The nurse is providing care to several children who have been brought to the clinic by the parents for complaints of cold-like symptoms. The nurse would most likely suspect sinusitis in which child? A) A 2-year-old with thin watery nasal discharge B) A 3-year-old with sneezing and coughing C) A 5-year-old with nasal congestion and sore throat D) A 7-year-old with halitosis and thick, yellow nasal discharge 14. A parent asks the nurse about immunizing her 7-month-old daughter against the flu. Which response by the nurse would be most appropriate? A) “She really doesn't need the vaccine until she reaches 1 year of age.” B) “She will probably receive it the next time she is to get her routine shots.” C) “Since your daughter is older than 6 months, she should get the vaccine every year.” D) “The vaccine has many side effects, so she wouldn't get it until she's ready to go to school.” 15. A nurse is preparing a teaching program for a parenting group about preventing foreign body aspiration. Which of the following would the nurse include? A) Avoid giving popcorn to children younger than the age of 2 years. B) Withhold peanuts from children until they are at least 5 years of age. C) If an object fits through a standard toilet paper roll, the child can aspirate it. D) Keep pennies and dimes out of the child's reach; quarters do not pose a problem. 16. A child with a pneumothorax has a chest tube attached to a water seal system. When assessing the child, the nurse notices that the chest tube has become disconnected from the drainage system. Which of the following would the nurse do first? A) Notify the physician B) Apply an occlusive dressing C) Clamp the chest tube D) Perform a respiratory assessment 17. A nurse is preparing a teaching plan for the family of a child with allergic rhinitis. When describing the immune reaction that occurs, the nurse would identify the role of which immunoglobulin? A) IgA B) IgE C) IgG D) IgM 18. A group of nursing students are reviewing the medications used to treat asthma. The students demonstrate understanding of the information when they identify which agent as appropriate for an acute episode of bronchospasm? A) Salmeterol B) Albuterol C) Ipratropium D) Cromolyn 19. The nurse is preparing to perform a physical examination of a child with asthma. Which of the following techniques would the nurse be least likely to perform? A) Inspection B) Palpation C) Percussion D) Auscultation 20. A nursing instructor is preparing a class on chronic lung disease. Which of the following would the instructor include when describing this disorder? A) It is a result of cystic fibrosis. B) It is seen most commonly in premature infants. C) It typically affects females more often than males. D) It is characterized by bradypnea. 21. A nurse is teaching the parents of a child diagnosed with cystic fibrosis about medication therapy. Which of the following would the nurse instruct the parents to administer orally? A) Recombinant human DNase B) Bronchodilators C) Anti-inflammatory agents D) Pancreatic enzymes 22. When performing the physical examination of a child with c, which of the following would the nurse expect to assess? A) Dullness over the lung fields B) Increased diaphragmatic excursion C) Decreased tactile fremitus D) Hyperresonance over the liver 23. The nurse is preparing to provide tracheostomy care to an infant. After gathering the necessary equipment, which of the following would the nurse do next? A) Position the infant supine with a towel roll under the neck B) Cut the new tracheostomy ties to the appropriate length C) Cut the tracheostomy ties from around the tracheostomy tube D) Cleanse around the site of the tracheostomy with the prescribed solution 24. A child is brought to the emergency department by his parents because he suddenly developed a barking cough. Further assessment leads the nurse to suspect that the child is experiencing croup. Which of the following would the nurse have most likely assessed? A) High fever B) Dysphagia C) Toxic appearance D) Inspiratory stridor   Answer Key 1. A 2. C 3. B 4. C 5. C 6. C 7. B 8. A 9. A 10. D 11. A 12. B 13. D 14. C 15. C 16. C 17. B 18. B 19. B 20. B 21. D 22. C 23. A 24. D Chapter 41- Nursing Care of the Child With an Alteration in Perfusion / Cardiovascular Disorder 1. The nurse is conducting a physical examination of a child with a ventricular septal defect. Which finding would the nurse expect to assess? A) Right ventricular heave B) Holosystolic harsh murmur along the left sternal border C) Fixed split-second heart sound D) Systolic ejection murmur 2. The nurse is administering digoxin as ordered and the child vomits the dose. What should the nurse do next? A) Contact the physician B) Offer a snack and administer another dose C) Immediately administer another dose D) Administer next dose as ordered in 12 hours 3. The nurse is caring for an infant with suspected patent ductus arteriosus. Which of the following assessment findings would the nurse identify as helping to confirm this suspicion? A) Thrill at the base of the heart B) Harsh, continuous, machine-like murmur under the left clavicle C) Faint pulses D) Systolic murmur best heard along the left sternal border 4. The nurse is conducting a physical examination of a child with a suspected cardiovascular disorder. Which of the following findings would the nurse most likely expect to assess if the child had transposition of the great vessels? A) Significant cyanosis without presence of a murmur B) Abrupt cessation of chest output with an increase in heart rate/filling pressure C) Soft systolic ejection D) Holosystolic murmur 5. The nurse is assessing a child with suspected infective endocarditis. Which of the following assessment findings would the nurse interpret as a sign of extracardiac emboli? A) Pruritus B) Roth spots C) Delayed capillary refill D) Erythema marginatum 6. When conducting a physical examination of a child with suspected Kawasaki disease, which of the following would the nurse expect to assess? A) Hirsutism or striae B) Strawberry tongue C) Malar rash D) Café au lait spots 7. After teaching a group of students about acute rheumatic fever, the instructor determines that the teaching was successful when the students identify which of the following as an assessment finding? A) Janeway lesions B) Jerky movements of the face and upper extremities C) Black lines D) Osler nodes 8. A nurse is reviewing the medical record of a child and finds that the child has a grade III murmur. After auscultating the child's heart sounds, the nurse would document this murmur as which of the following? A) Loud without a thrill B) Loud with a precordial thrill C) Soft and easily heard D) Loud, audible with a stethoscope 9. The nurse is caring for a 2-month-old infant who has been diagnosed with acute heart failure. The nurse is providing teaching about nutrition. Which of the following statements by the mother indicates a need for further teaching? A) “The baby may need as much as 150 calories/kg/day.” B) “Small, frequent feedings are best if tolerated.” C) “I need to feed him every hour to make sure he eats enough.” D) “Gavage feedings may be required for now.” 10. The nurse is caring for an infant girl with a suspected cardiovascular disorder. Which of the following statements by the mother would warrant further investigation? A) “My baby does not make any grunting noises.” B) “The baby seems more comfortable over my shoulder.” C) “The baby usually drinks all of her bottle.” D) “I don't notice any rapid breathing patterns.” 11. Auscultation of a child's heart reveals a loud murmur with a precordial thrill. The nurse documents this as which of the following? A) Grade II B) Grade III C) Grade IV D) Grade V 12. After assessing a child's blood pressure, the nurse determines the pulse pressure and finds that it is narrowed. Which of the following would the nurse identify as associated with this finding? A) Aortic stenosis B) Patent ductus arteriosus C) Aortic insufficiency D) Complete heart block 13. A 9-year-old child has undergone a cardiac catheterization and is being prepared for discharge. The nurse is instructing the parents and child about postprocedure care. Which statement by the parents indicates that the teaching was successful? A) “This pressure dressing needs to stay on for 5 days from now.” B) “He can't eat but he can drink fluids for the next 24 hours.” C) “He should avoid taking a bath for about 3 days but he can shower.” D) “It's normal if he says he feels like his heart skipped a beat.” 14. A newborn is diagnosed with patent ductus arteriosus. The nurse anticipates that the physician will most likely order which medication? A) Alprostadil B) Heparin C) Indomethacin D) Spironolactone 15. The nurse is preparing a teaching plan for the parents of a child who has been diagnosed with a congenital heart defect. Which of the following would the nurse be least likely to include? A) Daily weight assessment B) Maintenance of strict bed rest C) Prevention of infection D) Signs of complications 16. After teaching a class about the hemodynamic characteristics of congenital heart disease, the instructor determines that the teaching has been successful when the class identifies which defect as an example of a disorder involving increased pulmonary blood flow? A) Tetralogy of Fallot B) Atrial septal defect C) Hypoplastic left heart syndrome D) Transposition of the great vessels 17. A 7-year-old child with a family history of cardiovascular disease is being screened for hyperlipidemia. When reviewing the child's laboratory test results, which total cholesterol level would be of significant concern? A) 120 mg/dL B) 150 mg/dL C) 180 mg/dL D) 210 mg/dL 18. A child with a suspected cardiovascular disorder is to undergo diagnostic testing and is scheduled for an echocardiogram. When explaining this test to the child, which of the following would the nurse most likely include? A) “This test will check the pattern of how your heart is beating.” B) “They'll take a picture of your chest to look at the heart's size.” C) “A special wand that picks up sound is used to check your heart.” D) “Small patches are attached to your chest to check the heart rhythm.” 19. The nurse is reviewing the medical record of a child with infective endocarditis. Which of the following would the nurse expect to find? Select all answers that apply. A) White blood cell count revealing leukopenia B) Microscopic hematuria with urinalysis C) Electrocardiogram with prolonged PR interval D) Lungs clear on auscultation E) Petechiae on palpebral conjunctiva 20. A child with heart failure is receiving supplemental oxygen. The nurse understands that in addition to improving oxygen saturation, this intervention also helps to do which of the following? A) Cause vasodilation B) Increase pulmonary vascular resistance C) Promote diuresis D) Mobilize secretions 21. The nurse is developing a plan of care for an infant with heart failure who is receiving digoxin. The nurse would hold the dose of digoxin and notify the physician if the infant's apical pulse rate was: A) 140 beats per minute B) 120 beats per minute C) 100 beats per minute D) 80 beats per minute 22. A nurse is working with an adolescent who is slightly overweight and was recently diagnosed with hypertension. They are discussing nutritional management. Which statement by the adolescent demonstrates understanding of the information? A) “I have to make sure that I don't eat a lot of salty foods.” B) “I can eat any amount at a meal as long as I don't eat between meals.” C) “I should eat plenty of fresh fruits and vegetables.” D) “If I skip breakfast, I can eat a much bigger lunch.” 23. A child is diagnosed with Kawasaki disease and is in the acute phase of the disorder. Which of the following would the nurse expect the physician to prescribe? Select all answers that apply. A) Intravenous immunoglobulin B) Ibuprofen C) Acetaminophen D) Aspirin E) Alprostadil 24. An infant with congenital heart disease is to undergo surgery to correct the defect. The mother states, “I guess I'm going to have to stop breastfeeding her.” Which response by the nurse would be most appropriate? A) “That's true, but we'll make sure she gets the best intravenous nutrition.” B) “Unfortunately, your baby needs more nutrients than what breast milk can provide.” C) “Breast milk may help to boost her immune system, so you can continue to use it.” D) “She won't be able to suck, so we have to give her fortified formula through a tube.” 25. During a follow-up visit, the parents of a 5-month-old infant diagnosed with congenital heart disease tell the nurse, “We're just so tired and emotionally spent. All these tests and examinations are overwhelming. We just want to have a normal life. We're so focused on the baby that it seems like our 3-year-old is lost in the shuffle.” Which nursing diagnosis would the nurse identify as most appropriate? A) Risk for delayed growth and development related to necessary treatments B) Deficient knowledge related to the care of a child with congenital heart disease C) Interrupted family processes related to demands of caring for the ill child D) Fear related to infant's cardiac condition and need for ongoing care Answer Key 1. B 2. D 3. B 4. A 5. B 6. B 7. B 8. A 9. C 10. B 11. C 12. A 13. C 14. C 15. B 16. B 17. D 18. C 19. B, C, E 20. A 21. D 22. C 23. A, C, D 24. C 25. C Chapter 42- Nursing Care of the Child With an Alteration in Bowel Elimination / Gastrointestinal Disorder 1. The nurse is teaching the mother of a 5-year-old boy with a history of impaction how to administer enemas at home. Which of the following responses from the mother indicates a need for further teaching? A) “I should position him on his abdomen with knees bent.” B) “He will require 250 to 500 mL of enema solution.” C) “I should wash my hands and then wear gloves.” D) “He should retain the solution for 5 to 10 minutes.” 2. The nurse is taking a health history of an 11-year-old girl with recurrent abdominal pain. Which of the following responses would lead the nurse to suspect irritable bowel syndrome? A) “I always feel better after I have a bowel movement.” B) “I don't take any medicine right now.” C) “The pain comes and goes.” D) “The pain doesn't wake me up in the middle of the night.” 3. The nurse is caring for a 3-year-old girl with short bowel syndrome as a result of trauma to the small intestine. The girl's mother is extremely anxious and tells the nurse she is afraid she will never learn how to care for her daughter at home. How should the nurse respond? A) “I will help you become an expert on your daughter's care.” B) “You must learn how to care for your daughter at home.” C) “You really need the support of your husband.” D) “There is a lot to learn and you need a positive attitude.” 4. The nurse is conducting a physical examination of a child with suspected Crohn disease. Which of the following findings would be the most highly suspicious of Crohn disease? A) Normal growth patterns B) Perianal skin tags or fissures C) Poor growth patterns D) Abdominal tenderness 5. The nurse is caring for an infant with a temporary ileostomy. As part of the plan of care, the nurse monitors for skin breakdown around the stoma. If redness occurs, which of the following would be most appropriate to promote healing and prevent further skin breakdown? A) Clean the area well with a scented diaper wipe. B) Apply a barrier/healing cream or paste on the skin. C) Use a barrier wafer (such as Stomahesive) to attach the appliance. D) Sanitize the area with an alcohol wipe after each diaper change. 6. The nurse is caring for a 4-year-old boy who has undergone an appendectomy. The child is unwilling to use the incentive spirometer. Which approach would be most appropriate to elicit the child's cooperation? A) “Can you cough for me please?” B) “You must blow in this or you might get pneumonia.” C) “If you don't try, I will have to get the doctor.” D) “Can you blow this cotton ball across the tray?” 7. A nurse is caring for a 14-year-old girl scheduled for a barium swallow/upper gastrointestinal (GI) series. Before providing instructions, which of the following would be the priority? A) Screening the girl for pregnancy B) Reminding her to drink plenty of fluids after the procedure C) Ordering a bowel prep D) Reminding the girl about potential light-colored stools 8. The nurse has developed a plan of care for a 12-month-old hospitalized with dehydration as a result of rotavirus. Which intervention would the nurse include in the plan of care? A) Encouraging consumption of fruit juice B) Offering Kool-Aid or popsicles as tolerated C) Encouraging milk products to boost caloric intake D) Maintaining the intravenous fluid rate as ordered 9. The nurse is caring for a 2-month-old with a cleft palate. The child will undergo corrective surgery at age 3 months. The mother would like to continue breastfeeding the baby after surgery and wonders if it is possible. How should the nurse respond? A) “There is a good chance that you will be able to breastfeed almost immediately.” B) “Breastfeeding is likely to be possible, but check with the surgeon.” C) “After the suture line heals, breastfeeding can resume.” D) “We will have to wait and see what happens after the surgery.” 10. The school nurse is working with a 10-year-old girl with recurrent abdominal pain. The girl's teacher has been less than understanding about the frequent absences and trips to the nurse's office. How should the nurse respond? A) “Be patient; she is trying some new medication.” B) “The pain she is having is real.” C) “The family is working toward improvement.” D) “Please do not add to this family's stress.” 11. When examining the abdomen of a child, which technique would the nurse use last? A) Auscultation B) Percussion C) Palpation D) Inspection 12. Which finding would lead the nurse to suspect that a child is experiencing moderate dehydration? A) Dusky extremities B) Tenting of skin C) Sunken fontanels D) Hypotension 13. The nurse is determining maintenance fluid requirements for a child who weighs 25 kg. How much fluid would the child need per day? A) 1,560 mL B) 1,600 mL C) 1,650 mL D) 1,700 mL 14. The parents of a child diagnosed with celiac disease ask the nurse what types of food they can offer their child. Which of the following would the nurse include in the teaching plan? A) Frozen yogurt B) Rye bread C) Creamed spinach D) Fruit juice 15. The nurse is providing care to a child with an intussusception. The child has a bowel movement and the nurse inspects the stool. The nurse would document the stool's appearance most likely as which of the following? A) Greasy B) Clay-colored C) Currant jelly–like D) Bloody 16. The mother of a 3-week-old infant old brings her daughter in for an evaluation. During the visit, the mother tells the nurse that her baby is spitting up after feedings. Which response by the nurse would be most appropriate? A) “We need to tell the doctor about this.” B) “Infants this age commonly spit up.” C) “Your daughter might have an allergy.” D) “Don't worry; you're just feeding her too much.” 17. A group of students are reviewing information about fluid balance and losses in children in comparison to adults. The students demonstrate a need for additional review when they identify which of the following? A) Children have a proportionately greater amount of body water than do adults. B) Fever plays a greater role in insensible fluid losses in infants and children. C) A higher metabolic rate plays a major role in increased insensible fluid losses. D) The infant's immature kidneys have a tendency to overconcentrate urine more. 18. An 8-month-old infant is brought to the clinic for evaluation. The mother tells the nurse that she has noticed some white patches on the infant's tongue that look like curdled milk after breastfeeding. The nurse suspects oral candidiasis (thrush). Which question would the nurse use to help confirm this suspicion? A) “Are you having breast pain when you nurse the baby?” B) “Has he had any dairy problems recently?” C) “Is he experiencing any vomiting lately?” D) “How have his stools been this past week?” 19. The parents of a 6-week-old boy come to the clinic for evaluation because the infant has been vomiting. The parents report that the vomiting has been increasing in frequency and forcefulness over the last week. The mother says, “Sometimes, it seems like it just bursts out of his mouth.” A diagnosis of hypertrophic pyloric stenosis is suspected. When performing the physical examination, which of the following would the nurse most likely find? A) Sausage-shaped mass in the upper midabdomen B) Hard, moveable, olive-shaped mass in the right upper quadrant C) Tenderness over the McBurney point in the right lower quadrant D) Abdominal pain in the epigastric or umbilical region 20. A nursing instructor is developing a class presentation about the medications used to treat peptic ulcer disease. Which drug class would the instructor be least likely to include in the presentation? A) Antibiotics B) Proton pump inhibitors C) Histamine antagonists D) Prokinetics 21. The parents of a boy diagnosed with Hirschsprung disease are anxious and fearful of the upcoming surgery. The mother states, “I'm worried about having to care for our son's ostomy.” Which intervention would be most helpful for the parents? A) Explaining to them about the diagnosis and surgery B) Having a wound, ostomy, and continence nurse meet with them C) Reinforcing that the ostomy will be temporary D) Teaching them about the medications used to slow stool output 22. The nurse is providing care to a child with pancreatitis. When reviewing the child's laboratory test results, which of the following would the nurse expect to find? Select all answers that apply. A) Leukocytosis B) Decreased C-reactive protein C) Elevated serum amylase levels D) Positive stool culture E) Decreased serum lipase levels 23. A child is scheduled for a lower endoscopy. Which of the following would the nurse include in the child's plan of care in preparation for this test? A) Explaining about the need to ingest barium B) Establishing an intravenous access for radionuclide administration C) Administering the prescribed bowel cleansing regimen D) Withholding prescribed proton pump inhibitors for 5 days before 24. A group of students are reviewing information about gallbladder disease in children. The students demonstrate a need for additional review when they identify which of the following? A) Cholesterol gallstones are more frequently found in males. B) Pigment stones are found primarily in the common bile duct. C) Pancreatitis is a common complication of cholecystitis in children. D) Cholecystitis is due to chemical irritation from obstructed bile flow. 25. After teaching the parents of a child diagnosed with celiac disease about nutrition, the nurse determines that the teaching was effective when the parents identify which foods as appropriate for their child? Select all answers that apply. A) Wheat germ B) Peanut butter C) Carbonated drinks D) Shellfish E) Jelly F) Flavored yogurt 26. A group of nursing students are reviewing information about inflammatory bowel disease in preparation for a class discussion on the topic. The students demonstrate understanding of the material when they identify which of the following as characteristic of Crohn disease? Select all answers that apply. A) Distributed in a continuous fashion. B) Most common between the ages of 10 to 20 years C) Elevated erythrocyte sedimentation rate D) Low serum iron levels E) Tenesmus F) Loss of haustra within bowel 27. After teaching the parents of a 6-year-old how to administer an enema, the nurse determines that the teaching was successful when they state that they will give how much solution to their child? A) 100 to 200 mL B) 200 to 300 mL C) 250 to 500 mL D) 500 to 1,000 mL Answer Key 1. A 2. A 3. A 4. B 5. B 6. D 7. A 8. D 9. B 10. B 11. C 12. C 13. B 14. D 15. C 16. B 17. D 18. A 19. B 20. D 21. B 22. A, C 23. C 24. A 25. B, C, D, E 26. B 27. D Chapter 43- Nursing Care of the Child With an Alteration in Urinary Elimination / Genitourinary Disorder 1. The nurse is caring for a child who is experiencing an acute renal transplant rejection and is to receive muromonab-CD3. Which of the following would the nurse most likely expect to assess after the first dose is administered? A) Fever with chills, chest tightness B) Cough, hyperkalemia C) Photosensitivity, gastrointestinal (GI) upset D) Urinary retention, decreased appetite 2. The nurse is visually inspecting a urine specimen from a 12-year-old boy. The nurse documents gross hematuria with a specimen of which color? A) Cloudy yellow B) Cola colored C) Pale to almost clear urine D) Light orange to moderately yellow colored 3. The nurse is caring for a 4-year-old with a suspected urinary tract infection. Which of the following would be most appropriate when obtaining a urine specimen from the child? A) “I will need a urine sample.” B) “Let your mom help you tinkle in this cup.” C) “Please tinkle in this cup right now.” D) “Please void in this cup instead of the toilet.” 4. The nurse is providing postsurgical care for an infant who has undergone a hypospadias repair. Which action by the nurse would be most important to help keep the area clean while maintaining proper position of the drainage tubing? A) Keeping the drainage tube taped in an upright position B) Administering antibiotics as ordered C) Administering analgesics as prescribed D) Using a double-diapering technique 5. The nurse is caring for an infant with bladder exstrophy. As part of the infant's preoperative plan of care, the nurse monitors for abdominal skin excoriation. Which action would be most appropriate for promoting healing and preventing further skin breakdown? A) Cleaning the area well with a scented diaper wipe B) Applying a barrier/healing cream or paste on skin C) Keeping the bladder moist and covered with a sterile bag D) Covering the area with sterile gauze pads after tub baths 6. The nurse is caring for a 4-year-old girl with vulvovaginitis. After explaining to the girl's mother how to help prevent subsequent episodes, which statement by the mother indicates a need for additional teaching? A) “She needs to wipe from front to back.” B) “I will make sure she changes her underwear every day.” C) “She should probably avoid bubble baths.” D) “I will help supervise her wiping after bowel movements.” 7. A nurse is caring for a 7-year-old girl scheduled for an intravenous pyelogram (IVP). Which of the following would be the priority before the test? A) Checking with the parents for any allergies B) Ensuring adequate hydration C) Giving the girl an enema D) Screening her for pregnancy 8. A 6-year-old child has undergone a renal transplant and is receiving cyclosporine. The nurse instructs the parents to be especially alert for which of the following? A) Weight loss B) Hypotension C) Signs of infection D) Hair loss 9. The nurse is taking a health history of a child with suspected acute poststreptococcal glomerulonephritis. Which of the following responses would alert the nurse to a confirmed risk factor for this condition? A) “She has been very healthy up to now.” B) “He just got over a head cold with laryngitis.” C) “My child is just 18 months old.” D) “My child has not been sick at all.” 10. The nurse is caring for a 12-year-old girl with nephrotic syndrome. The girl confides that she feels like a “freak” compared to her peers because of her weight, edema, and moon face. Which response by the nurse would be most appropriate? A) “Let's put you in touch with some other girls who are also having the same body changes.” B) “Luckily, this is just a temporary, unfortunate part of your condition; you need to accept it.” C) “Your real friends do not care about your appearance and just want you to get well.” D) “You are beautiful in your own way; what matters is what is on the inside.” 11. An 8-year-old girl is scheduled for a renal ultrasound. Which of the following would the nurse include in the plan of care when preparing the child for this test? A) Withholding food and fluids after midnight B) Checking the child for allergies to shellfish C) Ensuring the child has a full bladder D) Informing the child she should feel no discomfort 12. The nurse is preparing a teaching plan for the parents of a child with a urinary tract infection (UTI). Which of the following would the nurse encourage the parents to avoid? A) Liberal fluid intake B) Caffeine C) Cranberry juice D) Cotton underwear 13. The mother of a child with end-stage renal disease asks the nurse why her son is getting an injection of erythropoietin. When responding to the mother, the nurse incorporates understanding of which of the following as the rationale? A) To treat low calcium levels B) To stimulate growth in stature C) To stimulate red blood cell growth D) To correct acidosis 14. A child is diagnosed with hemolytic-uremic syndrome (HUS). Review of the child's laboratory test results would reveal which of the following? A) Decreased blood urea nitrogen (BUN) and creatinine B) Decreased platelets and leukocytosis C) Hypernatremia and hypokalemia D) Respiratory acidosis and proteinuria 15. After teaching the parents of a child with a hydrocele about this condition, which statement indicates that the teaching was successful? A) “If this gets worse and we don't treat it, our son could become infertile.” B) “This condition should gradually go away on its own.” C) “The surgeon is going to operate on him immediately.” D) “It's going to be difficult putting ice packs on his scrotum.” 16. A nurse is conducting a physical examination of an infant and observes the urethral opening on the dorsal side of the penis. The nurse documents this finding as which of the following? A) Hypospadias B) Epispadias C) Varicocele D) Hydrocele 17. A 15-year-old boy comes to the emergency department accompanied by his parents. The boy reports an abrupt onset of sudden pain on the right side of his scrotum. When asked to rate his pain on a scale of 1 to 10, with 10 being the most severe, the boy states, “It's a 12.” Further assessment reveals a blue-black swelling on the affected side. The nurse suspects testicular torsion and immediately notifies the physician based on the understanding of which of the following? The condition is a surgical emergency. The boy is at risk for sepsis Intravenous antibiotics need to be initiated. Renal failure is imminent. 18. The nurse is reviewing the laboratory test results of a child with nephrotic syndrome. Which of the following would the nurse least likely expect to find? A) Hyperlipidemia B) Hypoalbuminemia C) Decreased blood urea nitrogen (BUN) D) Hypoproteinemia 19. The nurse is applying a urine bag to a 15-month-old boy to collect a urine specimen. Which of the following would the nurse do first? A) Apply benzoin to the scrotal area B) Tuck the bag downward inside the diaper C) Pat the perineal area dry after cleaning D) Apply the narrow portion of the bag on the perineal space 20. A group of students are reviewing information about renal failure in children. The students demonstrate a need for additional teaching when they identify which agent as a potential contributor to renal failure? A) Vancomycin B) Gentamicin C) Co-trimoxazole D) Amoxicillin 21. A nurse is preparing a presentation for a local parent group about urinary tract infections (UTIs) in children. Which of the following would the nurse incorporate into the presentation as the most common cause? A) Klebsiella B) Escherichia coli C) Staphylococcus aureus D) Pseudomonas 22. A nurse is interviewing the parents of a child diagnosed with obstructive uropathy. Which statement by the parents would the nurse identify as significant? A) “She's been constipated quite a few times.” B) “We've noticed that her bed is wet in the morning.” C) “She had surgery to repair a problem with her anus.” D) “She had a bacterial skin infection about a week ago.” 23. The nurse is assessing a child with acute poststreptococcal glomerulonephritis. Which of the following would the nurse expect to assess? Select all answers that apply. A) Irritability B) Abdominal pain C) Hypertension D) Crackles E) Polyphagia 24. A nurse identifies a nursing diagnosis of impaired urinary elimination related to urinary tract infection. When developing the plan of care, which of the following would be most important for the nurse to do first? A) Develop a schedule for bladder emptying B) Encourage fluid intake C) Assess usual voiding patterns D) Monitor intake and output 25. While presenting a panel discussion to a group of parents about urinary tract infections in children, one of the parents asks the nurse, “Why would my daughter be more at risk than my son?” Which response by the nurse would be most accurate? A) “Girls have a smaller bladder size than boys do.” B) “A girl's urethra is closer to the rectal opening.” C) “A girl's urethra is longer than a boy's urethra.” D) “Her kidneys are less well protected.” 26. A child returns from surgery in which a stoma was created in the abdominal wall to the bladder. The nurse identifies this as which of the following? A) Vesicostomy B) Ureteral stent C) Continent urinary diversion D) Bladder augmentation 27. The nurse is preparing an 8-year-old girl for a cystoscopy. Which instruction would be most appropriate to give to the child? A) “You need to make sure that you don't go to the bathroom before the test.” B) “You might feel some burning when you go to the bathroom afterward.” C) “I'm going to have to put a tube into your bladder to empty it.” D) “I have to put a thick tight rubber band around your arm to get a blood specimen.” 28. The nurse is assessing a 5-year-old child's genitourinary system. Which of the following would the nurse document as a normal finding? Select all answers that apply. Labial fusion Round abdomen Positive bowel sounds Dullness over the spleen E) Undescended testicles   Answer Key 1. A 2. B 3. B 4. D 5. B 6. A 7. A 8. C 9. B 10. A 11. D 12. B 13. C 14. B 15. B 16. B 17. A 18. C 19. C 20. D 21. B 22. C 23. B, C, D 24. C 25. B 26. A 27. B 28. B, C, D Chapter 44- Nursing Care of the Child With an Alteration in Mobility / Neuromuscular or Musculoskeletal Disorder 1. The nurse is teaching the mother of a 5-year-old boy with a myelomeningocele who has developed a sensitivity to latex. Which of the following responses from his mother indicates a need for further teaching? “He needs to get a medical alert identification.” “I will need to discuss this with his caregivers.” “A product's label indicates whether it is latex-free.” “He must avoid all contact with latex.” 2. The nurse is caring for an 8-year-old boy with myasthenia gravis and is teaching his parents about the signs of cholinergic crisis. Which of the following responses by the parents indicates a need for further teaching? A) “Low blood pressure is a sign of crisis.” B) “He might have difficulty swallowing.” C) “He may start to sweat a lot.” D) “More saliva in the mouth is a common sign.” 3. The nurse is providing postoperative care for a 14-month-old girl who has undergone a myelomeningocele repair. The girl's mother is extremely anxious and tells the nurse she is afraid she will never learn how to care for her daughter at home. Which response by the nurse would be most appropriate? A) “I will help you become comfortable in caring for your daughter.” B) “You must learn how to care for your daughter at home.” C) “You will need to learn to collaborate with all the caregivers.” D) “There is a lot to learn, and you need a positive attitude.” 4. The nurse is caring for a 10-year-old with Duchenne muscular dystrophy. As part of the plan of care, the nurse focuses on maintaining his cardiopulmonary function. Which intervention would the nurse implement to best promote maximum chest expansion? A) Deep-breathing exercises B) Upright positioning C) Coughing D) Chest percussion 5. A 6-year-old boy with cerebral palsy has been admitted to the hospital for some tests. His condition is stable. The boy's mother remains with her son, but she is obviously exhausted and stressed. Which response by the nurse would be most appropriate? A) “Would you like me to bring you a blanket and pillow?” B) “You are doing such a wonderful job with your son.” C) “He's in good hands; consider going home to get some sleep.” D) “Are you planning to spend the night or to go home?” 6. A nurse is caring for a 14-year-old girl following myelography. Which of the following would be the priority nursing action? A) Monitoring for a decrease in spasticity B) Observing for signs of meningeal irritation C) Assessing motor function D) Observing for mental confusion or hallucinations 7. The nurse has developed a plan of care for a 6-year-old with muscular dystrophy. He was recently injured when he fell out of bed at home. Which intervention would the nurse suggest to prevent further injury? A) Recommend the bed's side rails be raised throughout the day and night. B) Suggest a caregiver be present continuously to prevent falls from bed. C) Encourage a loose restraint to be used when he is in bed. D) Recommend raising the bed's side rails when a caregiver is not present. 8. The nurse is caring for a 2-month-old with cerebral palsy. The infant is limp and flaccid with uncontrolled, slow, worm-like, writhing, and twisting movements. Which of the following would the nurse use when documenting these observations? A) Spastic B) Athetoid C) Ataxic D) Mixed 9. The nurse is caring for a child with a spinal cord injury and providing instruction to the parents on promoting skin integrity. Which response from the mother indicates a need for further teaching? A) “I need to monitor his skin at least twice a week.” B) “I must monitor skin affected by his adaptive equipment.” C) “He must change positions frequently.” D) “We must avoid harsh cleaning products.” 10. The nurse is teaching a group of students about myelinization in a child. Which statement by the students indicates that the teaching was successful? A) Myelinization is completed by 4 years of age. B) The process occurs in a head-to-toe fashion. C) The speed of nerve impulses slows as myelinization occurs. D) Nerve impulses become less specific in focus with myelinization. 11. When developing the plan of care for a child with cerebral palsy, which treatment would the nurse expect as least likely? A) Skeletal traction B) Physical therapy C) Orthotics D) Occupational therapy 12. A nurse is preparing a program for a group of parents about injury prevention. Which of the following would the nurse include as an important contributing factor for cervical spine injury in a child? A) Exposure to teratogens while in utero B) Immaturity of the central nervous system C) Increased mobility of the spine D) Incomplete myelinization 13. A child with Duchenne muscular dystrophy is to receive prednisone as part of his treatment plan. After teaching the child's parents about this drug, which statement by the parents indicates the need for additional teaching? A) “We should give this drug before he eats anything.” B) “We need to keep a close eye for possible infection.” C) “The drug should not be stopped suddenly.” D) “He might gain some weight with this drug.” 14. Which of the following would the nurse include in the preoperative plan of care for an infant with myelomeningocele? A) Positioning supine with a pillow under the buttocks B) Covering the sac with saline-soaked nonadhesive gauze C) Wrapping the infant snugly in a blanket D) Applying a diaper to prevent fecal soiling of the sac 15. Which of the following would the nurse expect to find initially in a child with Guillain- Barré syndrome? A) Symmetric flaccid weakness B) Ataxia C) Sensory disturbances D) Lower extremity pain 16. The nurse is assessing a newborn who was delivered after a prolonged labor due to an abnormal presentation. The newborn sustained a cranial nerve injury. The nurse would most likely expect to assess deficits related to which cranial nerve? A) Optic B) Facial C) Acoustic D) Trigeminal 17. A child with spastic cerebral palsy is to receive botulin toxin. The nurse prepares the child for administration of this drug by which route? A) Oral B) Subcutaneous injection C) Intramuscular injection D) Intravenous infusion 18. The nurse is assessing the neuromusculoskeletal system of a newborn. Which of the following would the nurse identify as an abnormal finding? A) Sluggish deep tendon reflexes B) Full range of motion in extremities C) Absence of hypotonia D) Lack of purposeful muscular control 19. The nurse is developing a teaching plan for the parents of a child with a myelomeningocele who will require clean intermittent catheterization. Which of the following would the nurse include? A) Applying petroleum jelly to lubricate the catheter B) Cleaning the reusable catheter with peroxide after each use C) Storing the reusable cleaned catheter in a brown paper bag D) Soaking the catheter in a vinegar and water solution to sterilize 20. A child with cerebral palsy has undergone surgery for placement of a baclofen pump. Which of the following would the nurse include when teaching the parents about caring for their child? A) Waiting 48 hours before allowing the child to take a tub bath B) Not allowing the child to sleep on his side for about 4 weeks C) Calling the physician if the child's temperature is over 100.5°F D) Discouraging the child from stretching or bending forward for 4 weeks 21. A group of students are reviewing information about neuromuscular disorders. The students demonstrate understanding of the information when they identify which of the following as examples of autoimmune neuromuscular disorders? Select all answers that apply. A) Cerebral palsy B) Guillain-Barré syndrome C) Myasthenia gravis D) Spinal muscular atrophy E) Dermatomyositis 22. A nursing instructor is preparing for a class discussion on spinal muscular atrophy (SMA). When describing type 2 SMA, which of the following would the instructor include? Select all answers that apply. A) Onset before 6 months of age B) Weakness most severe in shoulders and hips C) Difficulty with swallowing D) Slowly progressing condition E) Genetic disease with autosomal recessive inheritance 23. The nurse is assessing a child who is suspected of having Guillain-Barré syndrome. Which assessment findings would the nurse correlate as supporting this diagnosis? Select all answers that apply. A) Recent cytomegalovirus infection B) Hyperactive deep tendon reflexes C) Numbness in the lower extremities D) Sustained clonus E) Difficulty swallowing 24. A child with myasthenia gravis is brought to the emergency department by his parents. The parents have noticed a sudden increase in respiratory difficulty. The nurse suspects myasthenic crisis based on which statement by the parents? A) “We gave him an extra dose of his medication earlier today.” B) “He was coughing and had a slight fever yesterday and today.” C) “Things have been pretty stress-free lately.” D) “He's been resting when he gets tired.” 25. After teaching a class of nursing students about muscular dystrophy, the instructor determines that the teaching was successful when the students identify which type of muscular dystrophy as demonstrating an X-linked recessive pattern of inheritance? A) Limb-girdle B) Myotonic C) Distal D) Duchenne Answer Key 1. C 2. B 3. A 4. B 5. C 6. B 7. D 8. B 9. A 10. B 11. A 12. C 13. A 14. B 15. D 16. B 17. C 18. A 19. D 20. D 21. B, C, E 22. B, D, E 23. A, C, D 24. B 25. D Extra Questions: Musculoskeletal Disorder 1. An 8-year-old girl was diagnosed with a closed fracture of the radius at approximately 2 p.m. The fracture was reduced in the emergency department and her arm placed in a cast. At 11 p.m. her mother brings her back to the emergency department due to unrelenting pain that has not been relieved by the prescribed narcotics. Which action would be the priority? A) Notifying the doctor immediately B) Applying ice C) Elevating the arm D) Giving additional pain medication as ordered 2. The nurse is caring for an active 14-year-old boy who has recently been diagnosed with scoliosis. He is dismayed that a “jock” like himself could have this condition, and is afraid it will impact his spot on the water polo team. Which response by the nurse would best address the boy's concerns? A) “If you wear your brace properly, you may not need surgery.” B) “The good news is that you have very minimal curvature of your spine.” C) “Let's talk to another boy with scoliosis, who is winning trophies for his swim team.” D) “Let's talk to the doctor about your treatment options.” 3. The nurse is caring for a female infant with torticollis and is providing instructions to the parents about how to help their daughter. Which statement by the parents indicates a need for further teaching? A) “We must encourage our daughter to turn her head both ways.” B) “Flatness on one side of the head is a common side effect.” C) “We must apply firm pressure and stretching every other day.” D) “We will do a daily stretching regimen with multiple sessions.” 4. The nurse is caring for a 10-year-old in traction. While performing a skin assessment, the nurse notices that the skin over the calcaneus appears slightly red and irritated. Which action would the nurse do first? A) Reposition the child's foot on a pressure-reducing device B) Apply lotion to his foot to maintain skin integrity C) Make sure the skin is clean and dry D) Gently massage his foot to promote circulation 5. The nurse is caring for a 14-month-old boy with rickets who was recently adopted from overseas. His condition was likely a result of a diet very low in milk products. The nurse is providing teaching regarding treatment. Which response by the parents indicates a need for further teaching? A) “We must give him calcium and phosphorus with food every morning.” B) “He must take vitamin D as prescribed and spend some time in the sunlight.” C) “He must take calcium at breakfast and phosphorus at bedtime.” D) “We should encourage him to have fish, dairy, and liver if he will eat it.” 6. The nurse is caring for a 13-year-old boy in traction prior to surgery for slipped capital femoral epiphysis. He has been in an acute care setting for 2 weeks and will require an additional 10 days in the hospital. He is complaining that he feels isolated and is resisting further treatment. Which response by the nurse would be most appropriate? A) “I know it is boring, but you must remain immobile for 2 more weeks.” B) “If there are no complications, you only have 2 more weeks here.” C) “Let's come up with things to do like books, movies, games, and friends to visit.” D) “If you resist your treatment, your condition will only get worse.” 7. A nurse is caring for a 10-year-old girl following joint fluid aspiration. The nurse would expect to perform which of the following immediately after the procedure? A) Transporting the aspirated fluid to the lab within 30 minutes B) Encouraging the child to drink fluids postprocedure C) Applying cold therapy and a pressure dressing to the site D) Elevating the extremity on a heating pad with several pillows 8. The nurse is caring for a 2-year-old girl in a bilateral brace with tibia vara. Her parents are upset by their toddler's limited mobility. Which response by the nurse would be most appropriate? A) “If you don't follow the therapy, your daughter could develop severe bowing of her legs.” B) “It's important to use the brace or your daughter may need surgery.” C) “You are doing a great job. Let's put our heads together on how to keep her busy.” D) “You'll need to accept this since treatment may be required for several years.” 9. The nurse is conducting a physical examination of a child with suspected developmental dysplasia of the hip. Which of the following findings would help confirm this diagnosis? A) Abduction occurs to 75 degrees and adduction to within 30 degrees (with stable pelvis). B) A distinct “clunk” is heard with Barlow and Ortolani maneuvers. C) A high-pitched “click” is heard with hip flexion or extension. D) The thigh and gluteal folds are symmetric. 10. The nurse is caring for an infant with osteogenesis imperfecta and is providing instruction on how to reduce the risk of injury. Which response from the mother indicates a need for further teaching? A) “I need to avoid pushing or pulling on an arm or leg.” B) “I must carefully lift the baby from under the armpits.” C) “I should not bend an arm or leg into an awkward position.” D) “We must avoid lifting the legs by the ankles to change diapers.” 11. The nurse is developing a teaching plan for a child who is to have his cast removed. Which of the following would the nurse most likely include? A) Applying petroleum jelly to the dry skin B) Rubbing the skin vigorously to remove the dead skin C) Soaking the area in warm water every day D) Washing the skin with dilute peroxide and water 12. When teaching a group of students about the skeletal development in children, which of the following would the instructor include? A) The growth plate is made up of the epiphysis. B) A young child's bones commonly bend instead of break with an injury. C) The infant's skeleton has undergone complete ossification by birth. D) Children's bones have a thin periosteum and limited blood supply. 13. A nurse is assessing a child with suspected osteomyelitis. Which finding would help support this suspicion? A) Swelling and point tenderness B) Decreased erythrocyte sedimentation rate C) Coolness of the affected site D) Increased range of motion 14. The school nurse is presenting a class to a group of students about common overuse disorders. Which of the following would the school nurse include? A) Dislocated radial head B) Transient synovitis of the hip C) Osgood-Schlatter disease D) Scoliosis 15. The nurse is assessing a child with a possible fracture. Which of the following would the nurse identify as the most reliable indicator? A) Lack of spontaneous movement B) Point tenderness C) Bruising D) Inability to bear weight 16. Which of the following would the nurse be least likely to assess in a 6-year-old with septic arthritis of the hip? A) Moderate to severe pain of the affected hip B) Previous otitis media infection C) Refusal to straighten the affected extremity D) Full range of motion of the hip 17. An 8-year-old boy with a fractured forearm is to have a fiberglass cast applied. Which of the following would the nurse include when teaching the child about the cast? A) The cast will take a day or two to dry completely. B) The edges will be covered with a soft material to prevent irritation. C) The child initially may experience a very warm feeling inside the cast. D) The child will need to keep his arm down at his side for 48 hours. 18. A child has undergone surgery using steel bar placement to correct pectus excavatum. Which of the following would the nurse instruct the parents to avoid? A) Semi-Fowler B) Supine C) High Fowler D) Side-lying 19. A newborn is diagnosed with metatarsus adductus. The parents ask the nurse about how this occurred. Which response by the nurse would be most appropriate? A) “This condition is due to a genetic defect in the bones.” B) “It's most likely from how the baby was positioned in utero.” C) “They really don't know what causes this condition.” D) “There is probably an underlying deformity of the baby's hip.” 20. A nurse is preparing a presentation for a parent group about musculoskeletal injuries. When describing a child's risk for this type of injury, the nurse integrates knowledge that bone growth occurs primarily in which area? A) Growth plate B) Epiphysis C) Physis D) Metaphysis 21. A group of nursing students are reviewing information about the type of skin and skeletal traction. The students demonstrate understanding of this information when they identify which of the following as a type of skeletal traction? A) Russell traction B) Bryant traction C) Buck traction D) 90-90 traction 22. The nurse is providing care to a child with a long-leg hip spica cast. Which of the following would be a priority nursing diagnosis? A) Risk for impaired skin integrity due to cast and location B) Deficient knowledge related to cast care C) Risk for delayed development related to immobility D) Self-care deficit related to immobility 23. A nurse is providing instructions to the parents of a 3-month-old with developmental dysplasia of the hip who is being treated with a Pavlik harness. Which statement by the parents demonstrates understanding of the instructions? Select all answers that apply. A) “We need to adjust the straps so that they are snug but not too tight.” B) “We should change her diaper without taking her out of the harness.” C) “We need to check the area behind her knees for redness and irritation.” D) “We need to send the harness to the dry cleaners to have it cleaned.” E) “We need to call the doctor if she is not able to actively kick her legs.” 24. When assessing a child for slipped capital femoral epiphysis, which of the following would the nurse identify as a possible risk factor? Select all answers that apply. A) Age younger than 8 years B) African American ethnicity C) History of cystic fibrosis D) Excessive activity E) Obesity 25. The nurse is reviewing the medical record of a child diagnosed with septic arthritis. Which of the following would the nurse expect to find? Select all answers that apply. A) Elevated neutrophil count B) Decreased C-reactive protein level C) Joint fluid with increased white blood cells D) Decreased joint space with radiograph E) Increased erythrocyte sedimentation rate 26. The nurse is assessing an 11-year-old girl with scoliosis. Which of the following would the nurse expect to find? Select all answers that apply. A) Complaints of severe back pain B) Asymmetric shoulder elevation C) Even curve at the waistline D) Pronounced one-sided hump on bending over E) Diminished motor function F) Hyperactive reflexes 27. An 18-month old was brought to the emergency department by her mother, who states, “I think she broke her arm.” The child is sent for a radiograph to confirm the fracture. Additional assessment of the child leads the nurse to suspect possible child abuse. Which type of fracture would the radiograph most likely reveal? A) Plastic deformity B) Buckle fracture C) Spiral fracture D) Greenstick fracture 28. A 10-year-old girl is brought to the emergency department by her father after tripping over a rock while running in the yard. She tells the nurse, “I think I twisted my ankle.” When assessing the child, which of the following would the nurse most likely assess? A) Bruising B) Edema C) Limited range of motion D) Absent pulse Answer Key - Extra Section 1. A 2. C 3. C 4. A 5. A 6. C 7. C 8. C 9. B 10. B 11. C 12. B 13. A 14. C 15. B 16. D 17. C 18. D 19. B 20. B 21. D 22. A 23. B, C, E 24. B, E 25. A, C, E 26. B, D 27. C 28. B Chapter 45- Nursing Care of the Child With an Alteration in Tissue Integrity / Integumentary Disorder 1. The nurse is teaching the mother of a toddler about burn prevention. Which response by the mother indicates a need for further teaching? A) “We will leave fireworks displays to the professionals.” B) “I will set our water heater at 130 degrees.” C) “All sleepwear should be flame retardant.” D) “The handles of pots on the stove should face inward.” 2. The nurse is providing parental teaching about home care for an 8-year-old boy with widespread sunburn on his back and shoulders. Which of the following responses indicates a need for further teaching? A) “Cool compresses may help cool the burn.” B) “He should manually peel off any flaking skin.” C) “Nonsteroidal anti-inflammatory drugs like ibuprofen are helpful.” D) “He should avoid hot showers or baths for a couple of days.” 3. The nurse is providing care for a 14-year-old girl with severe acne. The girl expresses sadness and distress about her appearance. Which response by the nurse would be most appropriate? A) “Are you using your medicine every day?” B) “Your condition will most likely improve in a year or two.” C) “Many people feel this way; I know someone who can help.” D) “If you have any scarring you can undergo dermabrasion.” 4. The nurse is conducting a physical examination of a 9-month-old baby with a flat, discolored area on the skin. The nurse documents this as which of the following? A) Papule B) Macule C) Vesicle D) Scaling 5. A nurse is caring for a 5-year-old in Buck traction. When conducting a skin examination for signs of pressure ulcers, the nurse pays particular attention to which area? A) Sacral area B) Hip area C) Occiput D) Upper arm 6. A 6-year-old boy has been admitted to the hospital with burns. The nurse notes carbonaceous sputum. Which of the following would be the priority? A) Determining the burn depth B) Eliciting a description of the burn C) Estimating burn extent D) Ensuring a patent airway 7. A nurse is caring for a 14-year-old girl who received an electrical burn. The nurse would anticipate preparing the girl for which diagnostic tests as ordered? A) Pulse oximetry B) Fiberoptic bronchoscopy C) Xenon ventilation–perfusion scanning D) Electrocardiographic monitoring 8. The nurse is caring for an infant with candidal diaper rash. Which topical agent would the nurse expect the physician to order? A) Corticosteroids B) Antifungals C) Antibiotics D) Retinoids 9. The nurse is caring for a 15-year-old boy who has sustained burn injuries. The nurse observes the burn developing a violaceous color with discharge and a foul odor. The nurse suspects which of the following infections? A) Burn wound cellulitis B) Invasive burn cellulitis C) Burn impetigo D) Staphylococcal scalded skin syndrome 10. The nurse is caring for a child with widespread itching and has recommended bathing as a relief measure. After teaching the mother about this, which statement from the mother indicates a need for further instruction? A) “After bathing, I need to rub his skin everywhere to make sure he is completely dry.” B) “I must make sure I use lukewarm water instead of hot water.” C) “Oatmeal baths are helpful; we can add Aveeno skin relief bath treatment.” D) “We should leave his skin moist before applying medication or moisturizer.” 11. After teaching a class about the differences in the skin of infants and adults, the nurse determines that additional teaching is necessary when the class states which of the following? A) “An infant's skin is thinner than an adult's, so substances placed on the skin are absorbed more readily.” B) “The infant's epidermis is loosely connected to the dermis, increasing the risk for breakdown.” C) “The infant has a lower risk for damage from ultraviolet radiation because the skin is more pigmented.” D) “An infant has less subcutaneous fat, which places the infant at a higher risk for heat loss.” 12. The nurse is preparing a class for a group of adolescents about reducing the risk of skin cancer. Which of the following would the nurse include? A) Using a sunscreen with para-aminobenzoic acid (PABA) with an SPF of at least 10 B) Applying sunscreen at least 1 hour before going outside in the sun C) Avoiding sun exposure between the hours of 10 a.m. and 2 p.m. D) Using artificial UV tanning beds instead of sun exposure 13. A nurse is assessing the skin of a child with cellulitis. Which of the following would the nurse expect to find? A) Red raised hair follicles B) Warmth at skin disruption site C) Papules progressing to vesicles D) Honey-colored exudate 14. When developing the plan of care for a child with burns requiring fluid replacement therapy, which of the following would the nurse expect to include? A) Administration of colloid initially followed by a crystalloid B) Determination of fluid replacement based on the type of burn C) Administration of most of the volume during the first 8 hours D) Monitoring of hourly urine output to achieve less than 1 mL/kg/hour 15. Which of the following would the nurse include when teaching an adolescent about tinea pedis? A) “Keep your feet moist and open to the air as much as possible.” B) “Dry the area between your toes really well.” C) “Wear nylon or synthetic socks every day.” D) “Go barefoot when you are in the locker room at school.” 16. A child is diagnosed with atopic dermatitis. Which laboratory test would the nurse expect the child to undergo to provide additional evidence for this condition? A) Erythrocyte sedimentation rate B) Potassium hydroxide prep C) Wound culture D) Serum immunoglobulin E level 17. The nurse is providing care to a child with folliculitis. Which of the following would the nurse expect to administer? A) Topical mupirocin B) Oral cephalosporin C) Intravenous oxacillin D) Topical Eucerin cream 18. A nurse is preparing a class for parents of infants about managing diaper dermatitis. Which of the following would the nurse include in the presentation? Select all answers that apply. A) Applying topical nystatin to the diaper area B) Using a blow dryer on warm to dry the diaper area C) Refraining from using rubber pants over diapers D) Using scented diaper wipes to clean the area E) Washing the diaper area with an antibacterial soap 19. A group of students are preparing for a class exam on skin disorders. As part of their preparation, they are reviewing information about acne vulgaris and its association with increased sebum production. The students demonstrate understanding of the information when they identify which areas as having the highest sebaceous gland activity? Select all answers that apply. A) Face B) Upper chest C) Neck D) Back E) Shoulders 20. An instructor is developing a plan for a class of nursing students on the various skin disorders. When describing urticaria, which of the following would the instructor include? A) It is a type IV hypersensitivity reaction. B) Histamine release leads to vasodilation C) Wheals appear first followed by erythema. D) The nonpruritic rash blanches with pressure. 21. A nurse is inspecting the skin of a child with atopic dermatitis. Which of the following would the nurse expect to observe? A) Erythematous papulovesicular rash B) Dry, red, scaly rash with lichenification C) Pustular vesicles with honey-colored exudates D) Hypopigmented oval scaly lesions 22. A nurse is performing a primary survey on a child who has sustained partial thickness burns over his upper body areas. Which of the following would the nurse do first? A) Inspect the child's skin color B) Assess for a patent airway C) Observe for symmetric breathing D) Palpate the child's pulse 23. A 3-year-old child has sustained significant severe burns and is ordered to receive 100% oxygen. Which of the following would the nurse use to administer the oxygen? A) Nasal cannula B) Venturi mask C) Nonrebreather mask D) Oxygen hood 24. As part of a clinical conference with a group of nursing students, the instructor is describing the burn classification. The instructor determines that the teaching has been successful when the group identifies which of the following as characteristic of full thickness burns? A) Skin that is reddened, dry, and slightly swollen B) Skin appearing wet with significant pain C) Skin with blistering and swelling D) Skin that is leathery and dry with some numbness 25. A 4-year-old is brought to the emergency department with a burn. Which of the following would alert the nurse to the possibility of child abuse? A) Burn assessment correlates with mother's report of contact with a portable heater. B) Parents state that the injury occurred approximately 15 to 20 minutes ago. C) Clear delineations are noted between burned and nonburned skin areas. D) The burn area appears asymmetric and nonuniform. 26. A nurse is preparing a presentation for a local parent group about burn prevention and care in children. Which of the following would the nurse be least likely to include in the presentation when describing how to care for a superficial burn? A) Using cool water over the burned area until the pain lessens B) Applying ice directly to the burned skin area C) Covering the burn with a clean, nonadhesive bandage D) Giving the child acetaminophen for pain relief Answer Key 1. B 2. B 3. C 4. B 5. C 6. D 7. D 8. B 9. B 10. A 11. C 12. C 13. B 14. C 15. B 16. D 17. A 18. B, C 19. A, B, D 20. B 21. B 22. B 23. C 24. D 25. C 26. B Chapter 46- Nursing Care of the Child With an Alteration in Cellular Regulation / Hematologic or Neoplastic Disorder 1. The nurse is conducting a physical examination of a toddler with suspected lead poisoning. Lab results indicate blood lead level 52 m g/dL. Which action would the nurse expect to happen next? A) Repeat testing within 2 days and prepare to begin chelation therapy as ordered B) Repeat testing within 1 week with education to decrease lead exposure C) Confirm with repeat testing in 1 month and referral to local health department D) Prepare to admit child to begin chelation therapy 2. A nurse is conducting a physical examination of a 5-year-old with suspected iron- deficiency anemia. How would the nurse evaluate for changes in neurologic functioning? A) “Open your mouth so I can look inside your cheeks and lips.” B) “Do you have any bruises on your feet or shins?” C) “Will you show me how you walk across the room?” D) “Let me see the palms of your hands and soles of your feet.” 3. The nurse is caring for a 12-year-old boy with idiopathic thrombocytopenia. The nurse is providing discharge instructions about home care and safety recommendations to the boy and his parents. Which response indicates a need for further teaching? A) “We should avoid aspirin and drugs like ibuprofen.” B) “He can resume participation in football in 2 weeks.” C) “Swimming would be a great activity.” D) “Our son cannot take any antihistamines.” 4. The nurse is assessing a child with suspected thalassemia. Which of the following would the nurse expect to assess? A) Dactylitis B) Frontal bossing C) Presence of clubbing D) Presence of spooning 5. The nurse is caring for a child recently diagnosed with glucose-6-phosphate dehydrogenase (G6PD) deficiency. The nurse is teaching the parents about triggers that may result in oxidative stress. Which of the following responses indicates a need for further teaching? A) “I doubt he will ever eat fava beans, but they could trigger hemolysis.” B) “He must avoid exposure to naphthalene, an agent found in mothballs.” C) “He must never take methylene blue for a urinary tract infection.” D) “My son can never take penicillin for an infection.” 6. The nurse is caring for a 13-year-old girl with von Willebrand disease. After teaching the adolescent and her parents about this disorder and care, which response by the parents indicates a need for additional teaching? A) “We need to administer Stimate prior to dental work.” B) “We should be aware that she may suffer from menorrhagia.” C) “We should administer desmopressin as often as needed.” D) “We understand that she may have frequent nosebleeds.” 7. The nurse is caring for a child who has been admitted for a sickle cell crisis. Which of the following would the nurse do first to provide adequate pain management? A) Administer a nonsteroidal anti-inflammatory drug as ordered. B) Use guided imagery and therapeutic touch. C) Administer meperidine as ordered. D) Initiate pain assessment with a standardized pain scale. 8. The nurse is caring for a 2-year-old boy with hemophilia. His parents are upset by the possibility that he will become infected with hepatitis or HIV from the clotting factor replacement therapy. Which response by the nurse would be most appropriate? A) “Parents commonly fear the worst; however, the factor will help your child lead a normal life.” B) “There are risks with any treatment including using blood products, but these are very minor.” C) “Although factor replacement is expensive, there's more financial strain from missing work if he has a bleeding episode.” D) “Since dry heat treatment of the factor began in 1986, there have been no reports of virus transmission.” 9. A child with iron-deficiency anemia is prescribed ferrous fumarate, 3 mg/kg/day in two divided doses. The nurse interprets this order as indicating which of the following? A) The child requires a prophylactic dose of iron. B) The child has mild to moderate iron deficiency. C) The child has severe iron deficiency. D) The child is being prepared for packed red blood cell administration. 10. The nurse is caring for a child with thalassemia who is receiving chelation therapy at home using a battery-operated pump. After teaching the parents about this treatment, which statement by the mother indicates a need for additional teaching? A) “I can have the nurse administer the chelation therapy if I am uncomfortable.” B) “I must be very careful to strictly adhere to the chelation regimen.” C) “The deferoxamine binds to the iron so it can be removed from the body.” D) “The medication can be administered while my child is sleeping.” 11. The nurse is teaching the parents of a child with a hematologic disorder about the functions of the various blood cells. The nurse determines that the teaching was successful when the parents state which blood cell as being primarily responsible for blood clotting? A) Granulocytes B) Erythrocytes C) Thrombocytes D) Leukocytes 12. When evaluating the hemogram of an 8-month-old infant, the nurse would identify which type of hemoglobin as being the predominant type? A) Hemoglobin A B) Hemoglobin F C) Hemoglobin A2 D) Hemoglobin S 13. The nurse is evaluating the laboratory test results of a 7-year-old child with a suspected hematologic disorder. Which finding would cause the nurse to be concerned? A) WBC: 5.6 × 103/mm3 B) RBC: 2.8 × 106/mm3 C) Hemoglobin: 11.4 mg/dL D) Hematocrit: 35% 14. When providing care to a child with aplastic anemia, which nursing diagnosis most likely would be the priority? A) Risk for injury B) Imbalanced nutrition, less than body requirements C) Ineffective tissue perfusion D) Impaired gas exchange 15. The nurse is reviewing the laboratory test results of a child diagnosed with disseminated intravascular coagulation (DIC). Which of the following would the nurse interpret as indicative of this disorder? A) Shortened prothrombin time B) Increased fibrinogen level C) Positive fibrin split products D) Increased platelets 16. A nurse is providing care to a child with idiopathic thrombocytopenic purpura with a platelet count of 18,000/mm3. Which medication would the nurse most likely expect to be ordered? A) Folic acid B) Intravenous immune globulin C) Dimercaprol D) Deferoxamine 17. The nurse is reviewing the white blood cell differential of a 4-year-old girl. Which value would lead the nurse to be concerned? A) Bands: 8% B) Segs: 28% C) Eosinophils: 10% D) Basophils: 0% 18. A child with suspected sickle cell disease is scheduled for a hemoglobin electrophoresis. When reviewing the child's history, which of the following would the nurse identify as potentially interfering with the accuracy of the results? A) Use of iron supplementation B) Blood transfusion 1 month ago C) Lack of fasting for 12 hours D) History of recent infection 19. The mother of a 5-year-old girl brings the child to the clinic for an evaluation. The mother tells the nurse, “She seems to be so tired and irritable lately. And she looks so pale.” Further assessment reveals pale conjunctiva and oral mucous membranes. The nurse suspects irondeficiency anemia. Which additional finding would help provide additional evidence for this suspicion? A) Spooned nails B) Negative splenomegaly C) Oxygen saturation: 99% D) Bradycardia 20. The nurse is teaching the parents of a child diagnosed with iron-deficiency anemia about ways to increase their child's intake of iron. The parents demonstrate understanding of the teaching when they identify which foods as good choices for the child? Select all that apply. A) Tuna B) Salmon C) Tofu D) Cow's milk E) Dried fruits 21. A child is prescribed monthly injections of vitamin B12. When developing the teaching plan for the family, the nurse would focus on which type of anemia? A) Aplastic anemia B) Pernicious anemia C) Folic acid anemia D) Sickle cell anemia 22. A nurse is leading a discussion with a group of new mothers about newborn nutrition and its importance for growth and development. One of the mothers asks, “Doesn't the baby get iron from me before birth?” Which response by the nurse would be most appropriate? A) “You give the baby some iron, but it is not enough to sustain him after birth.” B) “Because the baby grows rapidly during the first months, he uses up what you gave him.” C) “The iron you give him before birth is different from what he needs once he is born.” D) “If the baby didn't use up what you gave him before birth, he excretes it soon after birth.” 23. A 5-year-old girl is diagnosed with iron-deficiency anemia and is to receive iron supplements. The child has difficulty swallowing tablets, so a liquid formulation is prescribed. After teaching the parents about administering the iron supplement, which statement indicates the need for additional teaching? A) “She needs to eat foods that are high in fiber so she doesn't get constipated.” B) “We'll try to get her to drink lots of fluids throughout the day.” C) “We will place the liquid in the front of her gums, just below her teeth.” D) “We need to measure the liquid carefully so that we give her the correct amount.” 24. The nurse is assessing a child with aplastic anemia. Which of the following would the nurse expect to assess? Select all that apply. A) Ecchymoses B) Tachycardia C) Guaiac-positive stool D) Epistaxis E) Severe pain F) Warm tender joints 25. An adolescent weighing 55 kg is admitted to the hospital experiencing a sickle cell crisis. Intravenous fluid therapy as well as increased oral fluids are ordered as part of his treatment plan. Based on the understanding of the amount of fluids needed to promote hemodilution, the nurse would expect the adolescent to receive how much total fluid in 24 hours? mL 26. The nurse is developing a plan of care for a child with thalassemia. Which of the following would the nurse expect to include? Select all that apply. A) Packed RBC transfusions B) Deferoxamine therapy C) Heparin therapy D) Opioid analgesics E) Platelet transfusions F) Intravenous immunoglobulin 27. A group of students are reviewing information about glucose-6-phosphate dehydrogenase (G6PD) deficiency. The students demonstrate understanding of the material when they identify this disorder as due to which of the following? A) X-linked recessive inheritance B) Deficiency in clotting factors C) An excess supply of iron D) Autosomal recessive inheritance Answer Key 1. A 2. C 3. B 4. B 5. D 6. C 7. D 8. D 9. B 10. A 11. C 12. A 13. B 14. A 15. C 16. B 17. C 18. B 19. A 20. A, B, C, E 21. B 22. B 23. C 24. A, B, C, D 25. 8,250 26. A, B 27. A Extra Questions: Neoplastic Disorder 1. The nurse is caring for a 16-year-old boy with acute myelogenous leukemia who is having chemotherapy and who has incomplete records for varicella zoster immunization. Which of the following will be the priority nursing diagnosis? A) Pain related to adverse effects of treatment verbalized by the child B) Nausea related to side effects of chemotherapy verbalized by the child C) Constipation related to the use of opioid analgesics for pain D) Risk for infection related to neutropenia and immunosuppression 2. The nurse is caring for an 8-year-old girl who has been diagnosed with leukemia and will have a variety of tests, including a lumbar puncture, before beginning chemotherapy. Which of the following would be the priority? A) Applying EMLA to the lumbar puncture site B) Educating the child and family about the testing procedures C) Administering promethazine as ordered for nausea D) Educating the family about chemotherapy and its side effects 3. The nurse is caring for a 13-year-old boy with acute myelogenous leukemia who is experiencing feelings of powerlessness due to the effects of chemotherapy. Which of the following interventions will best help the teen's sense of control? A) Involving the boy in decisions whenever possible B) Acknowledging the boy's feelings of anger with the disease C) Providing realistic expectations of treatments and outcomes D) Recognizing abilities that are unaffected by the disease 4. The nurse is caring for a 5-year-old girl with a disseminated medulloblastoma. Which of the following interventions would be most appropriate for this situation? A) Providing emotional support to the parents and siblings of the child B) Recommending support groups for people whose children have cancer C) Encouraging the family to cry and express feelings away from the child D) Educating the family about the disease, its treatments, and side effects 5. The nurse is assessing a 3-year-old boy whose parents brought him to the clinic when they noticed that the right side of his abdomen was swollen. Which of the following findings would suggest this child has a neuroblastoma? A) The child has a maculopapular rash on his palms. B) The parents report that their son is vomiting and not eating well. C) The parents report that their son is irritable and not gaining weight. D) Auscultation reveals wheezing with diminished lung sounds. 6. The nurse is educating the parents of a 4-year-old boy with a Wilms tumor who is about to have chemotherapy prior to surgery. Which statement by the parents indicates that the nurse should review the instructions about preventing infection? A) “He takes his antibiotic twice a day.” B) “We check his temperature orally.” C) “We keep him away from crowds.” D) “He must be clean and his teeth brushed.” 7. The nurse is assessing a 13-year-old girl with a family history of kidney cancer who has come to the clinic complaining of abdominal pain, nausea, and vomiting. Which of the following findings would the nurse identify as least likely indicative of cancer in a child? A) The child reports rectal bleeding and diarrhea. B) Observation reveals an asymmetric abdomen. C) The child experiences a broken bone without trauma. D) Palpation determines an abdominal mass. 8. The nurse is caring for a 9-year-old boy who is having chemotherapy. The nurse is developing a teaching plan for the child and family about nutrition. Which of the following would the nurse be least likely to include? A) Emphasizing the intake of grains, fruits, and vegetables B) Featuring high-fiber foods if opioid analgesics are being taken C) Concentrating on consuming primarily high-calorie shakes and puddings D) Avoiding milk products if diarrhea is a problem 9. The nurse is caring for a 7-year-old girl who is undergoing a stem cell transplant. Which of the following would the nurse include in the child's postoperative plan of care? A) Assessing for petechiae, purpura, bruising, or bleeding B) Limiting blood draws to the minimum volume required C) Administering antiemetics around the clock as ordered D) Monitoring for severe diarrhea and maculopapular rash 10. The nurse is caring for a 5-year-old boy undergoing radiation treatment for a neuroblastoma. Which nursing diagnosis would be most applicable for this child? A) Activity intolerance related to anemia and weakness from medications B) Impaired skin integrity related to desquamation from cellular destruction C) Impaired oral mucosa related to the presence of oral lesions from malnutrition D) Imbalanced nutrition, less than body requirements related to nausea and vomiting 11. The nurse is planning a discussion group for parents with children who have cancer. Which of the following would the nurse include when describing the differences between cancer in children and adults? A) Most childhood cancers affect the tissues rather than organs. B) Childhood cancers are usually localized when found. C) Unlike adult cancers, childhood cancers are less responsive to treatment. D) The majority of childhood cancers can be prevented. 12. A child is receiving fluorouracil as part of a chemotherapy protocol. Which of the following would be most important for the nurse to include in the child's plan of care? A) Monitoring for visual changes B) Maintaining adequate hydration C) Using prescribed eye drops to prevent conjunctivitis D) Avoiding administration with food or meals 13. A child diagnosed with stage IV neuroblastoma has undergone abdominal surgery to remove the tumor. He is now receiving chemotherapy. Which nursing diagnosis would be most important? A) Risk for infection related to chemotherapy B) Impaired skin integrity related to abdominal surgery C) Grieving related to advanced disease and poor prognosis D) Imbalanced nutrition related to adverse effects of chemotherapy 14. Which of the following would be most appropriate to include in the plan of care for a child who has undergone surgery for removal of an astrocytoma? A) Elevating the foot of the bed B) Positioning the child on his unaffected side C) Raising the head of the bed at least 45 degrees D) Administering large volumes of intravenous fluids 15. Which test result would the nurse least likely expect to find in a child diagnosed with Wilms tumor? A) Complete blood count (CBC) within normal limits B) Urinalysis positive for blood C) Mass on kidney D) Elevated homovanillic acid (HVA) with 24-hour urine collection 16. The parents bring their 4-year-old son to the emergency department. The child is receiving chemotherapy for acute lymphoblastic leukemia. The parents report that the child has become lethargic and has had significant episodes of vomiting and diarrhea. The nurse suspects the child may be experiencing tumor lysis syndrome based on which of the following? Select all answers that apply. A) Hyperkalemia B) Hypophosphatemia C) Polyuria D) Hypocalcemia E) Hyperuricemia 17. The nurse is describing the phases of treatment to a child who was diagnosed with leukemia and his parents. The nurse explains the induction stage as which of the following? A) Intense therapy to strengthen remission B) Rapid promotion of complete remission C) Elimination of all residual leukemic cells D) Reduction of risk for central nervous system (CNS) disease 18. A 14-year-old boy is diagnosed with Hodgkin disease. When palpating for enlarged lymph nodes, the nurse would expect to find which nodes as most commonly enlarged? Select all answers that apply. A) Cervical B) Axillary C) Supraclavicular D) Occipital E) Inguinal 19. The nurse is preparing a presentation for a parent group about childhood cancers, focusing on brain tumors in children. Which of the following would the nurse include as being the most common type of brain tumor? A) Brain stem glioma B) Medulloblastoma C) Ependymoma D) Astrocytoma 20. A child is receiving methotrexate as part of his chemotherapy protocol. The nurse would anticipate administering which agent to counteract the toxic effects of methotrexate? A) Mesna B) Cyclosporine C) Leucovorin D) Nystatin 21. A group of nursing students are reviewing the various drug classes used for cancer chemotherapy. The students demonstrate an understanding of these classes when they identify which agent as an example of a nitrosourea? A) Busulfan B) Thiotepa C) Cisplatin D) Carmustine 22. The nurse is developing a plan of care for a child who is receiving cyclophosphamide. Which of the following would the nurse expect to include? A) Withholding food and fluids from the child during the infusion B) Encouraging frequent voiding during and after the infusion C) Monitoring for signs of anaphylaxis during infusion D) Assessing the child for complaints of bone pain 23. The nurse is reviewing the laboratory test results of a child who is receiving chemotherapy. To calculate the child's absolute neutrophil count, in addition to the total number of white blood cells, which results would the nurse use? Select all answers that apply. A) Bands B) Segs C) Eosinophils D) Basophils 24. A nurse is instituting neutropenic precautions for a child. Which of the following would the nurse most likely include? Select all answers that apply. A) Placing the child in a semiprivate room B) Avoiding rectal exams, suppositories, and enemas C) Placing a mask on the child when outside the room D) Encouraging an intake of raw fruits and vegetables E) Discouraging fresh flowers in the child's room. 25. A child is scheduled to undergo radiation therapy as part of his treatment plan for newly diagnosed cancer. After teaching the child and parents about this treatment, the nurse determines that additional teaching is needed when the parents state which of the following? A) “We should not wash off the markings on his skin.” B) “He can use petroleum jelly if the skin becomes reddened.” C) “He needs to use a sunscreen with an SPF of 30 or more.” D) “He should not apply deodorant to the treatment site.” Answer Key - Extra Section 1. D 2. B 3. A 4. C 5. B 6. A 7. A 8. C 9. D 10. B 11. A 12. B 13. C 14. B 15. D 16. A, D, E 17. B 18. A, C 19. B 20. C 21. D 22. B 23. A, B 24. B 25. B Chapter 47- Nursing Care of the Child With an Alteration in Immunity or Immunologic Disorder 1. While providing care to a 5-month-old girl whose family has a history of food allergies, the nurse instructs the parents about foods to be avoided in the first year of life. Which response by the parents indicates a need for further teaching? A) “She cannot have any cow's milk.” B) “I should continue breastfeeding until at least 6 months.” C) “Peanuts in any form should be avoided.” D) “Any kind of fruit is acceptable.” 2. A nurse is caring for a 12-year-old girl with a severe peanut allergy. The girl's parents are upset because the school does not permit her to carry her EpiPen with her. It must remain in the school's office per school regulations. Which response by the nurse would be most appropriate? A) “She is allowed by law to carry her EpiPen with her; I will talk to school authorities.” B) “Let's file an action plan and keep it in the school office in the event of anaphylaxis.” C) “Make sure she wears a medical alert bracelet so that school staff know she has allergies.” D) “I will be happy to train school authorities and staff to recognize anaphylaxis.” 3. The nurse is caring for a newborn whose mother is HIV positive. The nurse would expect to administer a 6-week course of which medication? A) Lopinavir B) Ritonavir C) Nevirapine D) Zidovudine 4. The nurse is caring for a child who is having an anaphylactic reaction with bronchospasm. The nurse would expect to administer which of the following for bronchospasm as ordered? A) Epinephrine B) Corticosteroid C) Albuterol D) Diphenhydramine 5. The nurse is caring for a child undergoing highly active antiretroviral therapy (HAART) for HIV infection. The nurse is preparing to administer the prescribed medication. In addition to the nucleoside analog reverse transcriptase inhibitors (NRTIs) and the nonnucleoside analog reverse transcriptase inhibitors (NNRTIs), the nurse is cognizant that the child will be taking which additional medication as part of the three-drug regimen? A) Protease inhibitors B) Corticosteroids C) Cytotoxic drugs D) Disease-modifying antirheumatic drugs (DMARDs) 6. The nurse is providing home care instructions for a 13-year-old girl recently diagnosed with systemic lupus erythematosus. Which response by the girl indicates a need for further teaching? A) “I need to wear sunscreen in the summer to prevent rashes.” B) “I need to eat a healthy diet, exercise, and get plenty of sleep.” C) “I need an eye examination every year.” D) “I need to be careful when it is cold; I should always wear gloves.” 7. The nurse is caring for a child who is taking corticosteroids for systemic lupus erythematosus. The nurse closely monitors the child based on the understanding that corticosteroids exert which major action? A) They increase liver enzymes. B) They can mask signs of infection. C) They cause bone marrow suppression. D) They decrease renal function. 8. The school nurse is walking through the lunchroom when one of the children says she feels strange after switching her lunch with her friend. Which assessment would be most important? A) Asking if she has a rash anywhere B) Checking if she has any nausea C) Determining if her throat itches D) Asking if she has abdominal pain 9. The nurse is caring for a child who has undergone stem cell transplantation for severe combined immune deficiency. Which of the following would the nurse interpret as indicative of graft-versus-host disease? A) Presence of wheezing B) Splenomegaly C) Maculopapular rash D) Chronic or recurrent diarrhea 10. The nurse is administering intravenous immune globulin (IVIG). The nurse assesses vital signs and for adverse reactions every 15 minutes for the first hour of administration. After the first hour, the nurse most likely would continue to assess the child at which frequency? A) Every 30 minutes B) Every 45 minutes C) Every 60 minutes D) Every 2 hours 11. When reviewing the history of a child with suspected primary immunodeficiency, which of the following would the nurse be least likely to find? A) Weight appropriate for height B) Antibiotic therapy for the past 3 months without effect C) 10 episodes of otitis media in the last year D) Three bouts of sinusitis within a year's time 12. Which of the following would the nurse expect to find in a male infant with Wiskott- Aldrich syndrome? A) Eczema B) Thrombocytosis C) Lymphadenopathy D) Pneumonia 13. After teaching a class about humoral and cellular immunity, the nurse recognizes that the additional teaching is needed when the class states which of the following? A) Humoral immunity crosses the placenta. B) Cellular immunity involves the T lymphocytes. C) Cellular immunity recognizes antigens. D) Humoral immunity does not destroy the foreign cell. 14. When teaching a group of new parents about newborn care and development, which immunoglobulin would the nurse explain as being primarily responsible for the passive immunity exhibited by newborns? A) IgA B) IgG C) IgM D) IgE 15. Which exercise would the nurse suggest as most helpful to maintain mobility in a child with juvenile idiopathic arthritis? A) Jogging every other day B) Using a treadmill C) Swimming D) Playing basketball 16. A child is diagnosed with juvenile idiopathic arthritis and is receiving several different medications listed in the medication administration record. Which agent would the nurse identify as being used to prevent disease progression? A) Aspirin B) Prednisone C) Ibuprofen D) Methotrexate 17. A group of nursing students are reviewing information about humoral and cellular immunity. The students demonstrate understanding of this material when they identify which of the following as involved in cellular immunity? A) B cells B) Antibodies C) Antigens D) T cells 18. A nurse is preparing a plan of care for a child with a primary immunodeficiency. Which nursing diagnosis would the nurse most likely identify as the priority? A) Imbalanced nutrition, less than body requirements related to poor appetite B) Ineffective protection related to impaired humoral defenses C) Acute pain related to inflammatory processes D) Risk for delayed growth and development related to chronic illness 19. A child with hypogammaglobulinemia is to receive intravenous immunoglobulin (IVIG). Which of the following would be least appropriate for the nurse to do? A) Shake the vial after reconstituting it B) Premedicate the child with acetaminophen C) Obtain preinfusion vital signs D) Check serum blood urea nitrogen and creatinine levels 20. A nursing student is reviewing information about primary immunodeficiencies. The student demonstrates understanding of the material by identifying which of the following as affecting only males? Select all answers that apply. A) X-linked agammaglobulinemia B) Wiskott-Aldrich syndrome C) Selective IgA deficiency D) X-linked hyper-IgM syndrome E) IgG subclass deficiency F) Severe combined immune deficiency 21. The nurse is reviewing the laboratory test results of a child who is suspected of having systemic lupus erythematosus (SLE). Which of the following would the nurse identify as supporting this diagnosis? Select all answers that apply. A) Positive antinuclear antibody (ANA) B) Increased C3 levels C) Thrombocytopenia D) Leukopenia E) Increased hematocrit 22. A child is diagnosed with a food allergy to milk. When teaching the parents about this allergy, which of the following would the nurse suggest as possible substitutions? Select all answers that apply. A) Fruit juice B) Rice milk C) Yogurt D) Nondairy creamers E) Soy milk 23. The nurse is talking to the parents of a child who has been diagnosed with severe combined immune deficiency. Which statement by the parents best indicates that they understand their child's condition? A) “He'll need to receive intravenous immunoglobulin routinely.” B) “We'll need to prepare him and ourselves for a bone marrow transplant.' C) “He'll need to receive several different types of antiviral medications.” D) “We'll make sure that he has his EpiPen with him at all times.” 24. The nurse is assessing a child with pauciarticular-type juvenile idiopathic arthritis. Which of the following would the nurse expect to assess? A) Fever B) Rash C) Eye inflammation D) Splenomegaly 25. A child with systemic lupus erythematosus is receiving high-dose corticosteroid therapy over the long term. The nurse would instruct the parents and child to report which of the following? A) Difficulty urinating B) Visual changes C) Joint pain D) Rash 26. A nurse is assessing a child who may have a latex allergy. The nurse asks the child about allergic reactions with certain foods. Which foods if identified by the child as experiencing an allergic reaction would help support the suspected latex allergy? Select all answers that apply. A) Peaches B) Plums C) Carrots D) Tomatoes E) Apples F) Lettuce   Answer Key 1. D 2. A 3. D 4. C 5. A 6. A 7. B 8. C 9. C 10. C 11. A 12. A 13. C 14. B 15. C 16. D 17. D 18. B 19. A 20. A, B, D 21. A, C, D 22. A, B, E 23. B 24. C 25. C 26. A, B, C, D Chapter 48- Nursing Care of the Child With an Alteration in Metabolism / Endocrine Disorder 1. The nurse is obtaining a health history from parents whose 4-month-old boy has congenital hypothyroidism. Which of the following would the nurse most likely assess? A) The child has above-normal growth for his age. B) The child is active and playful. C) The skin is pink and healthy looking. D) It is difficult to keep the child awake. 2. The nurse is caring for an 8-year-old girl with hyperpituitarism. Which of the following ordered treatments will the nurse expect to perform? A) Give desmopressin acetate intranasally B) Inject octreotide acetate C) Give 1 mg/kg/day of methimazole D) Administer glipizide orally 3. The nurse is developing a plan of care for a 7-year-old boy with diabetes insipidus. Which of the following would the nurse most likely identify as the priority nursing diagnosis? A) Deficient fluid volume related to dehydration B) Excess fluid volume related to edema C) Deficient knowledge related to fluid intake regimen D) Imbalanced nutrition, more than body requirements related to excess weight 4. The nurse is assessing a 9-year-old girl with a history of tuberculosis at age 6 years. She has been losing weight and has no appetite. The nurse suspects Addison disease based on which of the following assessment findings? A) Arrested height and increased weight B) Thin, fragile skin and multiple bruises C) Hyperpigmentation and hypotension D) Blurred vision and enuresis 5. The nurse is caring for a 10-year-old boy with hyperpituitarism due to a tumor on the anterior pituitary gland. Which of the following would be a priority for this child? A) Promoting a healthy body image B) Encouraging effective family coping C) Providing pre- and postoperative care D) Promoting knowledge about treatment options 6. The nurse is caring for a 13-year-old girl with delayed puberty. Based on the nurse's knowledge of this condition, the nurse would include which nursing diagnosis in the child's plan of care? A) Disabled family coping related to the child's disorder B) Imbalanced nutrition, less than body requirements related to the child's short stature C) Noncompliance related to the need for lifelong hormone therapy D) Deficient knowledge related to the administration of estradiol 7. The nurse is preparing a teaching plan for the family and their 6-year-old son who has just been diagnosed with diabetes mellitus. Which of the following would the nurse identify as the initial goal for the teaching plan? A) Developing management and decision-making skills B) Educating the parents about diabetes mellitus type 1 C) Developing a nutritionally sound, 30-day meal plan D) Promoting independence with self-administration of insulin 8. The nurse is caring for an 8-year-old girl with an endocrine disorder involving the posterior pituitary gland. Which of the following would the nurse expect to implement? A) Instructing the parents to report adverse reactions to the growth hormone treatment B) Teaching the parents how to administer the desmopressin acetate C) Informing the parents that treatment stops when puberty begins D) Educating the parents to report signs of acute adrenal crisis 9. The nurse is assessing a 13-year-old boy with type 2 diabetes mellitus. Which of the following would the nurse correlate with disorder? A) The parents report that their child had “a cold or flu” recently. B) Blood pressure is decreased when checking vital signs. C) The parents report that their son “can't drink enough water.” D) Auscultation reveals Kussmaul breathing. 10. The nurse is preparing a teaching plan for a 10-year-old girl with hyperthyroidism. Which of the following would the nurse include in the plan? A) Describing surgery to remove an anterior pituitary tumor B) Teaching her parents to give injections of growth hormone C) Explaining about the radioactive iodine procedure D) Showing her parents how to give DDAVP intranasally 11. Which of the following would the nurse expect to assess in a child with hypothyroidism? A) Nervousness B) Heat intolerance C) Smooth velvety skin D) Weight gain 12. The parents of a child with congenital adrenal hyperplasia bring the child to the emergency department for evaluation because the child has had persistent vomiting. Which of the following would lead the nurse to suspect that the child is experiencing an acute adrenal crisis? A) Hypernatremia B) Bradycardia C) Hypertension D) Hyperkalemia 13. A child with diabetes reports that he is feeling a little shaky. Further assessment reveals that the child is coherent but with some slight tremors and sweating. A fingerstick blood glucose level is 70 mg/dL. Which of the following would the nurse do next? A) Administer a sliding-scale dose of insulin B) Give 10 to 15 grams of a simple carbohydrate C) Offer a complex carbohydrate snack D) Administer glucagon intramuscularly 14. A child with growth hormone deficiency is receiving growth hormone. Which of the following would the nurse interpret as indicating effectiveness of this therapy? A) Rapid weight gain B) Complaints of headaches C) Height increase of 4 inches D) Growth plate closure 15. After teaching the parents of a daughter with central precocious puberty about medication therapy, which statement by the parents indicates successful teaching? A) “She needs to use the nasal spray once every day.” B) “She'll start puberty again when the medication stops.” C) “This medication will slow down the changes but not reverse them.” D) “Once therapy is done, she'll need surgery.” 16. A group of nursing students are reviewing information about the endocrine system in infants and children. The students demonstrate understanding of the information when they state which of the following? A) Endocrine glands begin developing in the third trimester of gestation. B) At birth, the endocrine glands are completely functional. C) Infants have difficulty balancing glucose and electrolytes. D) A child's endocrine system has little effect on growth and development. 17. A child is diagnosed with hyperthyroidism. Which agent would the nurse expect the physician to prescribe? A) Mineralocorticoid B) Methimazole C) Levothyroxine D) Dexamethasone 18. A child with diabetes insipidus is being treated with vasopressin. The nurse would assess the child closely for signs and symptoms of which of the following? A) Syndrome of inappropriate antidiuretic hormone (SIADH) B) Thyroid storm C) Cushing syndrome D) Vitamin D toxicity 19. The nurse is reviewing the laboratory test results of a child with Addison disease. Which of the following would the nurse expect to find? A) Hypernatremia B) Hyperkalemia C) Hyperglycemia D) Hypercalcemia 20. A nurse is teaching the parents of an infant with congenital adrenal hyperplasia about the signs and symptoms of adrenal crisis. The nurse determines that the teaching was successful when the parents identify which of the following? A) Bradycardia B) Constipation C) Fluid overload D) Persistent vomiting 21. A group of students are reviewing information about the various types of insulin used to treat type 1 diabetes. The students demonstrate understanding of the information when they identify which insulin listed below as having the longest duration? A) Lispro B) Regular C) NPH D) Glargine 22. A 5-year-old child with type 1 diabetes is brought to the clinic by his mother for a follow-up visit after having his hemoglobin A1C level drawn. Which result would indicate to the nurse that the child is achieving long-term glucose control? A) 9.0% B) 8.2% C) 7.3% D) 6.9% 23. The parents of a 7-year-old girl with type 1 diabetes has been recording her blood glucose measurements before meals and at bedtime for the past 4 days; they are as follows: Monday Tuesday Wednesday Thursday B: 120 mg/dL 135 mg/dL 124 mg/dL 200 mg/dL L: 110 mg/dL 120 mg/dL 140 mg/dL 220 mg/dL D: 90 mg/dL 140 mg/dL 130 mg/dL 200 mg/dL Bed: 110 mg/dL 110 mg/dL 160 mg/dL 240 mg/dL The parents bring the child in for a follow-up visit and show the nurse the results. Based on the results, the nurse would need to obtain additional information from the parents and child about which day? A) Monday B) Tuesday C) Wednesday D) Thursday 24. The nurse is administering 10 units of NPH insulin to a child at 8 a.m. The nurse would expect this insulin to begin acting at which time? A) By 8:15 a.m. B) Between 8:30 and 9 a.m. C) Between 9 and 11 a.m. D) Around 12 noon 25. The nurse suspects that a 4-year-old with type 1 diabetes is experiencing hypoglycemia based on which of the following? Select all that apply. A) Blurred vision B) Dry, flushed skin C) Diaphoresis D) Slurred speech E) Fruity breath odor F) Tachycardia 26. A nurse is preparing a presentation for a group of parents with children diagnosed with diabetes type 1. The children are all adolescents. Which of the following issues would the nurse need to address? Select all that apply. A) Self-monitoring of blood glucose levels B) Feelings of being different C) Deficient decision-making skills D) Body image conflicts E) Struggle for independence 27. A group of nursing students are reviewing the components of the endocrine system. The students demonstrate understanding of the review when they identify which of the following as the primary function of this system? A) Regulation of water balance B) Hormonal secretion C) Cellular metabolism D) Growth stimulation Answer Key 1. D 2. B 3. A 4. C 5. C 6. D 7. A 8. B 9. C 10. C 11. D 12. D 13. B 14. C 15. B 16. C 17. B 18. A 19. B 20. D 21. D 22. B 23. D 24. C 25. C, D, F 26. C, D, E 27. B Chapter 49- Nursing Care of the Child With an Alteration in Genetics 1. The nurse is teaching a couple about the pros and cons of genetic testing. Which of the following statements best describes the capabilities of genetic testing? A) “Various genetic tests help the physician choose appropriate treatments.” B) “Genetic testing helps couples avoid having children with fatal diseases.” C) “Genetic tests identify people at high risk for preventable conditions.” D) “Some genetic tests can give a probability for developing a disorder.” 2. The nurse is caring for a couple who is having a triple screen done. The nurse would least likely expect which of the following to be tested? A) a-Fetoprotein B) Human chorionic gonadotropin C) Unconjugated estriol D) Testosterone 3. The nurse is caring for 3-day-old girl with Down syndrome whose mother had no prenatal care. Which of the following will be the priority nursing diagnosis? A) Imbalanced nutrition, less than body requirements related to the effects of hypotonia B) Deficient knowledge related to the presence of a genetic disorder C) Delayed growth and development related to a cognitive impairment D) Impaired physical mobility related to poor muscle tone 4. The nurse is assessing a 4-year-old boy whose mother was 40 years old when he was born. Which of the following findings suggests this child has a genetic disorder? A) Inquiry determines the child had feeding problems. B) Observation shows nasal congestion and excess mucus. C) Inspection reveals low-set ears with lobe creases. D) Auscultation reveals the presence of wheezing. 5. The nurse is teaching the parents of a 1-month-old girl with Down syndrome how to maintain good health for the child. Which instruction would the nurse be least likely to include? A) Getting cervical radiographs between 3 and 5 years of age B) Adhering to the special dietary needs of the child C) Getting an echocardiogram before 3 months of age D) Monitoring for symptoms of respiratory infection 6. The nurse is counseling a couple who suspect that they could bear a child with a genetic abnormality. Which of the following would be most important for the nurse to incorporate into the plan of care when working with this family? A) Gathering information from at least three generations B) Informing the family of the need for a wide range of information C) Maintaining the confidentiality of the information D) Presenting the information in a nondirective manner 7. The nurse is obtaining the health history for a 15-month-old boy from the parents. The child is not yet speaking. Which finding would be eliminated as a risk factor for a possible genetic disorder? A) The child is male and Caucasian. B) The grandmother and father have hearing impairments. C) The child was a breech delivery 3 weeks early. D) The mother was 37 when she became pregnant. 8. The nurse is caring for a 1-month-old girl with low-set ears and severe hypotonia who was diagnosed with trisomy 18. Which nursing diagnosis would the nurse identify as most likely? A) Interrupted family process related to the child's diagnosis B) Deficient knowledge deficit related to the genetic disorder C) Grieving related to the child's poor prognosis D) Ineffective coping related to stress of providing care 9. The nurse is caring for a 9-year-old boy with achondroplasia. Which of the following would the nurse expect to assess? A) Narrow passages from the nose to the throat B) Slim stature, hypotonia, and a narrow face C) Craniosynostosis and a small nasopharynx D) Trident hand and persistent otitis media 10. The nurse is teaching a couple about X-linked disorders. They are concerned that they might pass on hemophilia to their children. Which of the following responses indicates the need for further teaching? A) “The father can't be a carrier if he doesn't have hemophilia.” B) “If the father doesn't have it, then his kids won't either.” C) “If the mother is a carrier, her daughter could be one too.” D) “If the mother is a carrier, her sons may have hemophilia.” 11. When teaching a class about trisomy 21, the instructor would identify this disorder as due to which of the following? A) Nondisjunction B) X-linked recessive inheritance C) Genomic imprinting D) Autosomal dominant inheritance 12. The nurse is preparing a presentation to a local community group about genetic disorders and the types of congenital anomalies that can occur. Which of the following would the nurse include as a major congenital anomaly? A) Overlapping digits B) Polydactyly C) Umbilical hernia D) Cleft palate 13. The nurse is caring for a couple who have just learned that their infant has a genetic disorder. Which of the following would be least appropriate for the nurse to do at this time? A) Actively listening to the parents' concerns B) Teaching the parents about the child's medical needs C) Providing time for the parents to ask questions D) Offering suggestions for support services 14. When providing guidance to the parents of a child with Down syndrome, which of the following would be most appropriate? A) Encourage the parents to home-school the child. B) Advise the parents that the child will need monthly thyroid testing. C) Instruct them on the need for yearly dental visits. D) Teach the parents about the need for a high-fiber diet. 15. Which of the following would lead the nurse to suspect that a child has Turner syndrome? A) Webbed neck B) Microcephaly C) Gynecomastia D) Cognitive delay 16. When performing a physical examination on a small child, the nurse observes approximately 8 to 10 light-brown spots concentrated primarily on the trunk and extremities, two small lumps on the posterior trunk, and axillary freckling. The nurse interprets these findings to suggest which of the following? A) Klinefelter syndrome B) Neurofibromatosis C) Fragile X syndrome D) Sturge-Weber syndrome 17. After teaching a class about inborn errors of metabolism, the instructor determines that additional teaching is needed when the class identifies which of the following as an example of an inborn error of metabolism? A) Galactosemia B) Maple syrup urine disease C) Achondroplasia D) Tay-Sachs disease 18. When teaching the parents of a child with phenylketonuria, the nurse would instruct them to include which of the following foods in the child's diet? A) Milk B) Oranges C) Meat D) Eggs 19. When providing support and education to the family of a child who is diagnosed with a serious genetic abnormality, which of the following would be the highest priority? A) Assisting with scheduling follow-up visits B) Establishing a trusting relationship C) Teaching the family what to expect D) Using measures to promote growth and development 20. A group of students are reviewing information about major and minor congenital disorders. The students demonstrate understanding of the information when they identify which of the following as a minor disorder? A) Webbed neck B) Omphalocele C) Cutaneous hemangioma D) Facial asymmetry 21. The nurse is assessing an infant and notes that the infant's urine has a musty odor. Which of the following would the nurse suspect? A) Maple syrup urine disease B) Tyrosinemia C) Phenylketonuria D) Trimethylaminuria 22. A nursing instructor is preparing a class discussion on the benefits and drawbacks associated with genetic advances and the Human Genome Project. Which of the following would the instructor address as a potential problem? A) Early detection possibilities B) Risk profiling C) Focus on causes D) Rapid diagnosis 23. A nurse is reviewing an article about genetic disorders and patterns of inheritance. The nurse demonstrates understanding of the information by identifying which of the following as an example of an autosomal dominant genetic disorder? A) Neurofibromatosis B) Cystic fibrosis C) Tay-Sachs disease D) Sickle cell disease 24. A nursing student is preparing an oral presentation about autosomal recessive inheritance. Which of the following must occur for an offspring to demonstrate signs and symptoms of the disorder with this type of inheritance? A) Both parents must be heterozygous carriers. B) One parent must have the disease. C) The mother must be a carrier. D) The father must be affected by the disease. 25. A pregnant woman is to undergo testing to evaluate for chromosomal abnormalities. Which test would the nurse expect to be done the earliest? A) Amniocentesis B) Chorionic villi sampling C) Triple screen D) Fetal nuchal translucency 26. A child is diagnosed with cri-du-chat syndrome. Which of the following would the nurse expect to assess? Select all answers that apply. A) Hypertonia B) Short stature C) Simian crease D) Wide and flat nasal bridge E) Hydrocephaly 27. The nurse is assessing a 2-day-old newborn and suspects Down syndrome based on which of the following? Select all answers that apply. A) Flat facial profile B) Downward slant to the eyes C) Large tongue compared to mouth D) Simian crease E) Epicanthal folds F) Rigid joints 28. When describing Prader-Willi syndrome to a group of nursing students, the instructor would describe this condition as one affecting which chromosome? A) 4 B) 5 C) 11 D) 15 29. A group of nursing students are reviewing information about neurocutaneous syndromes. The students demonstrate an understanding of these disorders when they identify which of the following as an example? A) Sturge-Weber syndrome B) Marfan syndrome C) Apert syndrome D) Achondroplasia   Answer Key 1. D 2. D 3. A 4. C 5. B 6. D 7. A 8. C 9. D 10. B 11. A 12. D 13. B 14. D 15. A 16. B 17. C 18. B 19. B 20. A 21. C 22. B 23. A 24. A 25. B 26. B, C, D 27. A 28. D 29. A Chapter 50- Nursing Care of the Child With an Alteration in Behavior, Cognition, or Development 1. The nurse is teaching the mother of a 12-year-old boy about the risk factors associated with drug and alcohol abuse. Which response by the mother indicates a need for further teaching? A) “A family history of alcoholism is a risk factor for substance abuse.” B) “Just because his friends are experimenting does not mean that he will.” C) “If my husband or I have a substance abuse problem it could increase his risk.” D) “Negative life events are a potential risk factor.” 2. The nurse is caring for an adolescent girl with anorexia nervosa. Which of the following findings would indicate to the nurse that the girl requires hospitalization? A) Weight gain of one-half pound per week B) Food refusal C) Body mass index of 18 D) Soft, sparse body hair and dry, sallow skin 3. The nurse is caring for an adolescent girl with a suspected anxiety disorder. The girl states that she is constantly double-checking that she has unplugged her curling iron and must make sure that everything is in perfect order in her room before she leaves the house. The nurse interprets these findings as indicating which of the following? A) Generalized anxiety disorder B) Posttraumatic stress disorder C) Social phobia D) Obsessive-compulsive disorder 4. The nurse is caring for a 7-year-old with Tourette syndrome. The nurse would be alert for which of the following comorbid conditions? A) Depression B) Anxiety disorder C) Attention deficit/hyperactivity disorder D) Asperger syndrome 5. A nurse is caring for a 10-year-old boy with a nursing diagnosis of ineffective coping related to an inability to deal with stressors secondary to anxiety. Which of the following would be most important for the nurse to do first? A) Set clear limits on the child's behavior B) Teach the child problem-solving skills C) Encourage a discussion of the child's thoughts and feelings D) Role model appropriate social and conversation skills 6. The nurse is caring for a 3-year-old boy. The parents are concerned that he is exhibiting signs of cognitive delays. Which statement by the parents would lead the nurse to suspect autism spectrum disorder rather than possible learning disability? A) “He is not speaking in complete sentences.” B) “We can understand a lot of what he says, but no one else can.” C) “He seems to be speaking words less and less frequently.” D) “He is unable to sit still for a short story.” 7. A nurse is caring for a 5-year-old girl with depression. The girl is having difficulty coping with her feelings of sadness and fear, which stem from her parents' separation and recent divorce. The girl has been prescribed antidepressant medication but the mother thinks the girl would benefit from therapy. The nurse anticipates a referral to a therapist specializing in which type of therapy? A) Individual therapy B) Play therapy C) Behavioral therapy D) Hypnosis 8. The nurse is caring for a 13-year-old boy with a history of inappropriate behavior. Which statement by the mother would lead the nurse to suspect oppositional defiant disorder rather than conduct disorder? A) “He has frequent temper tantrums.” B) “He was pulling the neighbor's dog around by his leash.” C) “He is constantly lying to me.” D) “He has stolen hundreds of dollars from my purse.” 9. The nurse is caring for a 5-year-old. The child's mother reports that he is extremely sensitive to sounds that most people do not notice and that he prefers complete silence. She explains that the boy is resisting going to school due to the noise and commotion. Additionally, the mother states that he will only wear 100% cotton clothing with all of the tags cut out. The nurse interprets these findings as indicating which of the following? A) Anxiety disorder B) Sensory integration dysfunction C) Depression D) Obsessive-compulsive disorder 10. The nurse is caring for a child with bipolar disorder. The child is taking lithium as ordered. The parents inquire about the potential side effects. Which response by the nurse would be most appropriate? A) “You might see excessive urination and thirst, tremor, nausea, weight gain, and diarrhea.” B) “He might experience a significant decrease in his appetite and difficulty sleeping.” C) “You need to watch for dry mouth, urinary retention, and constipation.” D) “This medication can cause seizures, agitation, headache, and nausea.” 11. A child with attention deficit/hyperactivity disorder is prescribed long-acting methylphenidate. Which of the following would the nurse include when teaching the child and his parents about this drug? A) “Give the drug three times a day: morning, midday, and after school.” B) “This drug may cause drowsiness, so be careful when doing things.” C) “Some increase in appetite may occur, so watch how much you eat.” D) “Take this drug every day in the morning when you wake up.” 12. When reviewing the medical record of a child, which of the following would the nurse interpret as the most sensitive indicator of intellectual disability? A) History of seizures B) Preterm birth C) Vision deficit D) Language delay 13. A school-age child diagnosed with depression is receiving antidepressant therapy. The nurse would instruct the parents to notify the physician immediately if the child demonstrates which of the following? A) Loss of interest B) Gastric upset C) Sedation D) Urinary retention 14. A nurse is providing an in-service program on child abuse for a group of newly hired nurses. When evaluating the effectiveness of the teaching, the nurse determines a need for additional review when the group identifies which of the following as an indicator of possible child abuse? A) Consistent delays in seeking treatment for the child's injuries B) Frequent changes in history information with visits C) Injuries that are inconsistent with the reported traumatic event D) Sexual behavior that correlates with the child's developmental age 15. Which of the following would lead the nurse to suspect that an adolescent has bulimia? A) Body mass index less than 17 B) Calluses on back of knuckles C) Nail pitting D) Bradycardia 16. A child with depression is prescribed fluoxetine. The nurse identifies this as belonging to which class of drugs? A) Atypical antidepressant B) Tricyclic antidepressant C) Selective serotonin reuptake inhibitor D) Psychostimulant 17. A child is receiving therapy in which he is learning to replace automatic negative thought patterns with alternative ones. The nurse interprets this as which type of therapy? A) Cognitive therapy B) Behavioral therapy C) Milieu therapy D) Individual therapy 18. A nurse is preparing a program for a parent group about various techniques that can be used to manage behavior. Which of the following would the nurse be least likely to include? A) Focus the child's attention on the negative behavior. B) Set limits with the child for responsible behavior. C) Ignore inappropriate behaviors. D) Provide positive feedback for self-control efforts. 19. The nurse is reviewing the medical record of a child who has dyspraxia. The nurse understands that this child experiences difficulty with which of the following? A) Reading and writing B) Mathematics and computation C) Manual dexterity and coordination D) Composition and spelling 20. A nurse is conducting a screening program for autism in infants and children. Which of the following would the nurse identify as a warning sign? A) Lack of babbling by 6 months B) Inability to say a single word by 16 months C) Lack of gestures by 8 months D) Inability to use two words by 18 months 21. A nurse is preparing a teaching session for a group of parents with children newly diagnosed with attention deficit/hyperactivity disorder (ADHD). When explaining this disorder to the parents, which of the following would the nurse include as being involved? Select all answers that apply. A) Impulsivity B) Inattention C) Distractibility D) Hyperactivity E) Defiance F) Anxiety 22. A school nurse is working with the parents of an 8-year-old who has Tourette syndrome on how best to accommodate the child. Which of the following would be most helpful? Select all answers that apply. A) Allowing for breaks when tics occur B) Providing for “time-outs” during the day C) Using a tape recorder to take notes D) Ensuring a specified amount of time for test taking E) Implementing a reward system for behavior 23. When assessing the adolescent with anorexia, which of the following would the nurse expect to find? A) Tachycardia B) Hypertension C) Fever D) Murmur 24. After teaching the parents of a child with attention deficit/hyperactivity disorder about ways to control the child's behavior, the nurse determines a need for additional teaching when the parents state which of the following? A) “If he starts to act out, we'll have him do a time-out to help him refocus.” B) “We can use a reward system when he behaves appropriately.” C) “If he misbehaves, we need to punish him instead of reward him.” D) “We need to help him set realistic goals that he can achieve.” 25. A nurse is reviewing the medical record of an 11-year-old child with a conduct disorder. Which of the following would the nurse identify as characteristic of this disorder? Select all answers that apply. A) Easily annoyed B) Initiator of physical fights C) Temper tantrums D) Truancy E) Arrest for arson 26. The nurse identifies a nursing diagnosis of impaired social interaction related to altered social skills as evidenced by impulsivity and intrusive behavior. The nurse plans to identify factors that aggravate the child's behavior for which reason? A) Minimize stimuli that exacerbate the child's undesired behaviors B) Improve the child's ability to deal with external stressors C) Promote increased ability to follow through D) Encourage the child to adopt expectations into his routine 27. A child is prescribed trazodone. Which of the following would the nurse be least likely to include in the plan of care related to this drug? A) Monitoring blood pressure for orthostatic hypotension B) Assessing the child for sedation and drowsiness C) Administering the drug with a snack D) Monitoring for tardive dyskinesia   Answer Key 1. B 2. B 3. D 4. C 5. C 6. C 7. B 8. A 9. B 10. A 11. D 12. D 13. A 14. D 15. B 16. C 17. A 18. A 19. C 20. B 21. A, B, C, D 22. A, C 23. D 24. C 25. B, D, E 26. A 27. D Chapter 51- Nursing Care During a Pediatric Emergency 1. The nurse is caring for a 6-year-old girl who was injured in a bicycle accident. Which question would be most important for the nurse to ask during the health history? A) “Has she been diagnosed with any chronic disorders?” B) “Is your daughter currently taking any medications?” C) “Is she allergic to any medications or drugs?” D) “Tell me how the bicycle accident happened.” 2. The nurse is caring for a 7-year-old boy experiencing respiratory distress who is scheduled to have a chest radiograph. Which of the following would be most important for the nurse to include in the child's plan of care? A) Administering a sedative to help calm the child B) Assisting the child to lie still during the chest radiograph C) Accompanying the child to continue observation D) Informing the child that he might hear a loud banging noise 3. A 5-year-old girl is cyanotic, dusky, and anxious when she arrives in the emergency department. Which of the following would be most appropriate? A) Ventilating the child with a bag-valve-mask B) Estimating the child's weight using a Broselow tape C) Providing therapy using automated external defibrillation D) Using rescue breathing and chest compressions 4. When caring for an 8-year-old boy injured in an automobile accident, the nurse demonstrates understanding of the principles of Pediatric Advanced Life Support (PALS) by which action? A) Assisting ventilation with a bag-valve-mask (BVM) device B) Treating ventricular fibrillation using a defibrillator C) Managing compensated shock to prevent decompensated shock D) Treating supraventricular tachycardia using cardioversion 5. A 9-year-old girl who has fallen from a second-story window is brought to the emergency department. Which assessment would be a priority? A) Evaluating pupils for equality and reactivity B) Monitoring oxygen saturation levels C) Asking the child if she knows where she is D) Using the appropriate pain assessment scale 6. The parents bring their 3-year-old son to the emergency department after having found that he has ingested some of his mother's medicine. Which assessment would be of critical importance for this child? A) Assessing mental status and skin moisture and color B) Evaluating the effectiveness of the child's breathing C) Noting the child's pulse rate and quality D) Auscultating all lung fields for signs of edema 7. Which of the following would the nurse do first for a 5-year-old girl with profound bradycardia? A) Provide oxygen at 100% B) Administer epinephrine as ordered C) Use warming blankets D) Perform gastric lavage 8. Which measure would be most appropriate for the nurse to do to ensure that a child's endotracheal (ET) tube is correctly positioned? A) Auscultate for abdominal breath sounds B) Mark the tracheal tube at the child's lip C) Watch for a yellow display on a CO2 monitor D) Inspect for water vapor in the tracheal tube 9. Which intervention would be most helpful in preventing barotrauma when ventilating a 3year-old girl with a bag-valve-mask? A) Choosing the correct size bag and face mask B) Setting the flow rate at exactly 10 L/minute C) Maintaining the airway in the open position D) Delivering one breath every 3 to 5 seconds 10. The nurse is providing care to a 4-year-old boy with a broken arm and an infected laceration from a fall. The nurse notes a significant elevation in the child's heart rate. Which intervention would be least appropriate? A) Administering antipyretics as ordered for fever B) Using a defibrillator to reduce the heart rate C) Administering analgesics to reduce pain D) Allowing the parents to comfort the child 11. A child weighing 51 lbs requires defibrillation. How many joules would the nurse expect to give initially? A) 46 B) 92 C) 102 D) 204 12. A 1-month-old infant admitted to the emergency department in respiratory distress exhibits a regular pattern of breathing followed by brief periods of apnea, then tachypnea for a short time, eventually returning to a normal respiratory rate. The nurse documents this finding as which of the following? A) Hypoventilation B) Hyperventilation C) Periodic breathing D) Stridor 13. The nurse is gathering the necessary equipment for tracheal intubation for a child who is 2 years old. Which tracheal tube size would the nurse obtain? A) 4.5 B) 5 C) 5.5 D) 6 14. Which of the following would lead the nurse to suspect that a 5-year-old child is experiencing supraventricular tachycardia? A) Heart rate 160 beats per minute B) Flattened P waves C) Normal QRS complex D) History of fever 15. Which of the following would be most appropriate to use to help maintain a patent airway in an infant experiencing a respiratory emergency? A) Neck hyperextension B) Head tilt–chin lift technique C) Jaw-thrust maneuver D) Small towel under shoulders 16. After teaching a group of nursing students about shock in children, the instructor determines that the teaching was successful when the students identify which type of shock as most common? A) Septic B) Cardiogenic C) Hypovolemic D) Distributive 17. A child who weighs 53 lbs is receiving fluid volume replacement as part of the treatment for shock. The nurse is evaluating the child's hourly urinary output to determine if the child's condition is improving. Which output would the nurse interpret as most indicative of improvement? A) 12 mL B) 15 mL C) 22 mL D) 30 mL 18. A child has a tracheal tube in place and will be receiving medications via this tube. Which of the following medications would the nurse expect to be administered in this manner? Select all answers that apply. A) Lidocaine B) Adenosine C) Atropine D) Dopamine E) Epinephrine F) Naloxone 19. A group of students are reviewing information about respiratory arrest in children. The students demonstrate understanding of this information when they identify which of the following as a common cause involving the upper airway? Select all answers that apply. A) Croup B) Asthma C) Pertussis D) Epiglottitis E) Pneumothorax 20. The nurse is preparing the plan of care for a child experiencing respiratory distress. Which of the following would be the priority? A) Providing supplemental oxygen B) Monitoring for changes in status C) Assisting ventilation D) Maintaining a patent airway 21. The nurse is providing care to a child who is intubated and the child's condition is deteriorating. Which of the following would the nurse do first? A) Check if the tracheal tube is obstructed B) Assess for displacement of the tracheal tube C) Look for signs of a possible pneumothorax D) Check the equipment for malfunction 22. The nurse is providing care to a child experiencing shock. Which of the following intravenous solutions would the nurse expect to administer? A) Ringer lactate B) Dextrose 5% and water C) Dextrose 5% and normal saline D) Dextrose 10% and water 23. A child is brought to the emergency department with a suspected poisoning. Which of the following would the nurse least likely expect to be used? A) Gastric lavage B) Syrup of ipecac C) Activated charcoal D) Whole bowel irrigation 24. A group of students are working on a presentation for a local health fair about safety for children. When developing this presentation, the students would address which of the following as the most common cause of pediatric injury? A) Sports B) Firearm use C) Falls D) Automobile accidents 25. As part of their orientation to their pediatric clinical rotation, an instructor is teaching a group of students how to perform cardiopulmonary resuscitation (CPR) on a child. Two students return demonstrate the skill using an infant manikin. Which of the following indicates the proper technique? A) Compressing 30 times for every 2 breaths B) Placing the heel of the hand on the midsternum C) Giving 2 breaths followed by 15 compressions D) Using two hands to perform chest compressions 26. A child is undergoing rapid sequence intubation and is receiving atropine. The nurse understands that this agent is used to accomplish which of the following? A) Lessen the vagal effects of intubation B) Reduce intracranial pressure C) Induce amnesia D) Provide short-term paralysis 27. A nurse determines that a child is exhibiting compensated supraventricular tachycardia (SVT). Which of the following would be attempted first? A) Adenosine B) Synchronized cardioversion C) Vagal maneuvers D) Amiodarone Answer Key 1. D 2. B 3. A 4. C 5. B 6. A 7. A 8. C 9. D 10. B 11. A 12. C 13. A 14. B 15. D 16. C 17. D 18. A, C, E, F 19. A, D 20. D 21. B 22. A 23. B 24. C 25. C 26. A 27. C [Show More]

Last updated: 3 weeks ago

Preview 1 out of 316 pages

Reviews( 0 )

$18.00

Add to cart

Instant download

Can't find what you want? Try our AI powered Search

OR

GET ASSIGNMENT HELP
93
0

Document information


Connected school, study & course


About the document


Uploaded On

May 20, 2021

Number of pages

316

Written in

Seller


seller-icon
Kevin

Member since 3 years

0 Documents Sold


Additional information

This document has been written for:

Uploaded

May 20, 2021

Downloads

 0

Views

 93

Document Keyword Tags

Recommended For You


$18.00
What is Browsegrades

In Browsegrades, a student can earn by offering help to other student. Students can help other students with materials by upploading their notes and earn money.

We are here to help

We're available through e-mail, Twitter, Facebook, and live chat.
 FAQ
 Questions? Leave a message!

Follow us on
 Twitter

Copyright © Browsegrades · High quality services·